Med-Surg 2 Exam 4

Pataasin ang iyong marka sa homework at exams ngayon gamit ang Quizwiz!

ANS: A The patient's statement that her lesions are disgusting suggests that disturbed body image is the major concern. There is no evidence to indicate ineffective coping or lack of knowledge about mode of transmission. The patient may be experiencing anxiety, but there is nothing in the data indicating that the genital warts are impacting interpersonal relationships.

A 19-year-old patient has genital warts around her external genitalia and perianal area. She tells the nurse that she has not sought treatment until now because "the warts are so disgusting." Which nursing diagnosis is most appropriate? a. Disturbed body image related to feelings about the genital warts b. Ineffective coping related to denial of increased risk for infection c. Risk for infection related to lack of knowledge about transmission d. Anxiety related to impact of condition on interpersonal relationships

ANS: A Testing for Chlamydia is recommended for all sexually active females under age 25 by the Centers for Disease Control and Prevention. HPV infection does not cause infertility. There is no vaccine available for herpes simplex, and herpes simplex infection does not cause cervical cancer.

A 20-year-old woman who is being seen in the family medicine clinic for an annual physical exam reports being sexually active. The nurse will plan to teach the patient about a. testing for Chlamydia infection. b. immunization for herpes simplex. c. infertility associated with the human papillomavirus (HPV). d. the relationship between the herpes virus and cervical cancer.

ANS: B Because there is a high incidence of co-infection with gonorrhea and chlamydia, patients are usually treated for both. The other explanations about the purpose of the antibiotic combination are not accurate.

A 22-year-old patient with gonorrhea is treated with a single IM dose of ceftriaxone (Rocephin) and is given a prescription for doxycycline (Vibramycin) 100 mg bid for 7 days. The nurse explains to the patient that this combination of antibiotics is prescribed to a. prevent reinfection during treatment. b. treat any coexisting chlamydial infection. c. eradicate resistant strains of N. gonorrhoeae. d. prevent the development of resistant organisms.

After obtaining the health history for a 25-year-old who smokes two packs of cigarettes daily, the nurse will plan to do teaching about the increased risk for a. kidney stones. b. bladder cancer. c. bladder infection. d. interstitial cystitis.

ANS: B Cigarette smoking is a risk factor for bladder cancer. The patient's risk for developing interstitial cystitis, urinary tract infection (UTI), or kidney stones will not be reduced by quitting smoking. DIF: Cognitive Level: Application REF: 1145-1146

A patient who has had a transurethral resection with fulguration for bladder cancer 3 days previously calls the nurse at the urology clinic. Which information given by the patient is most important to report to the health care provider? a. The patient is using opioids for pain. b. The patient has noticed clots in the urine. c. The patient is very anxious about the cancer. d. The patient is voiding every 4 hours at night.

ANS: B Clots in the urine are not expected and require further follow-up. Voiding every 4 hours, use of opioids for pain, and anxiety are typical after this procedure. DIF: Cognitive Level: Application REF: 1145-1146

A 27-year-old patient tells the nurse that she would like a prescription for oral contraceptives to control her premenstrual dysphoric disorder (PMD-D) symptoms. Which patient information is most important to communicate to the health care provider? a. Bilateral breast tenderness b. Frequent abdominal bloating c. History of migraine headaches d. Previous spontaneous abortion

ANS: C Oral contraceptives are contraindicated in patients with a history of migraine headaches. The other patient information would not prevent the patient from receiving oral contraceptives

Which instruction should the nurse provide when teaching a patient to exercise the pelvic floor? A. Tighten both buttocks together. B. Squeeze thighs together tightly. C. Contract muscles around rectum. D. Lie on back and lift legs together.

C. Contract muscles around rectum. To teach pelvic floor exercises, or Kegel exercise, the nurse should instruct the patient (without contracting the legs, buttocks, or abdomen) to contract the muscles around the rectum (pelvic floor muscles) as if stopping a stool, which should result in a pelvic lifting sensation

Which assessment finding for a 33-year-old female patient admitted with Graves' disease requires the most rapid intervention by the nurse? a. Bilateral exophthalmos b. Heart rate 136 beats/minute c. Temperature 103.8° F (40.4° C) d. Blood pressure 166/100 mm Hg

ANS: C The patient's temperature indicates that the patient may have thyrotoxic crisis and that interventions to lower the temperature are needed immediately. The other findings also require intervention but do not indicate potentially life-threatening complications.

prevent the recurrence of renal calculi, the nurse teaches the patient to a. use a filter to strain all urine. b. avoid dietary sources of calcium. c. drink diuretic fluids such as coffee. d. have 2000 to 3000 mL of fluid a day.

ANS: D A fluid intake of 2000 to 3000 mL daily is recommended to help flush out minerals before stones can form. Avoidance of calcium is not usually recommended for patients with renal calculi. Coffee tends to increase stone recurrence. There is no need for a patient to strain all urine routinely after a stone has passed, and this will not prevent stones.

Which information is most important for the nurse to communicate rapidly to the health care provider about a patient admitted with possible syndrome of inappropriate antidiuretic hormone (SIADH)? a. The patient has a recent weight gain of 9 lb. b. The patient complains of dyspnea with activity. c. The patient has a urine specific gravity of 1.025. d. The patient has a serum sodium level of 118 mEq/L.

ANS: D A serum sodium of less than 120 mEq/L increases the risk for complications such as seizures and needs rapid correction. The other data are not unusual for a patient with SIADH and do not indicate the need for rapid action

A 23-year-old patient is admitted with diabetes insipidus. Which action will be most appropriate for the registered nurse (RN) to delegate to an experienced licensed practical/vocational nurse (LPN/LVN)? a. Titrate the infusion of 5% dextrose in water. b. Teach the patient how to use desmopressin (DDAVP) nasal spray. c. Assess the patient's hydration status every 8 hours. d. Administer subcutaneous DDAVP.

ANS: D Administration of medications is included in LPN/LVN education and scope of practice. Assessments, patient teaching, and titrating fluid infusions are more complex skills and should be done by the RN.

A 44-year-old patient in the sexually transmitted infection clinic has a positive Venereal Disease Research Laboratory (VDRL) test, but no chancre is visible on assessment. The nurse will plan to send specimens for a. gram stain. b. cytologic studies. c. rapid plasma reagin (RPR) agglutination. d. fluorescent treponemal antibody absorption (FTA-Abs).

ANS: D Because false positives are common with VDRL and RPR testing, FTA-Abs testing is recommended to confirm a diagnosis of syphilis. Gram staining is used for other sexually transmitted infections (STIs) such as gonorrhea and Chlamydia and cytologic studies are used to detect abnormal cells (such as neoplastic cells).

The client is scheduled to have a renogram (kidney scan). She is concerned about discomfort during the procedure. What is the nurse's best response? A. "Before the test you will be given a sedative to reduce any pain." B. "A local anesthetic agent will be used, so you might feel a little pressure but no pain." C. "Although this test is very sensitive, there is no more discomfort than you would have with an ordinary x-ray." D. "The only pain associated with this procedure is a small needle stick when you are given the radioisotope

ANS: D The test involves an intravenous injection of the radioisotope and the subsequent recording of the emission by a scintillator.

A patient presents to the clinic complaining of symptoms that suggest diabetes. What criteria would support checking blood levels for the diagnosis of diabetes? A) Fasting plasma glucose greater than or equal to 126 mg/dL B) Random plasma glucose greater than 150 mg/dL C) Fasting plasma glucose greater than 116 mg/dL on 2 separate occasions D) Random plasma glucose greater than 126 mg/dL

Ans: A Feedback: Criteria for the diagnosis of diabetes include symptoms of diabetes plus random plasma glucose greater than or equal to 200 mg/dL, or a fasting plasma glucose greater than or equal to 126 mg/dL.

Which diagnostic evaluation is used for a patient with a suspected testicular cancer? A. Prostate-specific antigen (PSA) B. α-Fetoprotein (AFP) C. Complete blood cell count D. Urine and semen analyses

Answer: B Palpation of the scrotal contents is the first step in diagnosing testicular cancer. A cancerous mass is firm and does not transilluminate. Ultrasound of the testes is indicated when testicular cancer (e.g., palpable mass) is suspected or when persistent or painful testicular swelling is present. If testicular cancer is suspected, blood is obtained to determine the serum levels of AFP, lactate dehydrogenase (LDH), and human chorionic gonadotropin (hCG).

For the patient with inoperable prostate cancer, you expect the physician to order which type of hormone? A. Gonadotropin-releasing hormone B. Androgen deprivation C. Luteinizing hormone D. Estrogen

Answer: B Prostate cancer growth largely depends on androgens, and androgen deprivation is a primary therapeutic approach for some men with prostatic cancer. Hormone therapy, also known as androgen-deprivation therapy (ADT), focuses on reducing the levels of circulating androgens to diminish tumor growth.

What is the most common cause of male infertility? A. A hydrocele B. A varicocele C. Alcohol use D. Epididymitis.

Answer: B The most common cause of male infertility is a varicocele, not a hydrocele. Other factors that influence the testes include infection such as epididymitis and alcohol use, but they are not the most common causes.

You expect which finding in a patient as a complication of prostatic hyperplasia? A. Dysuria B. Hematuria C. Urinary retention D. Urinary frequency

Answer: C Obstructive symptoms caused by prostate enlargement include a decrease in the caliber and force of the urinary stream, difficulty in initiating voiding, intermittency (stopping and starting stream several times while voiding), and dribbling at the end of urination. These symptoms result from urinary retention.

In assessing a patient for testicular cancer, you understand that the manifestations of this disease often include A. acute back spasms and testicular pain. B. rapid onset of scrotal swelling and fever. C. fertility problems and bilateral scrotal tenderness. D. painless mass and heaviness sensation in the scrotal area.

Answer: D Clinical manifestations of testicular cancer include a painless lump in the scrotum, scrotal swelling, and a feeling of heaviness. The scrotal mass usually is not tender and is very firm. Some patients complain of a dull ache or heavy sensation in the lower abdomen, perianal area, or scrotum.

A priority nursing diagnosis for the patient with bacterial prostatitis is A. activity intolerance related to fatigue. B. sexual dysfunction related to painful ejaculation. C. deficient fluid volume related to decreased fluid intake. D. impaired urinary elimination related to urethral compression.

Answer: D Acute urinary retention can develop in acute prostatitis, and it requires bladder drainage with suprapubic catheterization. The patient may experience fatigue, but this is not the priority. Sexual dysfunction can occur, but the pain occurs after ejaculation. Fluid volume should be increased.

A 45-year-old man asks you if it is advisable to have his prostate-specific antigen (PSA) level tested, because his father and brother have prostate cancer. What is your response? A. "You should wait until you are age 50." B. "You should have a transurethral resection of the prostate as a preventive measure." C. "You should have a voiding cystourethrogram yearly." D. "You should have annual PSA levels assessed and a digital examination of the prostate."

Answer: D The American Cancer Society recommends an annual digital rectal examination (DRE) and a blood test for PSA beginning at age 50 for men who are at average risk for prostate cancer. During DRE, an abnormal prostate may feel hard, nodular, and asymmetric.

When providing teaching regarding potential complications after perineal resection of the prostate, what should you include? A. Deep vein thrombosis B. Pulmonary embolism C. Colonic constipation D. Urinary incontinence

Answer: D The two major complications after a radical prostatectomy are erectile dysfunction and urinary incontinence.

To minimize the patient experiencing nocturia, the nurse would teach him or her to: A) Perform perineal hygiene after urinating. B) Set up a toileting schedule. C) Double void. D) Limit fluids before bedtime.

Limit fluids before bedtime

What dietary restrictions might be given to a patient diagnosed with acute poststeptococcal glomerulonephritis?

Low-protein, low-sodium, fluid restricted diet.

*4.One of the nruse's most important roles in relation to acute poststreptococcal golmerulonephritis is to* a. promote early diagnosis and treatment of sore throats and skin lesions b.encourage patients to obtain antibiotic therapy for upper respiratory tract infections c.teach patients with APSGN that long term prophylactic antibiotic therapy is necessary to prevent recurrence d.monitor patients for respiratory symptoms that indicate the disease is affecting the alveolar basement membrane

a. promote early diagnosis and treatment of sore throats and skin lesions

*9. In planning nursing interventions to increase bladder control in the patient with urinary incontinence, the nurse includes:* a. teaching the patient to use Kegel exercises b.clamping and releasing a catheter to increase bladder tone c.teaching the patient biofeedback mechanisms to suppress the urge to void d.counseling the patient concerning choices of incontinence containment devices

a. teaching the patient to use Kegel exercises

*10. A patient with ureterolithotomy returns from surgery with a nephrostomy tube in place. Postoperative nursing care of the patient includes:* a.encourage the patient to drink fruit juices and milk b.encouraging fluids of at least 2-3 L/day after nausea has subsided c. irrigating the nephrostomy tube with 10ml of NS solution as needed d. notifying the physician if nephrostomy tube drainage is more than 30ml/hr

b.encouraging fluids of at least 2-3 L/day after nausea has subsided

*8. The nurse identifies a risk factor for kidney and bladder cancer in a patient who relates a history of* a.aspirin use b.tobacco use c.chronic alcohol abuse d.use of artificial sweeteners

b.tobacco use

*2. The nurse teaches the female paitent who has frequent UTIs that she should* a. take tub baths with bubble bath b.urinate before and after sexual intercourse c.take prophylactic sufonamides for the rest of her life d. restrict fluid intake to prevent the need for frequent voiding

b.urinate before and after sexual intercourse

The RN directs the LPN/LVN to remove a Foley catheter at 1300. The nurse would check if the patient has voided by: A) 1400. B) 1600 C) 1700. D) 2300.

1700.

Which of the following female patients should report her results from breast self-examination? A. Denser breast tissue B. Left nipple deviation C. Palpable rib margins D. Different size breasts

B. Unilateral deviation of a nipple and nipple retraction are clinical indicators of breast cancer and should be reported promplty to the HCP. The breasts of most women are slightly different sizes

The nurse is volunteering at a community center to teach women regarding breast cancer. The nurse would include which of the following when discussing risk factors (select all that apply)? A. Nulliparity B. Age 30 or over C. Early menarche D. Late menopause E. Personal history of colon cancer

ACDE. Women are at an increased risk for development of breast cancer if they are over the age of 50; have a family history of breast cancer; have a personal history of breast, colon, endometrial, or ovarian cancer; have a long menstrual history as seen with early menarche or late menopause; and have had a first full-term pregnancy after the age of 30 or are nulliparous.

A patient is to receive methylprednisolone (Solu-Medrol) 100 mg. The label on the medication states: methylprednisolone 125 mg in 2 mL. How many milliliters will the nurse administer?

ANS: 1.6 A concentration of 125 mg in 2 mL will result in 100 mg in 1.6 mL.

Which prescribed medication should the nurse administer first to a 60-year-old patient admitted to the emergency department in thyroid storm? a. Propranolol (Inderal) b. Propylthiouracil (PTU) c. Methimazole (Tapazole) d. Iodine (Lugol's solution)

ANS: A Adrenergic blockers work rapidly to decrease the cardiovascular manifestations of thyroid storm. The other medications take days to weeks to have an impact on thyroid function.

ANS: C With expedited partner therapy, the patient is given a prescription or medications for the partner. The partner does not need to be evaluated by the health care provider, but is presumed to be infected and should be treated concurrently with the patient. Use of a condom will not treat the presumed STI in the partner.

After the nurse has taught a patient with a newly diagnosed sexually transmitted infection about expedited partner therapy, which patient statement indicates that the teaching has been effective? a. "I will tell my partner that it is important to be examined at the clinic." b. "I will have my partner take the antibiotics if any STI symptoms occur." c. "I will make sure that my partner takes all of the prescribed medication." d. "I will have my partner use a condom until I have finished the antibiotics."

Nursing management for nephrotic syndrome?

Angiotensin-converting enzyme inhibitors (water retaining); nonsteriodal antiinflammatory drugs (NSAIDS); and low sodium (2-3 g/day), moderate protein diet. salt restrictions for edema or thiazide or loop diuretics. For the treatment of hyperlipidemia includes lipid-loweing agents, such as colestripol (Colestid) and lovastatin (Mevacor). Assess edema by weighing the patient daily, recording intake and output, and measuring abdominal girth or extremity size. Serve small, frequent meals in a pleasent setting to encourage better deitary intake.

A nurse is conducting a class on how to self-manage insulin regimens. A patient asks how long a vial of insulin can be stored at room temperature before it "goes bad." What would be the nurse's best answer? A) "If you are going to use up the vial within 1 month it can be kept at room temperature." B) "If a vial of insulin will be used up within 21 days, it may be kept at room temperature." C) "If a vial of insulin will be used up within 2 weeks, it may be kept at room temperature." D) "If a vial of insulin will be used up within 1 week, it may be kept at room temperature."

Ans: A Feedback: If a vial of insulin will be used up within 1 month, it may be kept at room temperature.

A diabetic nurse is working for the summer at a camp for adolescents with diabetes. When providing information on the prevention and management of hypoglycemia, what action should the nurse promote? A) Always carry a form of fast-acting sugar. B) Perform exercise prior to eating whenever possible. C) Eat a meal or snack every 8 hours. D) Check blood sugar at least every 24 hours.

Ans: A Feedback: The following teaching points should be included in information provided to the patient on how to prevent hypoglycemia: Always carry a form of fast-acting sugar, increase food prior to exercise, eat a meal or snack every 4 to 5 hours, and check blood sugar regularly.

A 15-year-old child is brought to the emergency department with symptoms of hyperglycemia and is subsequently diagnosed with diabetes. Based on the fact that the child's pancreatic beta cells are being destroyed, the patient would be diagnosed with what type of diabetes? A) Type 1 diabetes B) Type 2 diabetes C) Non-insulin-dependent diabetes D) Prediabetes

Ans: A Feedback: Beta cell destruction is the hallmark of type 1 diabetes. Non-insulin-dependent diabetes is synonymous with type 2 diabetes, which involves insulin resistance and impaired insulin secretion, but not beta cell destruction. Prediabetes is characterized by normal glucose metabolism, but a previous history of hyperglycemia, often during illness or pregnancy.

The patient is incontinent, and a condom catheter is placed. The nurse should take which action? A) Secure the condom with adhesive tape B) Change the condom every 48 hours C) Assess the patient for skin irritation D) Use sterile technique for placement

Assess the patient for skin irritation

A 22 year old female patient has come to the clinic for oral contraceptives. The nurse uses the opportunity for patient teaching about BSE. What shoudl the nurse include in patient teaching? A. Women at high risk for breast cancer begin BSE at 3o years old B. Perform BSE monthly on the day of the first pill in each package C. Older women use BSE annually as breast tissue density declines D. Healthy women need to have annual mammograms beginning at age 50

B. The nurse instructs the pt to use the packageing for the oral contraceptives as a convienient reminder to perform BSE monthly to increase adherence

Which benign breast disorder is the most similar to breast cancer? A. Ductal ectasia B. Fibroadenoma C. Breasat abscess D. Cyclic mastalgia

C. A breast abscess is a benign breast disorder most likely to be confused with a malignany breast lesion because it shares moe characteristics of breast cancers than other benign breast disorders. A breast abscess can be red and edematous in apperance characteristic of inflammatory breast cancer. The abscess also is likely to be a palpable mass, a potential characteristic of breast cancer.

The patient called the clinic with manifestations of burning on urination, dysuria, and frequency. What is the best advice for the nurse to give the patient? A. "Drink less fluid so you don't have to void so often." B. "Take some acetaminophen to decrease the discomfort." C. "Come in so we can check a clean catch urine specimen." D. "Avoid caffeine and spicy food to decrease inflammation."

C. "Come in so we can check a clean catch urine specimen." The patient's symptoms are typical of a urinary tract infection (UTI). To verify this, a clean catch urine specimen must be obtained for a specimen of urine to culture. Drinking less fluid will not improve the symptoms. Acetaminophen would not decrease the discomfort; an antibiotic would be needed. Avoiding caffeine and spicy food may decrease bladder inflammation but will not affect these symptoms.

The patient has scleroderma and is experiencing hypertension. The nurse should know that this could be related to which renal problem? A. Obstructive uropathy B. Goodpasture syndrome C. Chronic glomerulonephritis D. Calcium oxalate urinary calculi

C. Chronic glomerulonephritis Hypertension occurs with chronic glomerulonephritis that may be found in patients with scleroderma. Obstructive uropathy, Goodpasture syndrome, and calcium oxalate urinary calculi are not related to scleroderma and do not cause hypertension.

Since removal of the patient's Foley catheter, the patient has voided 50 to 100 mL every 2 to 3 hours. Which action should the nurse take first? A) Check for bladder distention B) Encourage fluid intake C) Obtain an order to recatheterize the patient D) Document the amount of each voiding for 24 hours

Check for bladder distention

The nurse notes that the patient's Foley catheter bag has been empty for 4 hours. The priority action would be to: A) Irrigate the Foley. B) Check for kinks in the tubing. C) Notify the health care provider. D) Assess the patient's intake.

Check for kinks in the tubing.

The nurse is providing care for a patient who has been admitted to the hospital for the treatment of nephrotic syndrome. What are priority nursing assessments in the care of this patient? A. Assessment of pain and level of consciousness B. Assessment of serum calcium and phosphorus levels C. Blood pressure and assessment for orthostatic hypotension D. Daily weights and measurement of the patient's abdominal girth

D. Daily weights and measurement of the patient's abdominal girth Peripheral edema is characteristic of nephrotic syndrome, and a key nursing responsibility in the care of patients with the disease is close monitoring of abdominal girth, weights, and extremity size. Pain, level of consciousness, and orthostatic blood pressure are less important in the care of patients with nephrotic syndrome. Abnormal calcium and phosphorus levels are not commonly associated with the diagnosis of nephrotic syndrome.

A.Decreased weight B. Increased appetite C.Increased urinary output D. Elevated creatinine leve

D. Elevated creatinine level Gentamicin can be toxic to the kidneys and the auditory system. The elevated creatinine level must be reported to the physician as it probably indicates renal damage. Other factors that may occur with renal damage would include increased weight and decreased urinary output. Many medications have side effects of anorexia.

The nurse coordinates postoperative care for a 70-year-old man with osteoarthritis after prostate surgery. Which task is appropriate for the nurse to delegate to a licensed practical/vocational nurse (LPN/LVN)? A. Teach the patient how to perform Kegel exercises. B. Provide instructions to the patient on catheter care. C. Administer oxybutynin (Ditropan) for bladder spasms. D. Manually irrigate the urinary catheter to determine patency.

D. Manually irrigate the urinary catheter to determine patency. The nurse may delegate the following to an LPN/LVN: monitor catheter drainage for increased blood or clots, increase flow of irrigating solution to maintain light pink color in outflow, administer antispasmodics and analgesics as needed. A registered nurse may not delegate teaching, assessments, or clinical judgments to a LPN/LVN.

What is the most important way to prevent Acute poststeptococcal glomerulonephritis?

Encourage early diagnosis and treatment of sore throat and skin lesions.

A patient with a Foley catheter carries the collection bag at waist level when ambulating. The nurse tells the patient that he or she is at risk for: (Select all that apply.) A) Infection. B) Retention. C) Stagnant urine. D) Reflux of urine.

Infection. Reflux of urine.

An older male patient states that he is having problems starting and stopping his stream of urine and he feels the urgency to void. The best way to assist this patient is to: A) Help him stand to void. B) Place a condom catheter. C) Have him practice Credé's method. D) Initiate Kegel exercises.

Initiate Kegel exercises.

The patient is to have an intravenous pyelogram (IVP). Which of the following apply to this procedure? (Select all that apply.) A) Note any allergies. B) Monitor intake and output. C) Provide for perineal hygiene. D) Assess vital signs. E) Encourage fluids after the procedure.

Note any allergies. Encourage fluids after the procedure.

ANS: A, B, D The HPV vaccines are recommended for male and female patients between ages 9 through 26. Ideally, the vaccines are administered before patients are sexually active, but they offer benefit even to those who already have HPV infection.

The nurse in the outpatient clinic notes that the following patients have not received the human papillomavirus (HPV) vaccine. Which patients should the nurse plan to teach about benefits of the vaccine (select all that apply)? a. 24-year-old man who has a history of genital warts b. 18-year-old man who has had one male sexual partner c. 28-year-old woman who has never been sexually active d. 20-year-old woman who has a newly diagnosed Chlamydia infection e. 30-year-old woman whose sexual partner has a history of genital warts

The postoperative patient has difficulty voiding after surgery and is feeling "uncomfortable" in the lower abdomen. Which action should the nurse implement first? A) Encourage fluid intake B) Administer pain medication C) Catheterize the patient D) Turn on the bathroom faucet as he tries to void

Turn on the bathroom faucet as he tries to void

The nurse assesses that the patient has a full bladder, and the patient states that he or she is having difficulty voiding. The nurse would teach the patient to: A) Use the double-voiding technique. B) Perform Kegel exercises. C) Use Credé's method. D) Keep a voiding diary.

Use Credé's method.

ANS: C A common reason for recurrence of symptoms is reinfection because infected partners have not been simultaneously treated. Because gonorrhea is treated with one dose of antibiotic, antibiotic therapy for a week is not needed. An adequate fluid intake is important, but a low fluid intake is not a likely cause for failed treatment. Poor hygiene may cause complications such as ocular trachoma but will not cause a failure of treatment.

When a 31-year-old male patient returns to the clinic for follow-up after treatment for gonococcal urethritis, a purulent urethral discharge is still present. When trying to determine the reason for the recurrent infection, which question is most appropriate for the nurse to ask the patient? a. "Did you take the prescribed antibiotic for a week?" b. "Did you drink at least 2 quarts of fluids every day?" c. "Were your sexual partners treated with antibiotics?" d. "Do you wash your hands after using the bathroom?"

ANS: A Complications of gonorrhea include scarring of the fallopian tubes, which can lead to tubal pregnancies and infertility. Syphilis, genital warts, and genital herpes do not lead to problems with conceiving, although transmission to the fetus (syphilis) or newborn (genital warts or genital herpes) is a concern.

Which infection, reported in the health history of a woman who is having difficulty conceiving, will the nurse identify as a risk factor for infertility? a. N. gonorrhoeae b. Treponema pallidum c. Condyloma acuminatum d. Herpes simplex virus type 2

ANS: A Gardasil is recommended for females ages 9 through 26, preferably those who have never been sexually active. It is not recommended for women during pregnancy or for older women.

Which patient will the nurse plan on teaching about the Gardasil vaccine? a. A 24-year-old female who has not been sexually active b. A 34-year-old woman who has multiple sexual partners c. A 19-year-old woman who is pregnant for the first time d. A 29-year-old woman who is in a monogamous relationship

ANS: C The treatment regimen for primary genital herpes infections includes acyclovir 400 mg 3 times daily for 7 to 10 days. The patient is taught to abstain from intercourse until the lesions are gone. (Condoms should be used even when the patient is asymptomatic.) Acyclovir ointment is not effective in treating lesions or reducing pain. Herpes infection is chronic and recurrent.

Which statement by a 24-year-old patient indicates that the nurse's teaching about management of primary genital herpes has been effective? a. "I will use acyclovir ointment on the area to relieve the pain." b. "I will use condoms for intercourse until the medication is all gone." c. "I will take the acyclovir (Zovirax) every 8 hours for the next week." d. "I will need to take all of the medication to be sure the infection is cured."

*6. A patient is admitted to the hospital with severe renal colic. The nurse's first priority in management of the patient is to* a. administer opioids as prescribed. b. obtain supplies for straining all urine c. encourage fluid intake of 3-4L/day d. keep the patient NPO in preparation for surgery

a. administer opioids as prescribed.

Acute postreptococcal glomerulonephritis is most common in

children and young adults. APSGN develops 5-21 days after an infection of the tonsils, pharynx, or skin.

clinical manifestations of glomerulonephritis

body edema, hypertension, oliguria, hematuria with smoky or rusty appearance, and proteinuria. (fluid retention occurs as a result of decreased glomerular filtration; periorbital edema may later progress to full body edema or peripheral edema in the legs; smoky urine indicates bleeding; hypertension results fro increased extracellular fluid volume.

*7. The nurse recommends genetic counseling for the children of a patient with* a.nephrotic syndrome b.chronic pyelonephritis c. malignant nephrosclerosis d.adult onset polycystic kidney disease

d.adult onset polycystic kidney disease

*5.The edema that occurs in nephrotic syndrome is due to* a. increased hydrostatic pressure caused by sodium retention. b. decreased aldosterone secretion from adrenal insufficiency. c. increased fluid retention caused by decreased glomerular filtration d.decreased colloidal osmotic pressure caused by loss of serum albumin

d.decreased colloidal osmotic pressure caused by loss of serum albumin

Clinical manifestations of good pasture syndrome include?

cough, shortness of breath, hemoptysis, crackles, rhonchi, hematuria, weakness, pallor, anemia, and renal failure. Pulmonary hemorrhage usually occurs.

*1. In teaching a patient with pyelonephritis about the disorder, the nurse informs the paitent that the organisms that cause pyelonephritis most commonly reach the kidneys through* a. the bloodstream b.the lymphatic system c. a descending infection d. an ascending infection

d. an ascending infection

*11. A patient has has a cystectomy and ileal conduit diversion performed. Four days postoperatively, mucous shred are seen in the drainage bag. The nurse should.* a. notify the physician b.notify the charge nurse c. irrigate the drainage tube d. chart it as a normal observation

d. chart it as a normal observation

Patient with nephrotic syndrome is at great risk for?

infection, and thromboembolism.

Goodpasture syndrome

is an autoimmune disease charcterized by circulating antibodies against glomerular and alveolar basement membrane.

Clinical manifestations of nephrotic syndrome?

peripheral, periorbital, and facial edema; massive proteinuria, hypotention (hypertention lewis); hyperlipidemia, and hypeoalbuminemia. Ascites (fluid in abdominal cavity), and anasarca (massive generalized edema). Lethargy, anorexia, and pallor. and weight gain.

nephrotic syndrome

results when the glomerulus is excessiviley permeable to plasma protein, causing proteinuria that leads to low plasma albumin and tissue edema.

Goodpasture syndrome is mostly found in

young male smokers.

The patient states that she "loses urine" every time she laughs or coughs. The nurse teaches the patient measures to regain urinary control. The nurse recognizes the need for further teaching when the patient states: A) "I will perform my Kegel exercises every day." B) "I joined weight watchers." C) "I drink two glasses of wine with dinner." D) "I have tried urinating every 3 hours."

"I drink two glasses of wine with dinner."

After a transurethral prostatectomy a patient returns to his room with a triple-lumen indwelling catheter and continuous bladder irrigation. The irrigation is normal saline at 150 mL/hr. The nurse empties the drainage bag for a total of 2520 mL after an 8-hour period. How much of the total is urine output?

1320

The nurse will plan to teach the female patient with genital warts about the a. importance of regular Pap tests. b. increased risk for endometrial cancer. c. appropriate use of oral contraceptives. d. symptoms of pelvic inflammatory disease (PID).

ANS: A Genital warts are caused by the human papillomavirus (HPV) and increase the risk for cervical cancer. There is no indication that the patient needs teaching about PID, oral contraceptives, or endometrial cancer

Which assessment maneuvers should the nurse perform first when assessing the renal system at the same time as the abdomen? A. Abdominal percussion B. Abdominal auscultation C. Abdominal palpation D. Renal palpation

ANS: B Auscultation precedes percussion and palpation because the nurse needs to auscultate for abdominal bruits before palpation or percussion of the abdominal and renal components of a physical assessment.

The female client's urinalysis shows all the following characteristics. Which should the nurse document as abnormal? A. pH 5.6 B. Ketone bodies present C. Specific gravity is 1.030 D. Two white blood cells per high-power field

ANS: B Ketone bodies are byproducts of incomplete metabolism of fatty acids. Normally, there are no ketones in urine. Ketone bodies are produced when fat sources are used instead of glucose to provide cellular energy.

A 28-year-old pregnant woman is spilling sugar in her urine. The physician orders a glucose tolerance test, which reveals gestational diabetes. The patient is shocked by the diagnosis, stating that she is conscientious about her health, and asks the nurse what causes gestational diabetes. The nurse should explain that gestational diabetes is a result of what etiologic factor? A) Increased caloric intake during the first trimester B) Changes in osmolality and fluid balance C) The effects of hormonal changes during pregnancy D) Overconsumption of carbohydrates during the first two trimesters

A 28-year-old pregnant woman is spilling sugar in her urine. The physician orders a glucose tolerance test, which reveals gestational diabetes. The patient is shocked by the diagnosis, stating that she is conscientious about her health, and asks the nurse what causes gestational diabetes. The nurse should explain that gestational diabetes is a result of what etiologic factor? A) Increased caloric intake during the first trimester B) Changes in osmolality and fluid balance C) The effects of hormonal changes during pregnancy D) Overconsumption of carbohydrates during the first two trimesters

ANS: A Sex partners should be treated simultaneously to prevent reinfection. Chlamydia is treated with oral antibiotics. Abstinence from sexual intercourse is recommended for 7 days after treatment, and condoms should be recommended during all sexual contacts to prevent infection. Chronic pelvic pain, as well as infertility, can result from untreated Chlamydia.

A 29-year-old female patient is diagnosed with Chlamydia during a routine pelvic examination. The nurse knows that teaching regarding the management of the condition has been effective when the patient says which of the following? a. "My partner will need to take antibiotics at the same time I do." b. "Go ahead and give me the antibiotic injection, so I will be cured." c. "I will use condoms during sex until I finish taking all the antibiotics." d. "I do not plan on having children, so treating the infection is not important."

ANS: D Information about sexual contacts is needed to help establish whether the patient has been exposed to a sexually transmitted infection (STI) and because sexual contacts also will need treatment. The other information also may be gathered but is not as important in determining the plan of care for the patient's current symptoms.

A 32-year-old man who has a profuse, purulent urethral discharge with painful urination is seen at the clinic. Which information will be most important for the nurse to obtain? a. Contraceptive use b. Sexual orientation c. Immunization history d. Recent sexual contacts

ANS: B This response expresses the nurse's acceptance of the patient's feelings and encourages further discussion and problem solving. The patient may need professional counseling, but more assessment of the patient is needed before making this judgment. The nurse should also assess further before suggesting that the patient share her feelings with the husband because problems such as abuse might be present in the relationship. Although it is important that both partners be treated, the patient's current anger suggests that this is not the appropriate time to bring this up.

A 32-year-old woman who is diagnosed with Chlamydia tells the nurse that she is very angry because her husband is her only sexual partner. Which response should the nurse make first? a. "You may need professional counseling to help resolve your anger." b. "It is understandable that you are angry with your husband right now." c. "Your feelings are justified and you should share them with your husband." d. "It is important that both you and your husband be treated for the infection."

ANS: A, D, E The patient's clinical manifestations and laboratory tests are consistent with tertiary syphilis. Valvular insufficiency, gummas, and changes in mentation are other clinical manifestations of this stage.

A 39-year-old patient with a history of IV drug use is seen at a community clinic. The patient reports difficulty walking, stating "I don't know where my feet are." Diagnostic screening reveals positive Venereal Disease Research Laboratory (VDRL) and fluorescent treponemal antibody absorption (FTA-Abs) tests. Based on the patient history, what will the nurse assess (select all that apply)? a. Heart sounds b. Genitalia for lesions c. Joints for swelling and inflammation d. Mental state for judgment and orientation e. Skin and mucous membranes for gummas

ANS: A Once antibody testing is positive for syphilis, the antibodies remain present for an indefinite period of time even after successful treatment, so the nurse should inquire about previous treatment before doing other assessments or testing. Culture, FAT-Abs testing, and assessment for symptoms may be appropriate, based on whether the patient has been previously treated for syphilis.

A 46-year-old man who has had blood drawn for an insurance screening has a positive Venereal Disease Research Laboratory (VDRL) test. Which action should the nurse take next? a. Ask the patient about past treatment for syphilis. b. Explain the need for blood and spinal fluid cultures. c. Obtain a specimen for fluorescent treponemal antibody absorption (FAT-Abs) testing. d. Assess for the presence of chancres, flulike symptoms, or a bilateral rash on the trunk.

ANS: B Many women with gonorrhea are asymptomatic or have minor symptoms that are overlooked. The disease may affect both the genitals and the other reproductive organs and cause complications such as pelvic inflammatory disease (PID). Women who can transmit the disease have active infections.

A 48-year-old male patient who has been diagnosed with gonococcal urethritis tells the nurse he had recent sexual contact with a woman but says she did not appear to have any disease. In responding to the patient, the nurse explains that a. women do not develop gonorrhea infections but can serve as carriers to spread the disease to males. b. women may not be aware they have gonorrhea because they often do not have symptoms of infection. c. women develop subclinical cases of gonorrhea that do not cause tissue damage or clinical manifestations. d. when gonorrhea infections occur in women, the disease affects only the ovaries and not the genital organs.

ANS: D NAAT has a high sensitivity (similar to a culture) for gonorrhea. Because women have few symptoms of gonorrhea, asking the patient about symptoms may not be helpful in making a diagnosis. Smears and Gram staining are not useful because the female genitourinary tract has many normal flora that resemble N. gonorrhoeae. RPR testing is used to detect syphilis.

A 55-year-old woman in the sexually transmitted infection (STI) clinic tells the nurse that she is concerned she may have been exposed to gonorrhea by her partner. To determine whether the patient has gonorrhea, the nurse will plan to a. interview the patient about symptoms of gonorrhea. b. take a sample of cervical discharge for Gram staining. c. draw a blood specimen or rapid plasma reagin (RPR) testing. d. obtain secretions for a nucleic acid amplification test (NAAT).

ANS: B Exudate from any lesions with syphilis is highly contagious. Systemic antibiotics, rather than local treatment of lesions, are used to treat syphilis. The patient does not require a private room because the disease is spread through contact with the lesions. This patient has clinical manifestations of secondary syphilis and does not need to be monitored for manifestations of tertiary syphilis.

A patient admitted with chest pain is also found to have positive Venereal Disease Research Laboratory (VDRL) and fluorescent treponemal antibody absorption (FAT-Abs) tests, rashes on the palms and the soles of the feet, and moist papules in the anal and vulvar area. Which action will the nurse include in the plan of care? a. Assess for arterial aneurysms. b. Wear gloves for patient contact. c. Place the patient in a private room. d. Apply antibiotic ointment to the perineum.

ANS: C A single injection of penicillin is recommended to treat primary syphilis. This will treat the mother and prevent transmission of the disease to the fetus. Instillation of erythromycin into the eyes of the newborn is used to prevent gonorrheal eye infections. C-section is used to prevent the transmission of herpes to the newborn. Although stillbirth can occur if the fetus is infected with syphilis, treatment before the tenth week of gestation will eliminate in utero transmission to the fetus.

A woman is diagnosed with primary syphilis during her eighth week of pregnancy. The nurse will plan to teach the patient about the a. likelihood of a stillbirth. b. plans for cesarean section c. intramuscular injection of penicillin. d. antibiotic eye drops for the newborn.

A female patient reports that she is experiencing burning on urination, frequency, and urgency. The nurse notes that a clean-voided urine specimen is markedly cloudy. The probable cause of these symptoms and findings is: A) Cystitis. B) Hematuria. C) Pyelonephritis. D) Dysuria.

A) Cystitis.

Elimination changes that result from inability of the bladder to empty properly may cause which of the following? (Select all that apply.) A) Incontinence B) Frequency C) Urgency D) Urinary retention E) Urinary tract infection

A) Incontinence B) Frequency C) Urgency D) Urinary retention E) Urinary tract infection

A male patient returned from the operating room 6 hours ago with a cast on his right arm. He has not yet voided. Which action would be the most beneficial in assisting the patient to void? A) Suggest he stand at the bedside B) Stay with the patient C) Give him the urinal to use in bed D) Tell him that, if he doesn't urinate, he will be catheterized

A) Suggest he stand at the bedside

Select the results (in italics) that are normal in a urinalysis. A. pH 6 B. Specific gravity 1.015 C. Protein small D. Sugar negative E. Nitrate small F. Leukocyte esterase positive G. Bilirubin negative

A, B, D, G Rationale: The abnormal values are indicative of a urinary tract infection. As a result of protein, nitrates, and leukoesterase in the urine, the nurse can expect the laboratory to analyze microscopic sediment including evaluating the sample for the presence of crystals, casts, WBCs, and RBCs.

A patient who has stage IIB breast cancer has a left mastectomy with axillary node dissection. Ten lymph nodes are resected and three contain malignant cells. What is teh best goal for the nurse to use in planning care? A Assess the pt's response to pathology report B. Maintain joint flexibility and l. arm function B. Prevent L. arm lymphatic fluid accumulation D. Prepare her for chemo and radiation therapy

A. Because the nurse acts as the pt. advocate and because this time is an extremely stressful period for the patient and family, the nurses initial plan focuses on the patients response to the pathology report as part of a comprhensive patient assessment.

What is the nurse's priority assessment in a 41 year old female paitent who reports a lump in the upper, outer margin of the right breast? A. Related pain B. Family history C. Onset of mass D. Characteristics

A. Breast cancer is usually associated with nonpainful and nontender lesions of the breast, so this information is very important in assessing breast masses

What should the nurse include in patient teaching when preparing a 45 year old patient for a routine mammography? A. Additional xrays can be needed for dense breast tissue B. Ultrasound confirms the presence of nonvisible mass C. Early detection of malignany tumore prevents mettastasis D. Continue to obtain clinical breast examinations every 3 years

A. Dense breast tissue on mammography can appear as a suspicious region of the breast. Once identified, additional mammograms can target the region for further examination

Two years after breast conserving surgery for a malignany left breast tumor, the nurse assesses the patient. Which patient assessment data is a priority? A. Irregular, fixed mass i the left axilla B. Temperature, HR, lung sounds C. Left upper extremity persistent edema D. Keloid formation around wound edges

A. The most frequent site of breast cancer recurrence is the site of the original tumor, so the presence of an irregular and fixed mass in the L. axilla is the nurse priority. Fixed and irregular are characteristics of a malignant breast tumor and require follow up care

A patient with a history of recurrent urinary tract infections has been scheduled for a cystoscopy. What teaching point should the nurse emphasize before the procedure? A. "You might have pink-tinged urine and burning after your cystoscopy." B. "You'll need to refrain from eating or drinking after midnight the day before the test." C. "You'll require a urinary catheter inserted before the cystoscopy, and it will be in place for a few days." D. "The morning of the test, the nurse will ask you to drink some water that contains a contrast solution."

A. "You might have pink-tinged urine and burning after your cystoscopy." Pink-tinged urine, burning, and frequency are common following a cystoscopy. The patient does not need to be NPO prior to the test, and contrast media is not needed. A cystoscopy does not always necessitate catheterization before or after the procedure.

After a vasectomy, what teaching should be included in the discharge teaching? A. "You will want to use an alternative form of contraception for 6 weeks." B. "You may lose some secondary sexual characteristics after this surgery." C. "You may have erectile dysfunction for several months after this surgery." D. "You will be uncomfortable, but you may safely have sexual intercourse today."

A. "You will want to use an alternative form of contraception for 6 weeks." As vasectomies are usually done for sterilization purposes, to safely have sexual intercourse, the patient will need to use an alternative form of contraception until semen examination reveals no sperm, usually at least 10 ejaculations or 6 weeks to evacuate sperm distal to the surgical site. Hormones are not affected, so there is no loss of secondary sexual characteristics or erectile function. Most men experience too much pain to have sexual intercourse on the day of their surgery, so this is not an appropriate comment by the nurse.

The nurse is teaching clinic patients about risk factors for testicular cancer. Which individual is at highest risk for developing testicular cancer? A. A 30-year-old white male with a history of cryptorchidism B. A 48-year-old African American male with erectile dysfunction C. A 19-year-old Asian male who had surgery for testicular torsion D. A 28-year-old Hispanic male with infertility caused by a varicocele

A. A 30-year-old white male with a history of cryptorchidism The incidence of testicular cancer is four times higher in white males than in African American males. Testicular tumors are also more common in males who have had undescended testes (cryptorchidism) or a family history of testicular cancer or anomalies. Other predisposing factors include orchitis, human immunodeficiency virus infection, maternal exposure to DES, and testicular cancer in the contralateral testis.

Which nursing diagnosis is a priority in the care of a patient with renal calculi? A. Acute pain B. Risk for constipation C. Deficient fluid volume D. Risk for powerlessness

A. Acute Pain Urinary stones are associated with severe abdominal or flank pain. Deficient fluid volume is unlikely to result from urinary stones, whereas constipation is more likely to be an indirect consequence rather than a primary clinical manifestation of the problem. The presence of pain supersedes powerlessness as an immediate focus of nursing care.

What should the nurse expect to do to prepare a patient for an intravenous pyelogram (IVP)? A. Administer a cathartic or enema. B. Assess patient for allergies to penicillin. C. Keep the patient NPO for 4 hours preprocedure. D. Advise the patient that a metallic taste may occur during procedure.

A. Administer a cathartic or enema. Nursing responsibilities in caring for a patient undergoing an IVP include administration of a cathartic or enema to empty the colon of feces and gas. The nurse will also assess the patient for iodine sensitivity, keep the patient NPO for 8 hours preprocedure, and advise the patient that warmth, a flushed face, and a salty taste during injection of contrast material may occur.

The nurse is caring for a 62-year-old man after a transurethral resection of the prostate (TURP). Which instructions should the nurse include in the teaching plan? A. Avoid straining during defecation. B. Restrict fluids to prevent incontinence. C. Sexual functioning will not be affected. D. Prostate exams are not needed after surgery.

A. Avoid straining during defecation. Activities that increase abdominal pressure, such as sitting or walking for prolonged periods and straining to have a bowel movement (Valsalva maneuver), should be avoided in the postoperative recovery period to prevent a postoperative hemorrhage. Instruct the patient to drink at least 2 L of fluid every day. Digital rectal examinations should be performed yearly. The prostate gland is not totally removed and may enlarge after a TURP. Sexual functioning may change after prostate surgery. Changes may include retrograde ejaculation, erectile dysfunction, and decreased orgasmic sensation.

The patient with type 2 diabetes has a second UTI within one month of being treated for a previous UTI. Which medication should the nurse expect to teach the patient about taking for this infection? A. Ciprofloxacin (Cipro) B. Fosfomycin (Monurol) C. Nitrofurantoin (Macrodantin) D. Trimethoprim/sulfamethoxazole (Bactrim)

A. Ciprofloxacin (Cipro) This UTI is a complicated UTI because the patient has type 2 diabetes and the UTI is recurrent. Ciprofloxacin (Cipro) would be used for a complicated UTI. Fosfomycin (Monurol), nitrofurantoin (Macrodantin), and trimethoprim/sulfamethoxazole (Bactrim) should be used for uncomplicated UTIs.

A pt. who had breast cancer is now having breast reconstructive surgery. To enhance the quality of the reconstructive surgery, what is the nurse's priority patient assessment when the patient returns for enlargement of the expander? A. Color of the affected skin B. Attitiude on reconstruction C. Desired size of new breast D. Tenderness near expander

A. Color of the affected skin. The nurses priority assessment is the color of the skin over and surrounding the expander because abnormal skin color can indicate poor perfusion to the region and risk the viability to the tissue. This is the nurses priority because the purpose of teh expander is to stretch the patiesn skin that will eventually cover the breast implant. The skin over the expander should be pink, warm, and supple. Skin with poor perfusion can look dusky and pale and be cool and friable.

Which of the following factors are poor prognostic indicators of breast cancer (select all that apply)? A. Previous breast cancer B. Well-differentiated tumor C. High incidence of aneuploidy D. Estrogen and progesterone receptor positive E. Many cells in synthesis phase of cell cycle

A. History of previous breast cancer increases the risk of recurrence. Tumors that are well differentiated tend to be less affressive than undifferentiated tumors. Tumors unreponsive to hormonal therapy tend to be more aggressive

What is the nurse's priority when changing the appliance of a patient with an ileal conduit? A. Keep the skin free of urine. B. Inspect the peristomal area. C. Cleanse and dry the area gently. D. Affix the appliance to the faceplate.

A. Keep the skin free of urine The nurse's priority is to keep the skin free of urine because the peristomal skin is at high risk for damage from the urine if it is alkaline. The peristomal area will be assessed; the area will be gently cleaned and dried, and the appliance will be affixed to the faceplate if one is being used, but these are not as much of a priority as keeping the skin free of urine to prevent skin damage.

Eight months after the delivery of her first child, a 31-year-old woman has sought care because of occasional incontinence that she experiences when sneezing or laughing. Which measure should the nurse first recommend in an attempt to resolve the woman's incontinence? A. Kegel exercises B. Use of adult incontinence pads C. Intermittent self-catheterization D. Dietary changes including fluid restriction

A. Kegel exercises Patients who experience stress incontinence frequently benefit from Kegel exercises (pelvic floor muscle exercises). The use of incontinence pads does not resolve the problem, and intermittent self-catheterization would be a premature recommendation. Dietary changes are not likely to influence the patient's urinary continence

The urinalysis of a male patient reveals a high microorganism count. What data should the nurse use to determine the area of the urinary tract that is infected (select all that apply)? A. Pain location B. Fever and chills C. Mental confusion D. Urinary hesitancy E. Urethral discharge F. Post-void dribbling

A. Pain location E. Urethral discharge Although all the manifestations are evident with urinary tract infections (UTIs), pain location is useful in differentiating among pyelonephritis, cystitis, and urethritis because flank pain is characteristic of pyelonephritis, but dysuria is characteristic of cystitis and urethritis. Urethral discharge is indicative of urethritis, not pyelonephritis or cystitis. Fever and chills and mental confusion are nonspecific indicators of UTIs. Urinary hesitancy and postvoid dribbling may occur with a UTI but may also occur with prostate enlargement in the male patient.

In addition to urine function, the nurse recognizes that the kidneys perform numerous other functions important to the maintenance of homeostasis. Which physiologic processes are performed by the kidneys (select all that apply)? A. Production of renin B. Activation of vitamin D C. Carbohydrate metabolism D. Erythropoietin production E. Hemolysis of old red blood cells (RBCs)

A. Production of renin B. Activation of vitamin D D. Erythropoietin production In addition to urine formation, the kidneys release renin to maintain blood pressure, activate vitamin D to maintain calcium levels, and produce erythropoietin to stimulate RBC production. Carbohydrate metabolism and hemolysis of old RBCs are not physiologic functions that are performed by the kidneys.

A 71-year-old patient with a diagnosis of benign prostatic hyperplasia (BPH) has been scheduled for a contact laser technique. What is the primary goal of this intervention? A. Resumption of normal urinary drainage B. Maintenance of normal sexual functioning C. Prevention of acute or chronic renal failure D. Prevention of fluid and electrolyte imbalances

A. Resumption of normal urinary drainage The most significant signs and symptoms of BPH relate to the disruption of normal urinary drainage and consequent urine retention, incontinence, and pain. A laser technique vaporizes prostate tissue and cauterizes blood vessels and is used as an effective alternative to a TURP to resolve these problems. Fluid imbalances, sexual functioning, and kidney disease may result from uncontrolled BPH, but the central focus remains urinary drainage.

A 33-year-old patient noticed a painless lump in his scrotum on self-examination of his testicles and a feeling of heaviness. The nurse should first teach him about what diagnostic test? A. Ultrasound B. Cremasteric reflex C. Doppler ultrasound D. Transillumination with a flashlight

A. Ultrasound When the scrotum has a painless lump, scrotal swelling, and a feeling of heaviness, testicular cancer is suspected, and an ultrasound of the testes is indicated. Blood tests will also be done. The cremasteric reflex and Doppler ultrasound are done to diagnose testicular torsion. Transillumination with a flashlight is done to diagnose a hydrocele.

The patient had surgery and a urinary catheter. Eight hours after catheter removal and drinking fluids, the patient has not been able to void. What should the nurse do first to assess for urinary retention? A. Bladder scan B.Cystometrogram C. Residual urine test D. Kidneys, ureters, bladder (KUB) x-ray

A.Bladder scan If the patient is unable to void, the bladder may be palpated for distention, percussed for dullness if it is full, or a bladder scan may be done to determine the approximate amount of urine in the bladder. A cystometrogram visualizes the bladder and evaluates vesicoureteral reflux. A KUB x-ray delineates size, shape, and positions of kidneys and possibly a full bladder. Neither of these would be useful in this situation. A residual urine test requires urination before catheterizing the patient to determine the amount of urine left in the bladder, so this assessment would not be helpful for this patient.

A 28-year-old patient with endometriosis asks why she is being treated with medroxyprogesterone (Depo-Provera), a medication that she thought was an oral contraceptive. The nurse explains that this therapy a. suppresses the menstrual cycle by mimicking pregnancy. b. will relieve symptoms such as vaginal atrophy and hot flashes. c. prevents a pregnancy that could worsen the menstrual bleeding. d. will lead to permanent suppression of abnormal endometrial tissues.

ANS: A Depo-Provera induces a pseudopregnancy, which suppresses ovulation and causes shrinkage of endometrial tissue. Menstrual bleeding does not occur during pregnancy. Vaginal atrophy and hot flashes are caused by synthetic androgens such as danazol or gonadotropin-releasing hormone agonists (GNRH) such as leuprolide. Although hormonal therapies will control endometriosis while the therapy is used, endometriosis will recur once the menstrual cycle is reestablished

A 56-year-old woman is concerned about having a moderate amount of vaginal bleeding after 4 years of menopause. The nurse will anticipate teaching the patient about a. endometrial biopsy. b. endometrial ablation. c. uterine balloon therapy. d. dilation and curettage (D&C).

ANS: A A postmenopausal woman with vaginal bleeding should be evaluated for endometrial cancer, and endometrial biopsy is the primary test for endometrial cancer. D&C will be needed only if the biopsy does not provide sufficient information to make a diagnosis. Endometrial ablation and balloon therapy are used to treat menorrhagia, which is unlikely in this patient

When admitting a patient with acute glomerulonephritis, it is most important that the nurse ask the patient about a. recent sore throat and fever. b. history of high blood pressure. c. frequency of bladder infections. d. family history of kidney stones.

ANS: A Acute glomerulonephritis frequently occurs after a streptococcal infection such as strep throat. It is not caused by hypertension, urinary tract infection (UTI), or kidney stones. DIF: Cognitive Level: Application REF: 1131-1132

Which finding for a patient who has hypothyroidism and hypertension indicates that the nurse should contact the health care provider before administering levothyroxine (Synthroid)? a. Increased thyroxine (T4) level b. Blood pressure 112/62 mm Hg c. Distant and difficult to hear heart sounds d. Elevated thyroid stimulating hormone level

ANS: A An increased thyroxine level indicates the levothyroxine dose needs to be decreased. The other data are consistent with hypothyroidism and the nurse should administer the levothyroxine.

Following rectal surgery, a patient voids about 50 mL of urine every 30 to 60 minutes. Which nursing action is most appropriate? a. Use an ultrasound scanner to check the postvoiding residual. b. Monitor the patient's intake and output over the next few hours. c. Have the patient take small amounts of fluid frequently throughout the day. d. Reassure the patient that this is normal after rectal surgery because of anesthesia.

ANS: A An ultrasound scanner can be used to check for residual urine after the patient voids. Because the patient's history and clinical manifestations are consistent with overflow incontinence, it is not appropriate to have the patient drink small amounts. Although overflow incontinence is not unusual after surgery, the nurse should intervene to correct the physiologic problem, not just reassure the patient. The patient may develop reflux into the renal pelvis as well as discomfort from a full bladder if the nurse waits to address the problem for several hours. DIF: Cognitive Level: Application REF: 1146-1147 | 1154

A 32-year-old patient is diagnosed with polycystic kidney disease. Which information is most appropriate for the nurse to include in teaching at this time? a. Importance of genetic counseling b. Complications of renal transplantation c. Methods for treating chronic and severe pain d. Differences between hemodialysis and peritoneal dialysis

ANS: A Because a 32-year-old patient may be considering having children, the nurse should include information about genetic counseling when teaching the patient. The well-managed patient will not need to choose between hemodialysis and peritoneal dialysis or know about the effects of transplantation for many years. There is no indication that the patient has chronic pain. DIF: Cognitive Level: Application REF: 1143

Which finding by the nurse when assessing a patient with Hashimoto's thyroiditis and a goiter will require the most immediate action? a. New-onset changes in the patient's voice b. Apical pulse rate at rest 112 beats/minute c. Elevation in the patient's T3 and T4 levels d. Bruit audible bilaterally over the thyroid gland

ANS: A Changes in the patient's voice indicate that the goiter is compressing the laryngeal nerve and may lead to airway compression. The other findings will also be reported but are expected with Hashimoto's thyroiditis and do not require immediate action.

Which statement by the patient indicates that the nurse's teaching about treating vaginal candidiasis has been effective? a. "I should clean carefully after each urination and bowel movement." b. "I can douche with warm water if the itching continues to bother me." c. "I will insert the antifungal cream right before I get up in the morning." d. "I will tell my husband that we cannot have intercourse for the next month."

ANS: A Cleaning of the perineal area will decrease itching caused by contact of the irritated tissues with urine and reduce the chance of further infection of irritated tissues by bacteria in the stool. Sexual intercourse should be avoided for 1 week. Douching will disrupt normal protective mechanisms in the vagina. The cream should be used at night so that it will remain in the vagina for longer periods of time

Which finding indicates to the nurse that the current therapies are effective for a patient with acute adrenal insufficiency? a. Increasing serum sodium levels b. Decreasing blood glucose levels c. Decreasing serum chloride levels d. Increasing serum potassium levels

ANS: A Clinical manifestations of Addison's disease include hyponatremia and an increase in sodium level indicates improvement. The other values indicate that treatment has not been effective.

A 58-year-old patient who has undergone a radical vulvectomy for vulvar carcinoma returns to the medical-surgical unit after the surgery. The priority nursing diagnosis for the patient at this time is a. risk for infection related to contact of the wound with urine and stool. b. self-care deficit: bathing/hygiene related to pain and difficulty moving. c. imbalanced nutrition: less than body requirements related to low-residue diet. d. risk for ineffective sexual pattern related to disfiguration caused by the surgery.

ANS: A Complex and meticulous wound care is needed to prevent infection and delayed wound healing. The other nursing diagnoses may also be appropriate for the patient but are not the highest priority immediately after surgery

A 56-year-old female patient has an adrenocortical adenoma, causing hyperaldosteronism. The nurse providing care should a. monitor the blood pressure every 4 hours. b. elevate the patient's legs to relieve edema. c. monitor blood glucose level every 4 hours. d. order the patient a potassium-restricted diet.

ANS: A Hypertension caused by sodium retention is a common complication of hyperaldosteronism. Hyperaldosteronism does not cause an elevation in blood glucose. The patient will be hypokalemic and require potassium supplementation before surgery. Edema does not usually occur with hyperaldosteronism.

When planning teaching for a patient with benign nephrosclerosis the nurse should include instructions regarding a. monitoring and recording blood pressure. b. obtaining and documenting daily weights. c. measuring daily intake and output amounts. d. preventing bleeding caused by anticoagulants.

ANS: A Hypertension is the major symptom of nephrosclerosis. Measurements of intake and output and daily weights are not necessary unless the patient develops renal insufficiency. Anticoagulants are not used to treat nephrosclerosis. DIF: Cognitive Level: Application REF: 1141-1142

A 50-year-old patient is diagnosed with uterine bleeding caused by a leiomyoma. Which information will the nurse include in the patient teaching plan? a. The symptoms may decrease after the patient undergoes menopause. b. The tumor size is likely to increase throughout the patient's lifetime. c. Aspirin or acetaminophen may be used to control mild to moderate pain. d. The patient will need frequent monitoring to detect any malignant changes.

ANS: A Leiomyomas appear to depend on ovarian hormones and will atrophy after menopause, leading to a decrease in symptoms. Aspirin use is discouraged because the antiplatelet effects may lead to heavier uterine bleeding. The size of the tumor will shrink after menopause. Leiomyomas are benign tumors that do not undergo malignant changes.

A patient in the hospital has a history of functional urinary incontinence. Which nursing action will be included in the plan of care? a. Place a bedside commode near the patient's bed. b. Demonstrate the use of the Credé maneuver to the patient. c. Use an ultrasound scanner to check postvoiding residuals. d. Teach the use of Kegel exercises to strengthen the pelvic floor.

ANS: A Modifications in the environment make it easier to avoid functional incontinence. Checking for residual urine and performing the Credé maneuver are interventions for overflow incontinence. Kegel exercises are useful for stress incontinence. DIF: Cognitive Level: Application REF: 1148

The nurse determines that additional instruction is needed for a 60-year-old patient with chronic syndrome of inappropriate antidiuretic hormone (SIADH) when the patient says which of the following? a. "I need to shop for foods low in sodium and avoid adding salt to food." b. "I should weigh myself daily and report any sudden weight loss or gain." c. "I need to limit my fluid intake to no more than 1 quart of liquids a day." d. "I will eat foods high in potassium because diuretics cause potassium loss."

ANS: A Patients with SIADH are at risk for hyponatremia, and a sodium supplement may be prescribed. The other patient statements are correct and indicate successful teaching has occurred.

A 22-year-old tells the nurse that she has not had a menstrual period for the last 2 months. Which action is most important for the nurse to take? a. Obtain a urine specimen for a pregnancy test. b. Ask about any recent stressful lifestyle changes. c. Measure the patient's current height and weight. d. Question the patient about prescribed medications.

ANS: A Pregnancy should always be considered a possible cause of amenorrhea in women of childbearing age. The other actions are also appropriate, but it is important to check for pregnancy in this patient because pregnancy will require rapid implementation of actions to promote normal fetal development such as changes in lifestyle, folic acid intake, etc.

Which nursing action will be most helpful in decreasing the risk for hospital-acquired infection (HAI) of the urinary tract in patients admitted to the hospital? a. Avoid unnecessary catheterizations. b. Encourage adequate oral fluid intake. c. Test urine with a dipstick daily for nitrites. d. Provide thorough perineal hygiene to patients.

ANS: A Since catheterization bypasses many of the protective mechanisms that prevent urinary tract infection (UTI), avoidance of catheterization is the most effective means of reducing HAI. The other actions will also be helpful, but are not as useful as decreasing urinary catheter use. DIF: Cognitive Level: Application REF: 1125-1127

A 48-year-old woman in the emergency department reports that she has been sexually assaulted. Which action by the nurse will be most important in maintaining the medicolegal chain of evidence? a. Labeling all specimens and other materials obtained from the patient. b. Assisting the patient in filling out the application for financial compensation. c. Discussing the availability of the "morning-after pill" for pregnancy prevention. d. Educating the patient about baseline sexually transmitted infection (STI) testing.

ANS: A The careful labeling of specimens and materials will assist in maintaining the chain of evidence. Assisting with paperwork, and discussing STIs and pregnancy prevention are interventions that might be appropriate after sexual assault, but they do not help maintain the legal chain of evidence

A 19-year-old visits the health clinic for a routine checkup. Which question should the nurse ask to determine whether a Pap test is needed? a. "Have you had sexual intercourse?" b. "Do you use any illegal substances?" c. "Do you have cramping with your periods?" d. "At what age did your menstrual periods start?"

ANS: A The current American Cancer Society recommendation is that a Pap test be done every 3 years, starting 3 years after the first sexual intercourse and no later than age 21. The information about menstrual periods and substance abuse will not help determine whether the patient requires a Pap test

A 54-year-old patient is on the surgical unit after a radical abdominal hysterectomy. Which finding is most important to report to the health care provider? a. Urine output of 125 mL in the first 8 hours after surgery b. Decreased bowel sounds in all four abdominal quadrants c. One-inch area of bloody drainage on the abdominal dressing d. Complaints of abdominal pain at the incision site with coughing

ANS: A The decreased urine output indicates possible low blood volume and further assessment is needed to assess for possible internal bleeding. Decreased bowel sounds, minor drainage on the dressing, and abdominal pain with coughing are expected after this surgery

A 32-year-old woman brought to the emergency department reports being sexually assaulted. The patient is confused about where she is and she has a large laceration above the right eye. Which action should the nurse take first? a. Assess the patient's neurologic status. b. Assist the patient to remove her clothing. c. Contact the sexual assault nurse examiner (SANE). d. Ask the patient to describe what occurred during the assault.

ANS: A The first priority is to treat urgent medical problems associated with the sexual assault. The patient's head injury may be associated with a head trauma such as a skull fracture or subdural hematoma. Therefore her neurologic status should be assessed first. The other nursing actions are also appropriate, but they are not as high in priority as assessment and treatment for acute physiologic injury

A patient who is diagnosed with nephrotic syndrome has 3+ ankle and leg edema and ascites. Which nursing diagnosis is a priority for the patient? a. Excess fluid volume related to low serum protein levels b. Activity intolerance related to increased weight and fatigue c. Disturbed body image related to peripheral edema and ascites d. Altered nutrition: less than required related to protein restriction

ANS: A The patient has massive edema, so the priority problem at this time is the excess fluid volume. The other nursing diagnoses also are appropriate, but the focus of nursing care should be resolution of the edema and ascites. DIF: Cognitive Level: Application REF: 1133-1135

The nurse is caring for a patient admitted with diabetes insipidus (DI). Which information is most important to report to the health care provider? a. The patient is confused and lethargic. b. The patient reports a recent head injury. c. The patient has a urine output of 400 mL/hr. d. The patient's urine specific gravity is 1.003.

ANS: A The patient's confusion and lethargy may indicate hypernatremia and should be addressed quickly. In addition, patients with DI compensate for fluid losses by drinking copious amounts of fluids, but a patient who is lethargic will be unable to drink enough fluids and will become hypovolemic. A high urine output, low urine specific gravity, and history of a recent head injury are consistent with diabetes insipidus, but they do not require immediate nursing action to avoid life-threatening complications.

An 88-year-old with benign prostatic hyperplasia (BPH) has a markedly distended bladder and is agitated and confused. Which of the following interventions prescribed by the health care provider should the nurse implement first? a. Insert a urinary retention catheter. b. Schedule an intravenous pyelogram. c. Administer lorazepam (Ativan) 0.5 mg PO. d. Draw blood for blood urea nitrogen (BUN) and creatinine testing.

ANS: A The patient's history and clinical manifestations are consistent with acute urinary retention, and the priority action is to relieve the retention by catheterization. The BUN and creatinine measurements can be obtained after the catheter is inserted. The patient's agitation may resolve once the bladder distention is corrected, and sedative drugs should be used cautiously in older patients. The IVP is an appropriate test, but does not need to be done urgently. DIF: Cognitive Level: Application REF: 1135-1136

A 28-year-old patient reports anxiety, headaches with dizziness, and abdominal bloating occurring before her menstrual periods. Which action is best for the nurse to take at this time? a. Ask the patient to keep track of her symptoms in a diary for 3 months. b. Suggest that the patient try aerobic exercise to decrease her symptoms. c. Teach the patient about appropriate lifestyle changes to reduce premenstrual syndrome (PMS) symptoms. d. Advise the patient to use nonsteroidal antiinflammatory drugs (NSAIDs) such as ibuprofen (Advil) to control symptoms.

ANS: A The patient's symptoms indicate possible PMS, but they also may be associated with other diagnoses. Having the patient keep a symptom diary for 2 or 3 months will help in confirming a diagnosis of PMS. The nurse should not implement interventions for PMS until a diagnosis is made

The nurse is assessing the sexual-reproductive functional health pattern of a 32-year-old woman. Which question is most useful in determining the patient's sexual orientation and risk factors? a. "Do you have sex with men, women, or both?" b. "Which gender do you prefer to have sex with?" c. "What types of sexual activities do you prefer?" d. "Are you heterosexual, homosexual, or bisexual?"

ANS: A This question is the most simply stated and will increase the likelihood of obtaining the relevant information about sexual orientation and possible risk factors associated with sexual activity. A patient who prefers sex with women may also have intercourse at times with men. The types of sexual activities engaged in may not indicate sexual orientation. Many patients who have sex with both men and women do not identify themselves as homosexual or bisexual.

Which topic will the nurse include in the preoperative teaching for a patient admitted for an abdominal hysterectomy? a. Purpose of ambulation and leg exercises b. Adverse effects of systemic chemotherapy c. Decrease in vaginal sensation after surgery d. Symptoms caused by the drop in estrogen level

ANS: A Venous thromboembolism (VTE) is a potential complication after the surgery, and the nurse will instruct the patient about ways to prevent it. Vaginal sensation is decreased after a vaginal hysterectomy but not after abdominal hysterectomy. Leiomyomas are benign tumors, so chemotherapy and radiation will not be prescribed. Because the patient will still have her ovaries, the estrogen level will not decrease

A patient who was admitted with myxedema coma and diagnosed with hypothyroidism is improving and expected to be discharged in 2 days. Which teaching strategy will be best for the nurse to use? a. Provide written reminders of self-care information. b. Offer multiple options for management of therapies. c. Ensure privacy for teaching by asking visitors to leave. d. Delay teaching until patient discharge date is confirmed.

ANS: A Written instructions will be helpful to the patient because initially the hypothyroid patient may be unable to remember to take medications and other aspects of self-care. Because the treatment regimen is somewhat complex, teaching should be initiated well before discharge. Family members or friends should be included in teaching because the hypothyroid patient is likely to forget some aspects of the treatment plan. A simpler regimen will be easier to understand until the patient is euthyroid.

The client scheduled for intravenous urography informs the nurse of the following allergies. Which one should the nurse report to the physician immediately? A. Seafood B. Penicillin C. Bee stings D. Red food dye

ANS: A Clients with seafood allergies often have severe allergic reactions to the standard dyes used during intravenous urography.

The client scheduled to have an intravenous urogram is a diabetic and taking the antidiabetic agent metformin. What should the nurse tell this client? A. "Call your diabetes doctor and tell him or her that you are having an intravenous urogram performed using dye." B. "Do not take your metformin the morning of the test because you are not going to be eating anything and could become hypoglycemic." C. "You must start on an antibiotic before this test because your risk of infection is greater as a result of your diabetes." D. "You must take your metformin immediately before the test is performed because the IV fluid and the dye contain a significant amount of sugar."

ANS: A Metformin can cause a lactic acidosis and renal impairment as an interaction with the dye. This drug must be discontinued for 48 hours before the procedure and not started again after the procedure until urine output is well established.

What would be the response if a person's nephrons were not able to filter normally due to scarring of the proximal convoluted tubule leading to inhibition of reabsorption? A. Increased urine output, fluid volume deficit B. Decreased urine output, fluid volume deficit C. Increased urine output, fluid volume overload D. Decreased urine output, fluid volume overload

ANS: A The nephrons filter about 120 mL/min. Most of this filtrate is reabsorbed in the proximal convoluted tubule. If the tubule were not able to reabsorb the fluid that has been filtered, urine output would greatly increase, leading to rapid and severe dehydration.

Which nonhormonal therapies will the nurse suggest for a healthy perimenopausal woman who prefers not to use hormone therapy (HT) (select all that apply)? a. Reduce coffee intake. b. Exercise several times a week. c. Take black cohosh supplements. d. Have a glass of wine in the evening. e. Increase intake of dietary soy products.

ANS: A, B, C, E Reduction in caffeine intake, use of black cohosh, increasing dietary soy intake, and exercising three to four times weekly are recommended to reduce symptoms associated with menopause. Alcohol intake in the evening may increase the sleep problems associated with menopause

After scheduling a patient with a possible ovarian cyst for ultrasound, the nurse will teach the patient that she should a. expect to receive IV contrast during the procedure. b. drink several glasses of fluids before the procedure. c. experience mild abdominal cramps after the procedure. d. discontinue taking aspirin for 7 days before the procedure.

ANS: B A full bladder is needed for many ultrasound procedures, so the nurse will have the patient drink fluids before arriving for the ultrasound. The other instructions are not accurate for this procedure.

A 45-year-old male patient with suspected acromegaly is seen at the clinic. To assist in making the diagnosis, which question should the nurse ask? a. "Have you had a recent head injury?" b. "Do you have to wear larger shoes now?" c. "Is there a family history of acromegaly?" d. "Are you experiencing tremors or anxiety?"

ANS: B Acromegaly causes an enlargement of the hands and feet. Head injury and family history are not risk factors for acromegaly. Tremors and anxiety are not clinical manifestations of acromegaly.

The nurse is planning postoperative care for a patient who is being admitted to the surgical unit form the recovery room after transsphenoidal resection of a pituitary tumor. Which nursing action should be included? a. Palpate extremities for edema. b. Measure urine volume every hour. c. Check hematocrit every 2 hours for 8 hours. d. Monitor continuous pulse oximetry for 24 hours.

ANS: B After pituitary surgery, the patient is at risk for diabetes insipidus caused by cerebral edema. Monitoring of urine output and urine specific gravity is essential. Hemorrhage is not a common problem. There is no need to check the hematocrit hourly. The patient is at risk for dehydration, not volume overload. The patient is not at high risk for problems with oxygenation, and continuous pulse oximetry is not needed.

A patient with renal calculi is hospitalized with gross hematuria and severe colicky left flank pain. Which nursing action will be of highest priority at this time? a. Encourage oral fluid intake. b. Administer prescribed analgesics. c. Monitor temperature every 4 hours. d. Give antiemetics as needed for nausea.

ANS: B Although all of the nursing actions may be used for patients with renal lithiasis, the patient's presentation indicates that management of pain is the highest priority action. If the patient has urinary obstruction, increasing oral fluids may increase the symptoms. There is no evidence of infection or nausea. DIF: Cognitive Level: Application REF: 1137-1138 | 1139-1141 | 1140

A 31-year-old patient who has been diagnosed with human papillomavirus (HPV) infection gives a health history that includes smoking tobacco, taking oral contraceptives, and having been treated twice for vaginal candidiasis. Which topic will the nurse include in patient teaching? a. Use of water-soluble lubricants b. Risk factors for cervical cancer c. Antifungal cream administration d. Possible difficulties with conception

ANS: B Because HPV infection and smoking are both associated with increased cervical cancer risk, the nurse should emphasize the importance of avoiding smoking. An HPV infection does not decrease vaginal lubrication, decrease ability to conceive, or require the use of antifungal creams.

When caring for a 58-year-old patient with persistent menorrhagia, the nurse will plan to monitor the a. estrogen level. b. complete blood count (CBC). c. gonadotropin-releasing hormone (GNRH) level. d. serial human chorionic gonadotropin (hCG) results.

ANS: B Because anemia is a likely complication of menorrhagia, the nurse will need to check the CBC. Estrogen and GNRH levels are checked for patients with other problems, such as infertility. Serial hCG levels are monitored in patients who may be pregnant, which is not likely for this patient

A 24-year-old female says she wants to begin using oral contraceptives. Which information from the nursing assessment is most important to report to the health care provider? a. The patient quit smoking 5 months previously. b. The patient's blood pressure is 154/86 mm Hg. c. The patient has not been vaccinated for rubella. d. The patient has chronic iron-deficiency anemia.

ANS: B Because hypertension increases the risk for morbidity and mortality in women taking oral contraceptives, the patient's blood pressure should be controlled before oral contraceptives are prescribed. The other information also will be reported but will not affect the choice of contraceptive.

A 25-year-old woman has an induced abortion with suction curettage at an ambulatory surgical center. Which instructions will the nurse include when discharging the patient? a. "Heavy vaginal bleeding is expected for about 2 weeks." b. "You should abstain from sexual intercourse for 2 weeks." c. "Contraceptives should be avoided until your reexamination." d. "Irregular menstrual periods are expected for the next few months."

ANS: B Because infection is a possible complication of this procedure, the patient is advised to avoid intercourse until the reexamination in 2 weeks. Patients may be started on contraceptives on the day of the procedure. The patient should call the doctor if heavy vaginal bleeding occurs. No change in the regularity of the menstrual periods is expected

The nurse will plan to teach a 34-year-old patient diagnosed with stage 0 cervical cancer about a. radiation. b. conization. c. chemotherapy. d. radical hysterectomy.

ANS: B Because the carcinoma is in situ, conization can be used for treatment. Radical hysterectomy, chemotherapy, or radiation will not be needed

A 32-year-old who was admitted to the emergency department with severe abdominal pain is diagnosed with an ectopic pregnancy. The patient begins to cry and asks the nurse to leave her alone to grieve. Which action should the nurse take next? a. Stay with the patient and encourage her to discuss her feelings. b. Explain the reason for taking vital signs every 15 to 30 minutes. c. Close the door to the patient's room and minimize disturbances. d. Provide teaching about options for termination of the pregnancy.

ANS: B Because the patient is at risk for rupture of the fallopian tube and hemorrhage, frequent monitoring of vital signs is needed. The patient has asked to be left alone, so staying with her and encouraging her to discuss her feelings are inappropriate actions. Minimizing contact with her and closing the door of the room is unsafe because of the risk for hemorrhage. Because the patient has requested time to grieve, it would be inappropriate to provide teaching about options for pregnancy termination

The nurse is assessing a 41-year-old African American male patient diagnosed with a pituitary tumor causing panhypopituitarism. Assessment findings consistent with panhypopituitarism include a. high blood pressure. b. decreased facial hair. c. elevated blood glucose. d. tachycardia and cardiac palpitations.

ANS: B Changes in male secondary sex characteristics such as decreased facial hair, testicular atrophy, diminished spermatogenesis, loss of libido, impotence, and decreased muscle mass are associated with decreases in follicle stimulating hormone (FSH) and luteinizing hormone (LH). Fasting hypoglycemia and hypotension occur in panhypopituitarism as a result of decreases in adrenocorticotropic hormone (ACTH) and cortisol. Bradycardia is likely due to the decrease in thyroid stimulating hormone (TSH) and thyroid hormones associated with panhypopituitarism.

Which nursing action should the nurse who is caring for a patient who has had an ileal conduit for several years delegate to nursing assistive personnel (NAP)? a. Assess for symptoms of urinary tract infection (UTI). b. Change the ostomy appliance. c. Choose the appropriate ostomy bag. d. Monitor the appearance of the stoma.

ANS: B Changing the ostomy appliance for a stable patient could be done by NAP. Assessments of the site, choosing the appropriate ostomy bag, and assessing for (UTI) symptoms require more education and scope of practice and should be done by the RN. DIF: Cognitive Level: Application REF: 1157 | 1159-1160 | 1158

The nurse determines that demeclocycline (Declomycin) is effective for a patient with syndrome of inappropriate antidiuretic hormone (SIADH) based on finding that the patient's a. weight has increased. b. urinary output is increased. c. peripheral edema is decreased. d. urine specific gravity is increased.

ANS: B Demeclocycline blocks the action of antidiuretic hormone (ADH) on the renal tubules and increases urine output. An increase in weight or an increase in urine specific gravity indicates that the SIADH is not corrected. Peripheral edema does not occur with SIADH. A sudden weight gain without edema is a common clinical manifestation of this disorder.

The cardiac telemetry unit charge nurse receives status reports from other nursing units about four patients who need cardiac monitoring. Which patient should be transferred to the cardiac unit first? a. Patient with Hashimoto's thyroiditis and a heart rate of 102 b. Patient with tetany who has a new order for IV calcium chloride c. Patient with Cushing syndrome and a blood glucose of 140 mg/dL d. Patient with Addison's disease who takes hydrocortisone twice daily

ANS: B Emergency treatment of tetany requires IV administration of calcium; ECG monitoring will be required because cardiac arrest may occur if high calcium levels result from too-rapid administration. The information about the other patients indicates that they are more stable than the patient with tetany.

A 22-year-old patient reports her concern about not having a menstrual period for the past 7 months. Which statement by the patient indicates a possible related factor to the amenorrhea? a. "I drink at least 3 glasses of nonfat milk every day." b. "I run 7 to 8 miles every day to keep my weight down." c. "I was treated for a sexually transmitted infection 2 years ago." d. "I am not sexually active but currently I have an IUD."

ANS: B Excessive exercise can cause amenorrhea. The other statements by the patient do not suggest any urgent teaching needs.

A patient with nephrotic syndrome develops flank pain. The nurse will anticipate teaching the patient about treatment with a. antibiotics. b. anticoagulants. c. corticosteroids. d. antihypertensives.

ANS: B Flank pain in a patient with nephrosis suggests a renal vein thrombosis, and anticoagulation is needed. Antibiotics are used to treat a patient with flank pain caused by pyelonephritis. Antihypertensives are used if the patient has high blood pressure. Corticosteroids may be used to treat nephrotic syndrome but will not resolve a thrombosis. DIF: Cognitive Level: Application REF: 1133-1134

A couple is scheduled to have a Huhner test for infertility. In preparation for the test, the nurse will instruct the couple about a. being sedated during the procedure. b. determining the estimated time of ovulation. c. experiencing shoulder pain after the procedure. d. refraining from intercourse before the appointment.

ANS: B For the Huhner test, the couple should have intercourse at the estimated time of ovulation and then arrive for the test 2 to 8 hours after intercourse. The other instructions would be used for other types of fertility testing.

After teaching a patient with interstitial cystitis about management of the condition, the nurse determines that further instruction is needed when the patient says, a. "I will have to stop having coffee and orange juice for breakfast." b. "I should start taking a high potency multiple vitamin every morning." c. "I will buy some calcium glycerophosphate (Prelief) at the pharmacy." d. "I should call the doctor about increased bladder pain or odorous urine."

ANS: B High-potency multiple vitamins may irritate the bladder and increase symptoms. The other patient statements indicate good understanding of the teaching.

Which nursing assessment of a 69-year-old patient is most important to make during initiation of thyroid replacement with levothyroxine (Synthroid)? a. Fluid balance b. Apical pulse rate c. Nutritional intake d. Orientation and alertness

ANS: B In older patients, initiation of levothyroxine therapy can increase myocardial oxygen demand and cause angina or dysrhythmias. The medication also is expected to improve mental status and fluid balance and will increase metabolic rate and nutritional needs, but these changes will not result in potentially life-threatening complications.

After receiving change-of-shift report about the following four patients, which patient should the nurse assess first? a. A 31-year-old female with Cushing syndrome and a blood glucose level of 244 mg/dL b. A 70-year-old female taking levothyroxine (Synthroid) who has an irregular pulse of 134 c. A 53-year-old male who has Addison's disease and is due for a scheduled dose of hydrocortisone (Solu-Cortef). d. A 22-year-old male admitted with syndrome of inappropriate antidiuretic hormone (SIADH) who has a serum sodium level of 130 mEq/L

ANS: B Initiation of thyroid replacement in older adults may cause angina and cardiac dysrhythmias. The patient's high pulse rate needs rapid investigation by the nurse to assess for and intervene with any cardiac problems. The other patients also require nursing assessment and/or actions but are not at risk for life-threatening complications.

Which question will the nurse in the endocrine clinic ask to help determine a patient's risk factors for goiter? a. "How much milk do you drink?" b. "What medications are you taking?" c. "Are your immunizations up to date?" d. "Have you had any recent neck injuries?"

ANS: B Medications that contain thyroid-inhibiting substances can cause goiter. Milk intake, neck injury, and immunization history are not risk factors for goiter.

A patient who had radical neck surgery to remove a malignant tumor developed hypoparathyroidism. The nurse should plan to teach the patient about a. bisphosphonates to reduce bone demineralization. b. calcium supplements to normalize serum calcium levels. c. increasing fluid intake to decrease risk for nephrolithiasis. d. including whole grains in the diet to prevent constipation.

ANS: B Oral calcium supplements are used to maintain the serum calcium in normal range and prevent the complications of hypocalcemia. Whole grain foods decrease calcium absorption and will not be recommended. Bisphosphonates will lower serum calcium levels further by preventing calcium from being reabsorbed from bone. Kidney stones are not a complication of hypoparathyroidism and low calcium levels.

A 29-year-old patient who is trying to become pregnant asks the nurse how to determine when she is most likely to conceive. The nurse explains that a. ovulation is unpredictable unless there are regular menstrual periods. b. ovulation prediction kits provide accurate information about ovulation. c. she will need to bring a specimen of cervical mucus to the clinic for testing. d. she should take her body temperature daily and have intercourse when it drops.

ANS: B Ovulation prediction kits indicate when luteinizing hormone (LH) levels first rise. Ovulation occurs about 28 to 36 hours after the first rise of LH. This information can be used to determine the best time for intercourse. Body temperature rises at ovulation. Postcoital cervical smears are used in infertility testing, but they do not predict the best time for conceiving and are not obtained by the patient. Determination of the time of ovulation can be predicted by basal body temperature charts or ovulation prediction kits and is not dependent on regular menstrual periods

After the home health nurse teaches a patient with a neurogenic bladder how to use intermittent catheterization for bladder emptying, which patient statement indicates that the teaching has been effective? a. "I will use a sterile catheter and gloves for each time I self-catheterize." b. "I will clean the catheter carefully before and after each catheterization." c. "I will need to buy seven new catheters weekly and use a new one every day." d. "I will need to take prophylactic antibiotics to prevent any urinary tract infections."

ANS: B Patients who are at home can use a clean technique for intermittent self-catheterization and change the catheter every 7 days. There is no need to use a new catheter every day, to use sterile catheters, or to take prophylactic antibiotics. DIF: Cognitive Level: Application REF: 1154

An 18-year-old female patient who has been admitted to the emergency department after a motor vehicle crash is scheduled for chest and abdominal x-rays. Which information is most important to report to the health care provider before the x-rays are obtained? a. Severity of abdominal pain b. Positive result of hCG test c. Blood pressure 172/88 mm Hg d. Temperature 102.1° F (38.9° C)

ANS: B Positive hCG testing indicates that the patient is pregnant and that unnecessary abdominal x-rays should be avoided. The other information is also important to report, but it will not affect whether the x-rays should be done.

A patient returns to the clinic with recurrent dysuria after being treated with trimethoprim and sulfamethoxazole (Bactrim) for 3 days. Which action will the nurse plan to take? a. Remind the patient about the need to drink 1000 mL of fluids daily. b. Obtain a midstream urine specimen for culture and sensitivity testing. c. Teach the patient to take the prescribed Bactrim for at least 3 more days. d. Suggest that the patient use acetaminophen (Tylenol) to treat the symptoms.

ANS: B Since uncomplicated urinary tract infections (UTIs) are usually successfully treated with 3 days of antibiotic therapy, this patient will need a urine culture and sensitivity to determine appropriate antibiotic therapy. Tylenol would not be as effective as other over-the-counter (OTC) medications such as phenazopyridine (Pyridium) in treating dysuria. The fluid intake should be increased to at least 1800 mL/day. Since the UTI has persisted after treatment with Bactrim, the patient is likely to need a different antibiotic. DIF: Cognitive Level: Application REF: 1123-1125

Following an open loop resection and fulguration of the bladder, a patient is unable to void. Which nursing action should be implemented first? a. Insert a straight catheter and drain the bladder. b. Assist the patient to take a 15-minute sitz bath. c. Encourage the patient to drink several glasses of water. d. Teach the patient how to do isometric perineal exercises.

ANS: B Sitz baths will relax the perineal muscles and promote voiding. Although the patient should be encouraged to drink fluids and Kegel exercises are helpful in the prevention of incontinence, these activities would not be helpful for a patient experiencing retention. Catheter insertion increases the risk for urinary tract infection (UTI) and should be avoided when possible DIF: Cognitive Level: Application REF: 1146

Which information will the nurse include when teaching a 50-year-old male patient about somatropin (Genotropin)? a. The medication will be needed for 3 to 6 months. b. Inject the medication subcutaneously every day. c. Blood glucose levels may decrease when taking the medication. d. Stop taking the medication if swelling of the hands or feet occurs.

ANS: B Somatropin is injected subcutaneously on a daily basis, preferably in the evening. The patient will need to continue on somatropin for life. If swelling or other common adverse effects occur, the health care provider should be notified. Growth hormone will increase blood glucose levels.

Which intervention will the nurse include in the plan of care for a 52-year-old male patient with syndrome of inappropriate antidiuretic hormone (SIADH)? a. Monitor for peripheral edema. b. Offer patient hard candies to suck on. c. Encourage fluids to 2 to 3 liters per day. d. Keep head of bed elevated to 30 degrees.

ANS: B Sucking on hard candies decreases thirst for a patient on fluid restriction. Patients with SIADH are on fluid restrictions of 800 to 1000 mL/day. Peripheral edema is not seen with SIADH. The head of the bed is elevated no more than 10 degrees to increase left atrial filling pressure and decrease antidiuretic hormone (ADH) release.

A patient who has bladder cancer had a cystectomy with creation of an Indiana pouch. Which topic will be included in patient teaching? a. Application of ostomy appliances b. Catheterization technique and schedule c. Analgesic use before emptying the pouch d. Use of barrier products for skin protection

ANS: B The Indiana pouch enables the patient to self-catheterize every 4 to 6 hours. There is no need for an ostomy device or barrier products. Catheterization of the pouch is not painful. DIF: Cognitive Level: Application REF: 1155-1156

The nurse will plan to monitor a patient diagnosed with a pheochromocytoma for a. flushing. b. headache. c. bradycardia. d. hypoglycemia.

ANS: B The classic clinical manifestations of pheochromocytoma are hypertension, tachycardia, severe headache, diaphoresis, and abdominal or chest pain. Elevated blood glucose may also occur because of sympathetic nervous system stimulation. Bradycardia and flushing would not be expected.

A healthy 28-year-old who has been vaccinated against human papillomavirus (HPV) has a normal Pap test. Which information will the nurse include in patient teaching when calling the patient with the results of the Pap test? a. You can wait until age 30 before having another Pap test. b. Pap testing is recommended every 3 years for women your age. c. No further Pap testing is needed until you decide to become pregnant. d. Yearly Pap testing is suggested for women with multiple sexual partners.

ANS: B The current national guidelines suggest Pap testing every 3 years for patients between ages 21 to 65. Although HPV immunization does protect against cervical cancer, the recommendations are unchanged for individuals who have received the HPV vaccination

A 37-year-old patient has just arrived in the postanesthesia recovery unit (PACU) after a thyroidectomy. Which information is most important to communicate to the surgeon? a. The patient reports 7/10 incisional pain. b. The patient has increasing neck swelling. c. The patient is sleepy and difficult to arouse. d. The patient's cardiac rate is 112 beats/minute.

ANS: B The neck swelling may lead to respiratory difficulty, and rapid intervention is needed to prevent airway obstruction. The incisional pain should be treated but is not unusual after surgery. A heart rate of 112 is not unusual in a patient who has been hyperthyroid and has just arrived in the PACU from surgery. Sleepiness in the immediate postoperative period is expected.

A 38-year-old male patient is admitted to the hospital in Addisonian crisis. Which patient statement supports a nursing diagnosis of ineffective self-health management related to lack of knowledge about management of Addison's disease? a. "I frequently eat at restaurants, and my food has a lot of added salt." b. "I had the stomach flu earlier this week, so I couldn't take the hydrocortisone." c. "I always double my dose of hydrocortisone on the days that I go for a long run." d. "I take twice as much hydrocortisone in the morning dose as I do in the afternoon."

ANS: B The need for hydrocortisone replacement is increased with stressors such as illness, and the patient needs to be taught to call the health care provider because medication and IV fluids and electrolytes may need to be given. The other patient statements indicate appropriate management of the Addison's disease.

A 24-year-old patient with pelvic inflammatory disease (PID) is being treated with oral antibiotics as an outpatient. Which instruction will be included in patient teaching? a. Abdominal pain may persist for several weeks. b. Return for a follow-up appointment in 2 to 3 days. c. Instruct a male partner to use a condom during sexual intercourse for the next week. d. Nonsteroidal antiinflammatory drug (NSAID) use may prevent pelvic organ scarring

ANS: B The patient is instructed to return for follow-up in 48 to 72 hours. The patient should abstain from intercourse for 3 weeks. Abdominal pain should subside with effective antibiotic therapy. Corticosteroids may help prevent inflammation and scarring, but NSAIDs will not decrease scarring

Which nursing action will be included in the plan of care for a 55-year-old patient with Graves' disease who has exophthalmos? a. Place cold packs on the eyes to relieve pain and swelling. b. Elevate the head of the patient's bed to reduce periorbital fluid. c. Apply alternating eye patches to protect the corneas from irritation. d. Teach the patient to blink every few seconds to lubricate the corneas.

ANS: B The patient should sit upright as much as possible to promote fluid drainage from the periorbital area. With exophthalmos, the patient is unable to close the eyes completely to blink. Lubrication of the eyes, rather than eye patches, will protect the eyes from developing corneal scarring. The swelling of the eye is not caused by excessive blood flow to the eye, so cold packs will not be helpful.

A 63-year-old woman undergoes an anterior and posterior (A&P) colporrhaphy for repair of a cystocele and rectocele. Which nursing action will be included in the postoperative care plan? a. Encourage a high-fiber diet. b. Perform indwelling catheter care. c. Repack the vagina with gauze daily. d. Teach the patient to insert a pessary.

ANS: B The patient will have a retention catheter for several days after surgery to keep the bladder empty and decrease strain on the suture. A pessary will not be needed after the surgery. Vaginal wound packing is not usually used after an A&P repair. A low-residue diet will be ordered after posterior colporrhaphy

A 63-year-old patient with primary hyperparathyroidism has a serum phosphorus level of 1.7 mg/dL (0.55 mmol/L) and calcium of 14 mg/dL (3.5 mmol/L). Which nursing action should be included in the plan of care? a. Restrict the patient to bed rest. b. Encourage 4000 mL of fluids daily. c. Institute routine seizure precautions. d. Assess for positive Chvostek's sign.

ANS: B The patient with hypercalcemia is at risk for kidney stones, which may be prevented by a high fluid intake. Seizure precautions and monitoring for Chvostek's or Trousseau's sign are appropriate for hypocalcemic patients. The patient should engage in weight-bearing exercise to decrease calcium loss from bone.

A patient who had a subtotal thyroidectomy earlier today develops laryngeal stridor and a cramp in the right hand upon returning to the surgical nursing unit. Which collaborative action will the nurse anticipate next? a. Suction the patient's airway. b. Administer IV calcium gluconate. c. Plan for emergency tracheostomy. d. Prepare for endotracheal intubation.

ANS: B The patient's clinical manifestations of stridor and cramping are consistent with tetany caused by hypocalcemia resulting from damage to the parathyroid glands during surgery. Endotracheal intubation or tracheostomy may be needed if the calcium does not resolve the stridor. Suctioning will not correct the stridor.

Two days after surgery for an ileal conduit, the patient will not look at the stoma or participate in care. The patient insists that no one but the ostomy nurse specialist care for the stoma. The nurse identifies a nursing diagnosis of a. anxiety related to effects of procedure on lifestyle. b. disturbed body image related to change in body function. c. readiness for enhanced coping related to need for information. d. self-care deficit, toileting, related to denial of altered body function.

ANS: B The patient's unwillingness to look at the stoma or participate in care indicates that disturbed body image is the best diagnosis. No data suggest that the impact on lifestyle is a concern for the patient. The patient does not appear to be ready for enhanced coping. The patient's insistence that only the ostomy nurse care for the stoma indicates that denial is not present. DIF: Cognitive Level: Application REF: 1157 | 1159-1160 | 1158-1159

A 34-year-old woman who is discussing contraceptive options with the nurse says, "I want to have children, but not for a few years." Which response by the nurse is appropriate? a. "If you do not become pregnant within the next few years, you never will." b. "You may have more difficulty becoming pregnant after about age 35." c. "You have many years of fertility left, so there is no rush to have children." d. "You should plan to stop taking oral contraceptives several years before you want to become pregnant."

ANS: B The probability of successfully becoming pregnant decreases after age 35, although some patients may have no difficulty in becoming pregnant. Oral contraceptives do not need to be withdrawn for several years for a woman to become pregnant. Although the patient may be fertile for many years, it would be inaccurate to indicate that there is no concern about fertility as she becomes older. Although the risk for infertility increases after age 35, not all patients have difficulty in conceiving

Which assessment finding in a woman who recently started taking hormone therapy (HT) is most important for the nurse to report to the health care provider? a. Breast tenderness b. Left calf swelling c. Weight gain of 3 lb d. Intermittent spotting

ANS: B Unilateral calf swelling may indicate deep vein thrombosis caused by the changes in coagulation associated with HT and would indicate that the HT should be discontinued. Breast tenderness, weight gain, and intermittent spotting are common side effects of HT and do not indicate a need for a change in therapy

A 32-year-old woman is scheduled for an induced abortion using instillation of hypertonic saline solution. Which information will the nurse plan to discuss with the patient before the procedure? a. The patient will require a general anesthetic. b. The expulsion of the fetus may take 1 to 2 days. c. There is a possibility that the patient may deliver a live fetus. d. The procedure may be unsuccessful in terminating the pregnancy.

ANS: B Uterine contractions take 12 to 36 hours to begin after the hypertonic saline is instilled. Because the saline is feticidal, the nurse does not need to discuss any possibility of a live delivery or that the pregnancy termination will not be successful. General anesthesia is not needed for this procedure

A 56-year-old patient who is disoriented and reports a headache and muscle cramps is hospitalized with possible syndrome of inappropriate antidiuretic hormone (SIADH). The nurse would expect the initial laboratory results to include a(n) a. elevated hematocrit. b. decreased serum sodium. c. low urine specific gravity. d. increased serum chloride.

ANS: B When water is retained, the serum sodium level will drop below normal, causing the clinical manifestations reported by the patient. The hematocrit will decrease because of the dilution caused by water retention. Urine will be more concentrated with a higher specific gravity. The serum chloride level will usually decrease along with the sodium level.

The client is taking a medication for an endocrine problem that inhibits aldosterone secretion and release. To what complications of this therapy should the nurse be alert? A. Dehydration, hypokalemia B. Dehydration, hyperkalemia C. Overhydration, hyponatremia D. Overhydration, hypernatremia

ANS: B Aldosterone is a mineralocorticoid that increases the reabsorption of water and sodium in the kidney at the same time that it promotes excretion of potassium. Any drug or condition that disrupts aldosterone secretion or release increases the client's risk for excessive water loss and potassium reabsorption.

Which condition would trigger the release of antidiuretic hormone (ADH)? A. Plasma osmolarity decreased secondary to overhydration. B. Plasma osmolarity increased secondary to dehydration. C. Plasma volume decreased secondary to hemorrhage. D. Plasma volume increased with edema formation.

ANS: B Antidiuretic hormone is triggered by a rising ECF osmolarity, especially hypernatremia.

Which of the following conditions are associated with oversecretion of rennin? A. Alzheimer's disease B. Hypertension C. Diabetes mellitus D. Diabetes insipidus

ANS: B Renin is secreted when special cells in the DCT, called the macula densa, sense changes in blood volume and pressure. When the macula densa cells sense that blood volume is low, blood pressure is low, or blood sodium levels are low, renin is secreted. Renin then converts angiotensinogen into angiotensin I. This leads to a series of reactions that cause the secretion of the hormone aldosterone. This hormone increases kidney reabsorption of sodium and water, increasing blood pressure, blood volume, and blood sodium levels. Inappropriate or excessive renin secretion is a major cause of persistent hypertension.

The client's urine specific gravity is 1.018. What is the nurse's best action? A. Ask the client for a 24-hour recall of liquid intake. B. Document the finding as the only action. C. Obtain a specimen for culture. D. Notify the physician.

ANS: B This specific gravity is within the normal range for urine.

Which of the following muscle actions results in voluntary urination? A. Detrusor contraction, external sphincter contraction B. Detrusor contraction, external sphincter relaxation C. Detrusor relaxation, external sphincter contraction D. Detrusor relaxation, external sphincter relaxation

ANS: B Voiding becomes a voluntary act as a result of learned responses controlled by the cerebral cortex that cause contraction of the bladder detrusor muscle and simultaneous relaxation of the external urethral sphincter muscle.

Which nursing actions can the nurse working in a women's health clinic delegate to unlicensed assistive personnel (UAP) (select all that apply)? a. Call a patient with the results of an endometrial biopsy. b. Assist the health care provider with performing a Pap test. c. Draw blood for CA-125 levels for a patient with ovarian cancer. d. Screen a patient for use of medications that may cause amenorrhea. e. Teach the parent of a 10-year-old about the human papilloma virus (HPV) vaccine (Gardasil).

ANS: B, C Assisting with a Pap test and drawing blood (if trained) are skills that require minimal critical thinking and judgment and can be safely delegated to UAP. Patient teaching, calling a patient who may have questions about results of diagnostic testing, and risk-factor screening all require more education and critical thinking and should be done by the registered nurse (RN).

A 29-year-old woman with systemic lupus erythematosus has been prescribed 2 weeks of high-dose prednisone therapy. Which information about the prednisone is most important for the nurse to include? a. "Weigh yourself daily to monitor for weight gain caused by increased appetite." b. "A weight-bearing exercise program will help minimize the risk for osteoporosis." c. "The prednisone dose should be decreased gradually rather than stopped suddenly." d. "Call the health care provider if you experience mood alterations with the prednisone."

ANS: C Acute adrenal insufficiency may occur if exogenous corticosteroids are suddenly stopped. Mood alterations and weight gain are possible adverse effects of corticosteroid use, but these are not life-threatening effects. Osteoporosis occurs when patients take corticosteroids for longer periods.

The nurse is caring for a patient following an adrenalectomy. The highest priority in the immediate postoperative period is to a. protect the patient's skin. b. monitor for signs of infection. c. balance fluids and electrolytes. d. prevent emotional disturbances.

ANS: C After adrenalectomy, the patient is at risk for circulatory instability caused by fluctuating hormone levels, and the focus of care is to assess and maintain fluid and electrolyte status through the use of IV fluids and corticosteroids. The other goals are also important for the patient but are not as immediately life threatening as the circulatory collapse that can occur with fluid and electrolyte disturbances.

A patient has just arrived on the unit after a thyroidectomy. Which action should the nurse take first? a. Observe the dressing for bleeding. b. Check the blood pressure and pulse. c. Assess the patient's respiratory effort. d. Support the patient's head with pillows.

ANS: C Airway obstruction is a possible complication after thyroidectomy because of swelling or bleeding at the site or tetany. The priority nursing action is to assess the airway. The other actions are also part of the standard nursing care postthyroidectomy but are not as high of a priority.

After a 22-year-old female patient with a pituitary adenoma has had a hypophysectomy, the nurse will teach about the need for a. sodium restriction to prevent fluid retention. b. insulin to maintain normal blood glucose levels. c. oral corticosteroids to replace endogenous cortisol. d. chemotherapy to prevent malignant tumor recurrence.

ANS: C Antidiuretic hormone (ADH), cortisol, and thyroid hormone replacement will be needed for life after hypophysectomy. Without the effects of adrenocorticotropic hormone (ACTH) and cortisol, the blood glucose and serum sodium will be low unless cortisol is replaced. An adenoma is a benign tumor, and chemotherapy will not be needed.

A 19-year-old patient calls the school clinic and tells the nurse, "My menstrual period is very heavy this time. I have to change my tampon every 4 hours." Which action should the nurse take next? a. Tell the patient that her flow is not unusually heavy. b. Schedule the patient for an appointment later that day. c. Ask the patient how heavy her usual menstrual flow is. d. Have the patient call again if the heavy flow continues.

ANS: C Because a heavy menstrual flow is usually indicated by saturating a pad or tampon in 1 to 2 hours, the nurse should first assess how heavy the patient's usual flow is. There is no need to schedule the patient for an appointment that day. The patient may need to call again, but this is not the first action that the nurse should take. Telling the patient that she does not have a heavy flow implies that the patient's concern is not important.

The nurse is providing teaching by telephone to a patient who is scheduled for a pelvic examination and Pap test next week. The nurse instructs the patient that she should a. shower, but not take a tub bath, before the examination. b. not have sexual intercourse the day before the Pap test. c. avoid douching for at least 24 hours before the examination. d. schedule to have the Pap test just after her menstrual period.

ANS: C Because the results of a Pap test may be affected by douching, the patient should not douche before the examination. The exam may be scheduled without regard to the menstrual period. The patient may shower or bathe before the examination. Sexual intercourse does not affect the results of the examination or Pap test.

A patient undergoes a nephrectomy after having massive trauma to the kidney. Which assessment finding obtained postoperatively is most important to communicate to the surgeon? a. Blood pressure is 102/58. b. Incisional pain level is 8/10. c. Urine output is 20 mL/hr for 2 hours. d. Crackles are heard at both lung bases.

ANS: C Because the urine output should be at least 0.5 mL/kg/hr, a 40 mL output for 2 hours indicates that the patient may have decreased renal perfusion because of bleeding, inadequate fluid intake, or obstruction at the suture site. The blood pressure requires ongoing monitoring but does not indicate inadequate perfusion at this time. The patient should cough and deep breathe, but the crackles do not indicate a need for an immediate change in therapy. The incisional pain should be addressed, but this is not as potentially life threatening as decreased renal perfusion. In addition, the nurse can medicate the patient for pain. DIF: Cognitive Level: Application REF: 1154-1155

Which information will the nurse include when teaching a patient who has developed a small vesicovaginal fistula 2 weeks into the postpartum period? a. Take stool softeners to prevent fecal contamination of the vagina. b. Limit oral fluid intake to minimize the quantity of urinary drainage. c. Change the perineal pad frequently to prevent perineal skin breakdown. d. Call the health care provider immediately if urine drains from the vagina.

ANS: C Because urine will leak from the bladder, the patient should plan to use perineal pads and change them frequently. A high fluid intake is recommended to decrease the risk for urinary tract infections. Drainage of urine from the vagina is expected with vesicovaginal fistulas. Fecal contamination is not a concern with vesicovaginal fistulas.

A 25-year-old woman who is scheduled for a routine gynecologic examination tells the nurse that she has had intercourse during the last year with several men. The nurse will plan to teach about the reason for a. contraceptive use. b. antibiotic therapy. c. Chlamydia testing. d. pregnancy testing.

ANS: C Chlamydia testing is recommended annually for women with multiple sex partners. There is no indication that the patient needs teaching about contraceptives, pregnancy testing, or antibiotic therapy

Which finding from the nurse's physical assessment of a 42-year-old male patient should be reported to the health care provider? a. One testis hangs lower than the other. b. Genital hair distribution is diamond shaped. c. Clear discharge is present at the penile meatus. d. Inguinal lymph nodes are nonpalpable bilaterally.

ANS: C Clear penile discharge may be indicative of a sexually transmitted infection (STI). The other findings are normal and do not need to be reported.

A 49-year-old woman is considering the use of combined estrogen-progesterone hormone replacement therapy (HT) during menopause. Which information will the nurse include during their discussion? a. Use of estrogen-containing vaginal creams provides most of the same benefits as oral HT. b. Increased incidence of colon cancer in women taking HT requires more frequent colonoscopy. c. HT decreases osteoporosis risk and increases the risk for cardiovascular disease and breast cancer. d. Use of HT for up to 10 years to prevent symptoms such as hot flashes is generally considered safe.

ANS: C Data from the Women's Health Initiative indicate an increased risk for cardiovascular disease and breast cancer in women taking combination HT but a decrease in hip fractures. Vaginal creams decrease symptoms related to vaginal atrophy and dryness, but they do not offer the other benefits of HT, such as decreased hot flashes. Most women who use HT are placed on short-term treatment and are not treated for up to 10 years. The incidence of colon cancer decreases in women taking HRT

Which question should the nurse ask when assessing a 60-year-old patient who has a history of benign prostatic hyperplasia (BPH)? a. "Have you noticed any unusual discharge from your penis?" b. "Has there been any change in your sex life in the last year?" c. "Has there been a decrease in the force of your urinary stream?" d. "Have you been experiencing any difficulty in achieving an erection?"

ANS: C Enlargement of the prostate blocks the urethra, leading to urinary changes such as a decrease in the force of the urinary stream. The other questions address possible problems with infection or sexual difficulties, but they would not be helpful in determining whether there were functional changes caused by BPH.

Which information noted by the nurse when caring for a patient with a bladder infection is most important to report to the health care provider? a. Dysuria b. Hematuria c. Left-sided flank pain d. Temperature 100.1° F

ANS: C Flank pain indicates that the patient may have developed pyelonephritis as a complication of the bladder infection. The other clinical manifestations are consistent with a lower urinary tract infection (UTI). DIF: Cognitive Level: Application REF: 1132-1133

Which information will the nurse teach a 48-year-old patient who has been newly diagnosed with Graves' disease? a. Exercise is contraindicated to avoid increasing metabolic rate. b. Restriction of iodine intake is needed to reduce thyroid activity. c. Antithyroid medications may take several months for full effect. d. Surgery will eventually be required to remove the thyroid gland.

ANS: C Medications used to block the synthesis of thyroid hormones may take 2 to 3 months before the full effect is seen. Large doses of iodine are used to inhibit the synthesis of thyroid hormones. Exercise using large muscle groups is encouraged to decrease the irritability and hyperactivity associated with high levels of thyroid hormones. Radioactive iodine is the most common treatment for Graves' disease although surgery may be used.

An expected nursing diagnosis for a 30-year-old patient admitted to the hospital with symptoms of diabetes insipidus is a. excess fluid volume related to intake greater than output. b. impaired gas exchange related to fluid retention in lungs. c. sleep pattern disturbance related to frequent waking to void. d. risk for impaired skin integrity related to generalized edema.

ANS: C Nocturia occurs as a result of the polyuria caused by diabetes insipidus. Edema, excess fluid volume, and fluid retention are not expected.

A 26-year-old patient with a history of polycystic kidney disease is admitted to the surgical unit after having knee surgery. Which of the routine postoperative orders is most important for the nurse to discuss with the health care provider? a. Infuse 5% dextrose in normal saline at 75 mL/hr. b. Order regular diet after patient is awake and alert. c. Give ketorolac (Toradol) 10 mg PO PRN for pain. d. Obtain blood urea nitrogen (BUN), creatinine, and electrolytes in 2 hours.

ANS: C The NSAIDs should be avoided in patients with decreased renal function because nephrotoxicity is a potential adverse effect. The other orders do not need any clarification or change. DIF: Cognitive Level: Application REF: 1142-1143

An 18-year-old requests a prescription for birth control pills to control severe abdominal cramping and headaches during her menstrual periods. Which should the nurse take first? a. Determine whether the patient is sexually active. b. Teach about the side effects of oral contraceptives. c. Take a personal and family health history from the patient. d. Suggest nonsteroidal antiinflammatory drugs (NSAIDs) for relief.

ANS: C Oral contraceptives may be appropriate to control this patient's symptoms, but the patient's health history may indicate contraindications to oral contraceptive use. Because the patient is requesting contraceptives for management of dysmenorrhea, whether she is sexually active is irrelevant. Because the patient is asking for birth control pills, responding that she should try NSAIDs is nontherapeutic. The patient does not need teaching about oral contraceptive side effects at this time.

A patient's renal calculus is analyzed as being very high in uric acid. To prevent recurrence of stones, the nurse teaches the patient to avoid eating a. milk and dairy products. b. legumes and dried fruits. c. organ meats and sardines. d. spinach, chocolate, and tea.

ANS: C Organ meats and fish such as sardines increase purine levels and uric acid. Spinach, chocolate, and tomatoes should be avoided in patients who have oxalate stones. Milk, dairy products, legumes, and dried fruits may increase the incidence of calcium-containing stones. DIF: Cognitive Level: Application REF: 1139

Which information will the nurse include when teaching the patient with a urinary tract infection (UTI) about the use of phenazopyridine (Pyridium)? a. Take the medication for at least 7 days. b. Use sunscreen while taking the Pyridium. c. The urine may turn a reddish-orange color. d. Use the Pyridium before sexual intercourse.

ANS: C Patients should be taught that Pyridium will color the urine deep orange. Urinary analgesics should only be needed for a few days until the prescribed antibiotics decrease the bacterial count. Taking Pyridium before intercourse will not be helpful in reducing the risk for UTI. Pyridium does not cause photosensitivity.

Which patient in the women's health clinic will the nurse expect to teach about an endometrial biopsy? a. The 55-year-old patient who has 3 to 4 alcoholic drinks each day b. The 35-year-old patient who has used oral contraceptives for 15 years c. The 25-year-old patient who has a family history of hereditary nonpolyposis colorectal cancer d. The 45-year-old patient who has had 6 full-term pregnancies and 2 spontaneous abortions

ANS: C Patients with a personal or familial history of hereditary nonpolyposis colorectal cancer are at increased risk for endometrial cancer. Alcohol addiction does not increase this risk. Multiple pregnancies and oral contraceptive use offer protection from endometrial cancer

Which action should the nurse take when a 35-year-old patient has a result of minor cellular changes on her Pap test? a. Teach the patient about colposcopy. b. Teach the patient about punch biopsy. c. Schedule another Pap test in 4 months. d. Administer the human papillomavirus (HPV) vaccine.

ANS: C Patients with minor changes on the Pap test can be followed with Pap tests every 4 to 6 months because these changes may revert to normal. Punch biopsy or colposcopy may be used if the Pap test shows more prominent changes. The HPV vaccine may reduce the risk for cervical cancer, but it is recommended only for ages 9 through 26.

A 37-year-old patient is being admitted with a diagnosis of Cushing syndrome. Which findings will the nurse expect during the assessment? a. Chronically low blood pressure b. Bronzed appearance of the skin c. Purplish streaks on the abdomen d. Decreased axillary and pubic hair

ANS: C Purplish-red striae on the abdomen are a common clinical manifestation of Cushing syndrome. Hypotension and bronzed-appearing skin are manifestations of Addison's disease. Decreased axillary and pubic hair occur with androgen deficiency.

Which finding by the nurse for a patient admitted with glomerulonephritis indicates that treatment has been effective? a. The patient denies pain with voiding. b. The urine dipstick is negative for nitrites. c. Peripheral and periorbital edema is resolved. d. The antistreptolysin-O (ASO) titer is decreased.

ANS: C Since edema is a common clinical manifestation of glomerulonephritis, resolution of the edema indicates that the prescribed therapies have been effective. Antibodies to streptococcus will persist after a streptococcal infection. Nitrites will be negative and the patient will not experience dysuria since the patient does not have a urinary tract infection. DIF: Cognitive Level: Application REF: 1131-1133

A 49-year-old man who has type 2 diabetes, high blood pressure, hyperlipidemia, and gastroesophageal reflux tells the nurse that he has had recent difficulty in achieving an erection. Which of the following drugs from his current medications list may cause erectile dysfunction (ED)? a. Ranitidine (Zantac) b. Atorvastatin (Lipitor) c. Propranolol (Inderal) d. Metformin (Glucophage)

ANS: C Some antihypertensives may cause erectile dysfunction, and the nurse should anticipate a change in antihypertensive therapy. The other medications will not affect erectile function.

Which information obtained by the nurse in the endocrine clinic about a patient who has been taking prednisone (Deltasone) 40 mg daily for 3 weeks is most important to report to the health care provider? a. Patient's blood pressure is 148/94 mm Hg. b. Patient has bilateral 2+ pitting ankle edema. c. Patient stopped taking the medication 2 days ago. d. Patient has not been taking the prescribed vitamin D.

ANS: C Sudden cessation of corticosteroids after taking the medication for a week or more can lead to adrenal insufficiency, with problems such as severe hypotension and hypoglycemia. The patient will need immediate evaluation by the health care provider to prevent and/or treat adrenal insufficiency. The other information will also be reported, but does not require rapid treatment.

Which assessment finding for a patient who has had a cystectomy with an ileal conduit the previous day is most important for the nurse to communicate to the physician? a. Cloudy appearing urine b. Hypotonic bowel sounds c. Heart rate 102 beats/minute d. Continuous drainage from stoma

ANS: C Tachycardia may indicate infection, hemorrhage, or hypovolemia, which are all serious complications of this surgery. The urine from an ileal conduit normally contains mucus and is cloudy. Hypotonic bowel sounds are expected after bowel surgery. Continuous drainage of urine from the stoma is normal. DIF: Cognitive Level: Application REF: 1157 | 1159-1160 | 1158-1159

Which assessment finding of a 42-year-old patient who had a bilateral adrenalectomy requires the most rapid action by the nurse? a. The blood glucose is 176 mg/dL. b. The lungs have bibasilar crackles. c. The blood pressure (BP) is 88/50 mm Hg. d. The patient reports 5/10 incisional pain.

ANS: C The decreased BP indicates possible adrenal insufficiency. The nurse should immediately notify the health care provider so that corticosteroid medications can be administered. The nurse should also address the elevated glucose, incisional pain, and crackles with appropriate collaborative or nursing actions, but prevention and treatment of acute adrenal insufficiency is the priority after adrenalectomy.

The nurse is caring for a 20-year-old patient with pelvic inflammatory disease (PID) requiring hospitalization. Which nursing intervention will be included in the plan of care? a. Monitor liver function tests. b. Use cold packs PRN for pelvic pain. c. Elevate the head of the bed to at least 30 degrees. d. Teach the patient how to perform Kegel exercises.

ANS: C The head of the bed should be elevated to at least 30 degrees to promote drainage of the pelvic cavity and prevent abscess formation higher in the abdomen. Although a possible complication of PID is acute perihepatitis, liver function tests will remain normal. There is no indication for increased fluid intake. Application of heat is used to reduce pain. Kegel exercises are not helpful in PID

A 47-year-old woman asks whether she is going into menopause if she has not had a menstrual period for 3 months. The best response by the nurse is which of the following? a. "Have you thought about using hormone replacement therapy?" b. "Most women feel a little depressed about entering menopause." c. "What was your menstrual pattern before your periods stopped?" d. "Since you are in your mid-40s, it is likely that you are menopausal."

ANS: C The initial response by the nurse should be to assess the patient's baseline menstrual pattern. Although many women do enter menopause in the mid-40s, more information about this patient is needed before telling her that it is likely she is menopausal. Although hormone therapy (HT) may be prescribed, further assessment of the patient is needed before discussing therapies for menopause. Because the response to menopause is very individual, the nurse should not assume that the patient is experiencing any adverse emotional reactions.

Which assessment finding for a patient who has just been admitted with acute pyelonephritis is most important for the nurse to report to the health care provider? a. Foul-smelling urine b. Complaint of flank pain c. Blood pressure 88/45 mm Hg d. Temperature 100.1° F (57.8° C)

ANS: C The low blood pressure indicates that urosepsis and septic shock may be occurring and should be immediately reported. The other findings are typical of pyelonephritis. DIF: Cognitive Level: Application REF: 1126

A patient is admitted to the hospital with new onset nephrotic syndrome. Which assessment data will the nurse expect to find related to this illness? a. Poor skin turgor b. High urine ketones c. Recent weight gain d. Low blood pressure

ANS: C The patient with a nephrotic syndrome will have weight gain associated with edema. Hypertension is a clinical manifestation of nephrotic syndrome. Skin turgor is normal because of the edema. Urine protein is high. DIF: Cognitive Level: Comprehension REF: 1132-1134

A nursing diagnosis that is likely to be appropriate for a 67-year-old woman who has just been diagnosed with stage III ovarian cancer is a. sexual dysfunction related to loss of vaginal sensation. b. risk for infection related to impaired immune function. c. anxiety related to cancer diagnosis and need for treatment decisions. d. situational low self-esteem related to guilt about delaying medical care.

ANS: C The patient with stage III ovarian cancer is likely to be anxious about the poor prognosis and about the need to make decisions about the multiple treatments that may be used. Decreased vaginal sensation does not occur with ovarian cancer. The patient may develop immune dysfunction when she receives chemotherapy, but she is not currently at risk. It is unlikely that the patient has delayed seeking medical care because the symptoms of ovarian cancer are vague and occur late in the course of the cancer

The nurse has just received change-of-shift report about the following four patients. Which patient should be assessed first? a. A patient with a cervical radium implant in place who is crying in her room b. A patient who is complaining of 5/10 pain after an abdominal hysterectomy c. A patient with a possible ectopic pregnancy who is complaining of shoulder pain d. A patient in the fifteenth week of gestation who has uterine cramping and spotting

ANS: C The patient with the ectopic pregnancy has symptoms consistent with rupture and needs immediate assessment for signs of hemorrhage and possible transfer to surgery. The other patients should also be assessed as quickly as possible but do not have symptoms of life-threatening complications

When assessing a 30-year-old man who complains of a feeling of incomplete bladder emptying and a split, spraying urine stream, the nurse asks about a history of a. bladder infection. b. recent kidney trauma. c. gonococcal urethritis. d. benign prostatic hyperplasia.

ANS: C The patient's clinical manifestations are consistent with urethral strictures, a possible complication of gonococcal urethritis. These symptoms are not consistent with benign prostatic hyperplasia, kidney trauma, or bladder infection. DIF: Cognitive Level: Application REF: 1141

The following patients call the outpatient clinic. Which phone call should the nurse return first? a. A 44-year-old patient who has bloody discharge after a hysteroscopy earlier today b. A 64-year-old patient who is experiencing shoulder pain after a laparoscopy yesterday c. A 34-year-old patient who is short of breath after pelvic computed tomography (CT) with contrast d. A 54-year-old patient who has severe breast tenderness following a needle aspiration breast biopsy

ANS: C The patient's dyspnea suggests a delayed reaction to the iodine dye used for the CT scan. The other patient's symptoms are not unusual after the procedures they had done.

A female patient tells the nurse that she has been having nightmares and acute anxiety around men since being sexually assaulted 3 months ago. The most appropriate nursing diagnosis for the patient is a. anxiety related to effects of being raped. b. sleep deprivation related to frightening dreams. c. rape-trauma syndrome related to rape experience. d. ineffective coping related to inability to resolve incident.

ANS: C The patient's symptoms are most consistent with the nursing diagnosis of rape-trauma syndrome. The nursing diagnoses of sleep deprivation, ineffective coping, and anxiety address some aspects of the patient's symptoms but do not address the problem as completely as the rape-trauma syndrome diagnosis.

A patient develops carpopedal spasms and tingling of the lips following a parathyroidectomy. Which action should the nurse take first? a. Administer the ordered muscle relaxant. b. Give the ordered oral calcium supplement. c. Have the patient rebreathe from a paper bag. d. Start the PRN oxygen at 2 L/min per cannula.

ANS: C The patient's symptoms suggest mild hypocalcemia. The symptoms of hypocalcemia will be temporarily reduced by having the patient breathe into a paper bag, which will raise the PaCO2 and create a more acidic pH. The muscle relaxant will have no impact on the ionized calcium level. Although severe hypocalcemia can cause laryngeal stridor, there is no indication that this patient is experiencing laryngeal stridor or needs oxygen. Calcium supplements will be given to normalize calcium levels quickly, but oral supplements will take time to be absorbed.

The nurse in the infertility clinic is explaining in vitro fertilization (IVF) to a couple. The woman tells the nurse that they cannot afford IVF on her husband's salary. The man replies that if his wife worked outside the home, they would have enough money. Which nursing diagnosis is appropriate? a. Decisional conflict related to inadequate financial resources b. Ineffective sexuality patterns related to psychological stress c. Defensive coping related to anxiety about lack of conception d. Ineffective denial related to frustration about continued infertility

ANS: C The statements made by the couple are consistent with the diagnosis of defensive coping. No data indicate that ineffective sexuality and ineffective denial are problems. Although the couple is quarreling about finances, the data do not provide information indicating that the finances are inadequate

A 62-year-old patient with hyperthyroidism is to be treated with radioactive iodine (RAI). The nurse instructs the patient a. about radioactive precautions to take with all body secretions. b. that symptoms of hyperthyroidism should be relieved in about a week. c. that symptoms of hypothyroidism may occur as the RAI therapy takes effect. d. to discontinue the antithyroid medications taken before the radioactive therapy.

ANS: C There is a high incidence of postradiation hypothyroidism after RAI, and the patient should be monitored for symptoms of hypothyroidism. RAI has a delayed response, with the maximum effect not seen for 2 to 3 months, and the patient will continue to take antithyroid medications during this time. The therapeutic dose of radioactive iodine is low enough that no radiation safety precautions are needed.

A 42-year-old female patient is scheduled for transsphenoidal hypophysectomy to treat a pituitary adenoma. During preoperative teaching, the nurse instructs the patient about the need to a. cough and deep breathe every 2 hours postoperatively. b. remain on bed rest for the first 48 hours after the surgery. c. avoid brushing teeth for at least 10 days after the surgery. d. be positioned flat with sandbags at the head postoperatively.

ANS: C To avoid disruption of the suture line, the patient should avoid brushing the teeth for 10 days after surgery. It is not necessary to remain on bed rest after this surgery. Coughing is discouraged because it may cause leakage of cerebrospinal fluid (CSF) from the suture line. The head of the bed should be elevated 30 degrees to reduce pressure on the sella turcica and decrease the risk for headaches.

An 82-year-old patient in a long-term care facility has several medications prescribed. After the patient is newly diagnosed with hypothyroidism, the nurse will need to consult with the health care provider before administering a. docusate (Colace). b. ibuprofen (Motrin). c. diazepam (Valium). d. cefoxitin (Mefoxin).

ANS: C Worsening of mental status and myxedema coma can be precipitated by the use of sedatives, especially in older adults. The nurse should discuss the use of diazepam with the health care provider before administration. The other medications may be given safely to the patient.

Two hours after a closed percutaneous kidney biopsy, the client reports a dramatic increase in pain. What is the nurse's best first action? A. Reposition the client on the operative side. B. Administer prescribed opioid analgesic. C. Assess pulse rate and blood pressure. D. Check the Foley catheter for kinks.

ANS: C An increase in the intensity of pain after a percutaneous kidney biopsy is a symptom of internal hemorrhage.

With a renal threshold for glucose of 220 mg/dL, what is the expected response when a client has a blood glucose level of 400 mg/dL? A. 400 mg/dL of excreted glucose in the urine B. 220 mg/dL of excreted glucose in the urine C. 180 mg/dL of glucose is excreted in the urine D. No excreted glucose in the urine

ANS: C Blood glucose is freely filtered at the glomerulus. Therefore, if a client has a blood sugar level of 400 mg/dl, the filtrate in the proximal convoluted tubule will have a glucose concentration of 400 mg/dL. With a renal threshold of 220 mg/dl, a total of 220 mg/dL of the 400 mg/dL will be reabsorbed back into the systemic circulation, and the final urine will have a glucose concentration of 180 mg/dL.

The client is going home after urography. Which instruction or precaution should the nurse teach this client? A. "Avoid direct contact with the urine for 24 hours until the radioisotope clears." B. "You are likely to experience some dribbling of urine for several weeks after this procedure." C. "Be sure to drink at least 3 L of fluids today to help eliminate the dye faster." D. "Your skin may become slightly yellow-tinged from the dye used in this procedure."

ANS: C Dyes used in urography are potentially nephrotoxic.

6. What is the result of stimulation of erythropoietin production in the kidney tissue? A. Increased blood flow to the kidney B. Inhibition of vitamin D and loss of bone density C. Increased bone marrow production of red blood cells D. Inhibition of the active transport of sodium, leading to hyponatremi

ANS: C Erythropoietin is produced in the kidney and released in response to decreased oxygen tension in the renal blood supply. Erythropoietin stimulates red blood cell (RBC) production in the bone marrow.

Confirmed by palpation and x-ray study, the client's right kidney is lower than the left kidney. What is the nurse's interpretation of this finding? A. The client has a problem involving the right kidney. B. The client has a problem involving the left kidney. C. The client has both kidneys in the normal position. D. The client is at increased risk for kidney impairment.

ANS: C Normally, the right kidney is positioned somewhat lower than the left kidney. This anatomic difference in otherwise symmetric organs is caused by liver displacement. The significance of this difference is that the right kidney is easier to palpate in an adult than is the left kidney.

The client has an elevated blood urea nitrogen (BUN) level and an increased ratio of blood urea nitrogen to creatinine. What is the nurse's interpretation of these laboratory results? A. The client probably has a urinary tract infection. B. The client may be overhydrated. C. The kidney may be hypoperfused. D. The kidney may be damaged.

ANS: C When dehydration or renal hypoperfusion exist, the BUN level rises more rapidly than the serum creatinine level, causing the ratio to be increased, even when no renal dysfunction is present.

To prevent the recurrence of renal calculi, the nurse teaches the patient to: a. use a filter to strain all urine. b. avoid dietary sources of calcium. c. drink diuretic fluids such as coffee. d. have 2000 to 3000 mL of fluid a day.

ANS: D A fluid intake of 2000 to 3000 mL daily is recommended to help flush out minerals before stones can form. Avoidance of calcium is not usually recommended for patients with renal calculi. Coffee tends to increase stone recurrence. There is no need for a patient to strain all urine routinely after a stone has passed, and this will not prevent stones.

When the nurse is caring for a patient who has had left-sided extracorporeal shock wave lithotripsy, which assessment finding is most important to report to the health care provider? a. Blood in urine b. Left flank pain c. Left flank bruising d. Drop in urine output

ANS: D Because lithotripsy breaks the stone into small sand, which could cause obstruction, it is important to report a drop in urine output. Left flank pain, bruising, and hematuria are common after lithotripsy. DIF: Cognitive Level: Application REF: 1138-1139

The nurse notes that a patient who has a large cystocele, admitted 10 hours ago, has not yet voided. Which action should the nurse take first? a. Insert a straight catheter per the PRN order. b. Encourage the patient to increase oral fluids. c. Notify the health care provider of the inability to void. d. Use an ultrasound scanner to check for urinary retention.

ANS: D Because urinary retention is common with a large cystocele, the nurse's first action should be to use an ultrasound bladder scanner to check for the presence of urine in the bladder. The other actions may be appropriate, depending on the findings with the bladder scanner

A 31-year-old patient has just been instructed in the treatment for a Chlamydia trachomatis vaginal infection. Which patient statement indicates that the nurse's teaching has been effective? a. "I can purchase an over-the-counter medication to treat this infection." b. "The symptoms are due to the overgrowth of normal vaginal bacteria." c. "The medication will need to be inserted once daily with an applicator." d. "Both my partner and I will need to take the medication for a full week."

ANS: D Chlamydia is a sexually transmitted bacterial infection that requires treatment of both partners with antibiotics for 7 days. The other statements are true for the treatment of Candida albicans infection

The nurse explains to a 37-year-old patient being prepared for colposcopy with a cervical biopsy that the procedure a. involves dilation of the cervix and biopsy of the tissue lining the uterus. b. will take place in a same-day surgery center so that local anesthesia can be used. c. requires that the patient have nothing to eat or drink for 6 hours before the procedure. d. is similar to a speculum examination of the cervix and should result in little discomfort.

ANS: D Colposcopy involves visualization of the cervix with a binocular microscope and is similar to a speculum examination. Anesthesia is not required and fasting is not necessary. A cervical biopsy may cause a minimal amount of pain.

A 72-year-old who has benign prostatic hyperplasia is admitted to the hospital with chills, fever, and vomiting. Which finding by the nurse will be most helpful in determining whether the patient has an upper urinary tract infection (UTI)? a. Suprapubic pain b. Bladder distention c. Foul-smelling urine d. Costovertebral tenderness

ANS: D Costovertebral tenderness is characteristic of pyelonephritis. The other symptoms are characteristic of lower UTI and are likely to be present if the patient also has an upper UTI. DIF: Cognitive Level: Application REF: 1128

A 62-year-old asks the nurse for a perineal pad, stating that laughing or coughing causes leakage of urine. Which intervention is most appropriate to include in the care plan? a. Assist the patient to the bathroom q3hr. b. Place a commode at the patient's bedside. c. Demonstrate how to perform the Credé maneuver. d. Teach the patient how to perform Kegel exercises.

ANS: D Exercises to strengthen the pelvic floor muscles will help reduce stress incontinence. The Credé maneuver is used to help empty the bladder for patients with overflow incontinence. Placing the commode close to the bedside and assisting the patient to the bathroom are helpful for functional incontinence. DIF: Cognitive Level: Application REF: 1148

A 78-year-old who has been admitted to the hospital with dehydration is confused and incontinent of urine. Which nursing action will be best to include in the plan of care? a. Apply absorbent incontinent pads. b. Restrict fluids after the evening meal. c. Insert an indwelling catheter until the symptoms have resolved. d. Assist the patient to the bathroom every 2 hours during the day.

ANS: D In older or confused patients, incontinence may be avoided by using scheduled toileting times. Indwelling catheters increase the risk for urinary tract infection (UTI). Incontinent pads increase the risk for skin breakdown. Restricting fluids is not appropriate in a patient with dehydration. DIF: Cognitive Level: Application REF: 1151-1152

A 49-year-old woman tells the nurse that she is postmenopausal but has occasional spotting. Which initial response by the nurse is most appropriate? a. "A frequent cause of spotting is endometrial cancer." b. "How long has it been since your last menstrual period?" c. "Breakthrough bleeding is not unusual in women your age." d. "Are you using prescription hormone replacement therapy?"

ANS: D In postmenopausal women, a common cause of spotting is hormone therapy (HT). Because breakthrough bleeding may be a sign of problems such as cancer or infection, the nurse would not imply that this is normal. The length of time since the last menstrual period is not relevant to the patient's symptoms. Although endometrial cancer may cause spotting, this information is not appropriate as an initial response

A 19-year-old has been diagnosed with primary dysmenorrhea. How will the nurse suggest that the patient prevent discomfort? a. Avoid aerobic exercise during her menstrual period. b. Use cold packs on the abdomen and back for pain relief. c. Talk with her health care provider about beginning antidepressant therapy. d. Take nonsteroidal antiinflammatory drugs (NSAIDs) when her period starts.

ANS: D NSAIDs should be started as soon as the menstrual period begins and taken at regular intervals during the usual time frame in which pain occurs. Aerobic exercise may help reduce symptoms. Heat therapy, such as warm packs, is recommended for relief of pain. Antidepressant therapy is not a typical treatment for dysmenorrhea

Which finding by the nurse when assessing a patient with a large pituitary adenoma is most important to report to the health care provider? a. Changes in visual field b. Milk leaking from breasts c. Blood glucose 150 mg/dL d. Nausea and projectile vomiting

ANS: D Nausea and projectile vomiting may indicate increased intracranial pressure, which will require rapid actions for diagnosis and treatment. Changes in the visual field, elevated blood glucose, and galactorrhea are common with pituitary adenoma, but these do not require rapid action to prevent life-threatening complications.

19. A 28-year-old patient was recently diagnosed with polycystic ovary syndrome. It is most important for the nurse to teach the patient a. reasons for a total hysterectomy. b. how to decrease facial hair growth. c. ways to reduce the occurrence of acne. d. methods to maintain appropriate weight.

ANS: D Obesity exacerbates the problems associated with polycystic ovary syndrome, such as insulin resistance and type 2 diabetes. The nurse should also address the problems of acne and hirsutism, but these symptoms are lower priority because they do not have long-term health consequences. Although some patients do require total hysterectomy, this is usually performed only after other therapies have been unsuccessful

The nurse will plan to teach a 51-year-old man who is scheduled for an annual physical exam about a(n) a. increased risk for testicular cancer. b. possible changes in erectile function. c. normal decreases in testosterone level. d. prostate specific antigen (PSA) testing.

ANS: D PSA testing may be recommended annually for men, starting at age 50. There is no indication that the other patient teaching topics are appropriate for this patient.

When assessing the patient who has a lower urinary tract infection (UTI), the nurse will initially ask about a. nausea. b. flank pain. c. poor urine output. d. pain with urination.

ANS: D Pain with urination is a common symptom of a lower UTI. Urine output does not decrease, but frequency may be experienced. Flank pain and nausea are associated with an upper UTI. DIF: Cognitive Level: Application REF: 1123-1124

After a 26-year-old patient has been treated for pelvic inflammatory disease, the nurse will plan to teach about a. use of hormone therapy (HT). b. irregularities in the menstrual cycle. c. changes in secondary sex characteristics. d. possible difficulty with becoming pregnant.

ANS: D Pelvic inflammatory disease may cause scarring of the fallopian tubes and result in difficulty in fertilization or implantation of the fertilized egg. Because ovarian function is not affected, the patient will not require HT, have irregular menstrual cycles, or experience changes in secondary sex characteristics.

To prevent pregnancy in a patient who has been sexually assaulted, the nurse in the emergency department will plan to teach the patient about the use of a. mifepristone (RU-486). b. dilation and evacuation. c. methotrexate with misoprostol. d. levonorgestrel (Plan-B One-Step).

ANS: D Plan B One-Step reduces the risk of pregnancy when taken within 72 hours of intercourse. The other methods are used for therapeutic abortion, but not for pregnancy prevention after unprotected intercourse

A woman calls the clinic because she is having an unusually heavy menstrual flow. She tells the nurse that she has saturated three tampons in the past 2 hours. The nurse estimates that the amount of blood loss over the past 2 hours is _____ mL. a. 20 to 30 b. 30 to 40 c. 40 to 60 d. 60 to 90

ANS: D The average tampon absorbs 20 to 30 mL.

The nurse observes nursing assistive personnel (NAP) taking the following actions when caring for a patient with a retention catheter. Which action requires that the nurse intervene? a. Taping the catheter to the skin on the patient's upper inner thigh b. Cleaning around the patient's urinary meatus with soap and water c. Using an alcohol-based hand cleaner before performing catheter care d. Disconnecting the catheter from the drainage tube to obtain a specimen

ANS: D The catheter should not be disconnected from the drainage tube because this increases the risk for urinary tract infection (UTI). The other actions are appropriate and do not require any intervention. DIF: Cognitive Level: Application REF: 1152-1154

Which action will the nurse include in the plan of care for a patient who has had a ureterolithotomy and has a left ureteral catheter and a urethral catheter in place? a. Provide education about home care for both catheters. b. Apply continuous steady tension to the ureteral catheter. c. Clamp the ureteral catheter unless output from the urethral catheter stops. d. Call the health care provider if the ureteral catheter output drops suddenly.

ANS: D The health care provider should be notified if the ureteral catheter output decreases since obstruction of this catheter may result in an increase in pressure in the renal pelvis. Tension on the ureteral catheter should be avoided in order to prevent catheter displacement. To avoid pressure in the renal pelvis, the catheter is not clamped. Since the patient is not usually discharged with a ureteral catheter in place, patient teaching about both catheters is not needed. DIF: Cognitive Level: Application REF: 1153-1154

A 68-year-old male patient tells the nurse that he is worried because he does not respond to sexual stimulation the same way he did when he was younger. The nurse's best response to the patient's concern is which of the following? a. "Interest in sex frequently decreases as men get older." b. "Many men need additional sexual stimulation with aging." c. "Erectile dysfunction is a common problem with older men." d. "Tell me more about how your sexual response has changed."

ANS: D The initial response by the nurse should be further assessment of the problem. The other statements by the nurse are accurate but may not respond to the patient's concerns.

A 44-year-old female patient with Cushing syndrome is admitted for adrenalectomy. Which intervention by the nurse will be most helpful for a nursing diagnosis of disturbed body image related to changes in appearance? a. Reassure the patient that the physical changes are very common in patients with Cushing syndrome. b. Discuss the use of diet and exercise in controlling the weight gain associated with Cushing syndrome. c. Teach the patient that the metabolic impact of Cushing syndrome is of more importance than appearance. d. Remind the patient that most of the physical changes caused by Cushing syndrome will resolve after surgery.

ANS: D The most reassuring communication to the patient is that the physical and emotional changes caused by the Cushing syndrome will resolve after hormone levels return to normal postoperatively. Reassurance that the physical changes are expected or that there are more serious physiologic problems associated with Cushing syndrome are not therapeutic responses. The patient's physiological changes are caused by the high hormone levels, not by the patient's diet or exercise choices.

When caring for a patient who has a radium implant for treatment of cancer of the cervix, the nurse will a. assist the patient to ambulate every 2 to 3 hours. b. use gloves and gown when changing the patient's bed. c. flush the toilet several times right after the patient voids. d. encourage the patient to discuss needs or concerns by telephone.

ANS: D The nurse should spend minimal time in the patient's room to avoid exposure to radiation. The patient and nurse can have longer conversations by telephone between the patient room and nursing station. To prevent displacement of the implant, absolute bed rest is required. Wearing of gloves and gown when changing linens, and flushing the toilet several times are not necessary because the isotope is confined to the implant

A patient with bladder cancer is scheduled for intravesical chemotherapy. In preparation for the treatment the nurse will teach the patient about a. premedicating to prevent nausea. b. where to obtain wigs and scarves. c. the importance of oral care during treatment. d. the need to empty the bladder before treatment.

ANS: D The patient will be asked to empty the bladder before instillation of the chemotherapy. Systemic side effects are not experienced with intravesical chemotherapy. DIF: Cognitive Level: Application REF: 1146

The nurse in the women's health clinic has four patients who are waiting to be seen. Which patient should the nurse see first? a. 22-year-old with persistent red-brown vaginal drainage 3 days after having balloon thermotherapy b. 42-year-old with secondary amenorrhea who says that her last menstrual cycle was 3 months ago c. 35-year-old with heavy spotting after having a progestin-containing IUD (Mirena) inserted a month ago d. 19-year-old with menorrhagia who has been using superabsorbent tampons and has fever with weakness

ANS: D The patient's history and clinical manifestations suggest possible toxic shock syndrome, which will require rapid intervention. The symptoms for the other patients are consistent with their diagnoses and do not indicate life-threatening complications

The nurse determines that instruction regarding prevention of future urinary tract infections (UTIs) for a patient with cystitis has been effective when the patient states, a. "I can use vaginal sprays to reduce bacteria." b. "I will drink a quart of water or other fluids every day." c. "I will wash with soap and water before sexual intercourse." d. "I will empty my bladder every 3 to 4 hours during the day."

ANS: D Voiding every 3 to 4 hours is recommended to prevent UTIs. Use of vaginal sprays is dis- couraged. The bladder should be emptied before and after intercourse, but cleaning with soap and water is not necessary. A quart of fluids is insufficient to provide adequate urine output to decrease risk for UTI.

A nurse observes that the client's left flank region is larger than the right flank region. What is the nurse's best action? A. Ask the client if he or she participates in contact sports and has been recently injured. B. Document the finding as the only action on the appropriate flowsheet. C. Apply a heating pad to the left flank after inspecting the site for signs of infection. D. Anticipate further diagnostic testing after sharing informing the physician of this finding

ANS: D Asymmetry of the flank or a unilateral protrusion may indicate an enlargement of a kidney. The enlargement may be benign or may be associated with a hydronephrosis or mass on the kidney.

Which change in renal or urinary functioning as a result of the normal aging process increases the older client's risk for infection? A. Decreased glomerular filtration B. Decreased filtrate reabsorption C. Weakened sphincter muscles D. Urinary retention

ANS: D Incomplete bladder emptying for whatever reason increases the client's risk for urinary tract infections as a result of urine stasis providing an excellent culture medium that promotes the growth of microorganisms.

The client reports the regular use of all the following medications. Which one alerts the nurse to the possibility of renal impairment when used consistently? A. Antacids B. Penicillin C. Antihistamine nasal sprays D. Nonsteroidal anti-inflammatory drug

ANS: D NSAIDs inhibit prostaglandin production and decrease blood flow to the nephrons. They can cause an interstitial nephritis and renal impairment.

The most recent blood work of a patient with a longstanding diagnosis of type 1 diabetes has shown the presence of microalbuminuria. What is the nurse's most appropriate action? A) Teach the patient about actions to slow the progression of nephropathy. B) Ensure that the patient receives a comprehensive assessment of liver function. C) Determine whether the patient has been using expired insulin. D) Administer a fluid challenge and have the test repeated.

Ans: A Feedback: Clinical nephropathy eventually develops in more than 85% of people with microalbuminuria. As such, educational interventions addressing this microvascular complication are warranted. Expired insulin does not cause nephropathy, and the patient's liver function is not likely affected. There is no indication for the use of a fluid challenge.

A patient with a longstanding diagnosis of type 1 diabetes has a history of poor glycemic control. The nurse recognizes the need to assess the patient for signs and symptoms of peripheral neuropathy. Peripheral neuropathy constitutes a risk for what nursing diagnosis? A) Infection B) Acute pain C) Acute confusion D) Impaired urinary elimination

Ans: A Feedback: Decreased sensations of pain and temperature place patients with neuropathy at increased risk for injury and undetected foot infections. The neurologic changes associated with peripheral neuropathy do not normally result in pain, confusion, or impairments in urinary function.

A patient has been brought to the emergency department by paramedics after being found unconscious. The patient's Medic Alert bracelet indicates that the patient has type 1 diabetes and the patient's blood glucose is 22 mg/dL (1.2 mmol/L). The nurse should anticipate what intervention? A) IV administration of 50% dextrose in water B) Subcutaneous administration of 10 units of Humalog C) Subcutaneous administration of 12 to 15 units of regular insulin D) IV bolus of 5% dextrose in 0.45% NaCl

Ans: A Feedback: In hospitals and emergency departments, for patients who are unconscious or cannot swallow, 25 to 50 mL of 50% dextrose in water (D50W) may be administered IV for the treatment of hypoglycemia. Five percent dextrose would be inadequate and insulin would exacerbate the patient's condition.

A patient has been living with type 2 diabetes for several years, and the nurse realizes that the patient is likely to have minimal contact with the health care system. In order to ensure that the patient maintains adequate blood sugar control over the long term, the nurse should recommend which of the following? A) Participation in a support group for persons with diabetes B) Regular consultation of websites that address diabetes management C) Weekly telephone "check-ins" with an endocrinologist D) Participation in clinical trials relating to antihyperglycemics

Ans: A Feedback: Participation in support groups is encouraged for patients who have had diabetes for many years as well as for those who are newly diagnosed. This is more interactive and instructive than simply consulting websites. Weekly telephone contact with an endocrinologist is not realistic in most cases. Participation in research trials may or may not be beneficial and appropriate, depending on patients' circumstances.

A diabetes nurse educator is teaching a group of patients with type 1 diabetes about "sick day rules." What guideline applies to periods of illness in a diabetic patient? A) Do not eliminate insulin when nauseated and vomiting. B) Report elevated glucose levels greater than 150 mg/dL. C) Eat three substantial meals a day, if possible. D) Reduce food intake and insulin doses in times of illness.

Ans: A Feedback: The most important issue to teach patients with diabetes who become ill is not to eliminate insulin doses when nausea and vomiting occur. Rather, they should take their usual insulin or oral hypoglycemic agent dose, then attempt to consume frequent, small portions of carbohydrates. In general, blood sugar levels will rise but should be reported if they are greater than 300 mg/dL.

A medical nurse is caring for a patient with type 1 diabetes. The patient's medication administration record includes the administration of regular insulin three times daily. Knowing that the patient's lunch tray will arrive at 11:45, when should the nurse administer the patient's insulin? A) 10:45 B) 11:15 C) 11:45 D) 11:50

Ans: B Feedback: Regular insulin is usually administered 20-30 min before a meal. Earlier administration creates a risk for hypoglycemia; later administration creates a risk for hyperglycemia.

A diabetic educator is discussing "sick day rules" with a newly diagnosed type 1 diabetic. The educator is aware that the patient will require further teaching when the patient states what? A) "I will not take my insulin on the days when I am sick, but I will certainly check my blood sugar every 2 hours." B) "If I cannot eat a meal, I will eat a soft food such as soup, gelatin, or pudding six to eight times a day." C) "I will call the doctor if I am not able to keep liquids in my body due to vomiting or diarrhea." D) "I will call the doctor if my blood sugar is over 300 mg/dL or if I have ketones in my urine."

Ans: A Feedback: The nurse must explanation the "sick day rules" again to the patient who plans to stop taking insulin when sick. The nurse should emphasize that the patient should take insulin agents as usual and test one's blood sugar and urine ketones every 3 to 4 hours. In fact, insulin-requiring patients may need supplemental doses of regular insulin every 3 to 4 hours. The patient should report elevated glucose levels (greater than 300 mg/dL or as otherwise instructed) or urine ketones to the physician. If the patient is not able to eat normally, the patient should be instructed to substitute soft foods such a gelatin, soup, and pudding. If vomiting, diarrhea, or fever persists, the patient should have an intake of liquids every 30 to 60 minutes to prevent dehydration.

A nurse is caring for a patient newly diagnosed with type 1 diabetes. The nurse is educating the patient about self-administration of insulin in the home setting. The nurse should teach the patient to do which of the following? A) Avoid using the same injection site more than once in 2 to 3 weeks. B) Avoid mixing more than one type of insulin in a syringe. C) Cleanse the injection site thoroughly with alcohol prior to injecting. D) Inject at a 45º angle.

Ans: A Feedback: To prevent lipodystrophy, the patient should try not to use the same site more than once in 2 to 3 weeks. Mixing different types of insulin in a syringe is acceptable, within specific guidelines, and the needle is usually inserted at a 90º angle. Cleansing the injection site with alcohol is optional.

The nurse is discussing macrovascular complications of diabetes with a patient. The nurse would address what topic during this dialogue? A) The need for frequent eye examinations for patients with diabetes B) The fact that patients with diabetes have an elevated risk of myocardial infarction C) The relationship between kidney function and blood glucose levels D) The need to monitor urine for the presence of albumin

Ans: B Feedback: Myocardial infarction and stroke are considered macrovascular complications of diabetes, while the effects on vision and renal function are considered to be microvascular.

An occupational health nurse is screening a group of workers for diabetes. What statement should the nurse interpret as suggestive of diabetes? A) "I've always been a fan of sweet foods, but lately I'm turned off by them." B) "Lately, I drink and drink and can't seem to quench my thirst." C) "No matter how much sleep I get, it seems to take me hours to wake up." D) "When I went to the washroom the last few days, my urine smelled odd."

Ans: B Feedback: Classic clinical manifestations of diabetes include the "three Ps": polyuria, polydipsia, and polyphagia. Lack of interest in sweet foods, fatigue, and foul-smelling urine are not suggestive of diabetes.

A patient with type 2 diabetes achieves adequate glycemic control through diet and exercise. Upon being admitted to the hospital for a cholecystectomy, however, the patient has required insulin injections on two occasions. The nurse would identify what likely cause for this short-term change in treatment? A) Alterations in bile metabolism and release have likely caused hyperglycemia. B) Stress has likely caused an increase in the patient's blood sugar levels. C) The patient has likely overestimated her ability to control her diabetes using nonpharmacologic measures. D) The patient's volatile fluid balance surrounding surgery has likely caused unstable blood sugars.

Ans: B Feedback: During periods of physiologic stress, such as surgery, blood glucose levels tend to increase, because levels of stress hormones (epinephrine, norepinephrine, glucagon, cortisol, and growth hormone) increase. The patient's need for insulin is unrelated to the action of bile, the patient's overestimation of previous blood sugar control, or fluid imbalance.

A diabetic patient calls the clinic complaining of having a "flu bug." The nurse tells him to take his regular dose of insulin. What else should the nurse tell the patient? A) "Make sure to stick to your normal diet." B) "Try to eat small amounts of carbs, if possible." C) "Ensure that you check your blood glucose every hour." D) "For now, check your urine for ketones every 8 hours."

Ans: B Feedback: For prevention of DKA related to illness, the patient should attempt to consume frequent small portions of carbohydrates (including foods usually avoided, such as juices, regular sodas, and gelatin). Drinking fluids every hour is important to prevent dehydration. Blood glucose and urine ketones must be assessed every 3 to 4 hours.

A patient with a history of type 1 diabetes has just been admitted to the critical care unit (CCU) for diabetic ketoacidosis. The CCU nurse should prioritize what assessment during the patient's initial phase of treatment? A) Monitoring the patient for dysrhythmias B) Maintaining and monitoring the patient's fluid balance C) Assessing the patient's level of consciousness D) Assessing the patient for signs and symptoms of venous thromboembolism

Ans: B Feedback: In addition to treating hyperglycemia, management of DKA is aimed at correcting dehydration, electrolyte loss, and acidosis before correcting the hyperglycemia with insulin. The nurse should monitor the patient for dysrhythmias, decreased LOC and VTE, but restoration and maintenance of fluid balance is the highest priority.

A nurse is caring for a patient with type 1 diabetes who is being discharged home tomorrow. What is the best way to assess the patient's ability to prepare and self-administer insulin? A) Ask the patient to describe the process in detail. B) Observe the patient drawing up and administering the insulin. C) Provide a health education session reviewing the main points of insulin delivery. D) Review the patient's first hemoglobin A1C result after discharge.

Ans: B Feedback: Nurses should assess the patient's ability to perform diabetes related self-care as soon as possible during the hospitalization or office visit to determine whether the patient requires further diabetes teaching. While consulting a home care nurse is beneficial, an initial assessment should be performed during the hospitalization or office visit. Nurses should directly observe the patient performing the skills such as insulin preparation and infection, blood glucose monitoring, and foot care. Simply questioning the patient about these skills without actually observing performance of the skill is not sufficient. Further education does not guarantee learning.

A diabetes nurse is assessing a patient's knowledge of self-care skills. What would be the most appropriate way for the educator to assess the patient's knowledge of nutritional therapy in diabetes? A) Ask the patient to describe an optimally healthy meal. B) Ask the patient to keep a food diary and review it with the nurse. C) Ask the patient's family what he typically eats. D) Ask the patient to describe a typical day's food intake.

Ans: B Feedback: Reviewing the patient's actual food intake is the most accurate method of gauging the patient's diet.

A patient has just been diagnosed with type 2 diabetes. The physician has prescribed an oral antidiabetic agent that will inhibit the production of glucose by the liver and thereby aid in the control of blood glucose. What type of oral antidiabetic agent did the physician prescribe for this patient? A) A sulfonylurea B) A biguanide C) A thiazolidinedione D) An alpha glucosidase inhibitor

Ans: B Feedback: Sulfonylureas exert their primary action by directly stimulating the pancreas to secrete insulin and therefore require a functioning pancreas to be effective. Biguanides inhibit the production of glucose by the liver and are in used in type 2 diabetes to control blood glucose levels. Thiazolidinediones enhance insulin action at the receptor site without increasing insulin secretion from the beta cells of the pancreas. Alpha glucosidase inhibitors work by delaying the absorption of glucose in the intestinal system, resulting in a lower postprandial blood glucose level.

An elderly patient comes to the clinic with her daughter. The patient is a diabetic and is concerned about foot care. The nurse goes over foot care with the patient and her daughter as the nurse realizes that foot care is extremely important. Why would the nurse feel that foot care is so important to this patient? A) An elderly patient with foot ulcers experiences severe foot pain due to the diabetic polyneuropathy. B) Avoiding foot ulcers may mean the difference between institutionalization and continued independent living. C) Hypoglycemia is linked with a risk for falls; this risk is elevated in older adults with diabetes. D) Oral antihyperglycemics have the possible adverse effect of decreased circulation to the lower extremities.

Ans: B Feedback: The nurse recognizes that providing information on the long-term complications—especially foot and eye problems—associated with diabetes is important. Avoiding amputation through early detection of foot ulcers may mean the difference between institutionalization and continued independent living for the elderly person with diabetes. While the nurse recognizes that hypoglycemia is a dangerous situation and may lead to falls, hypoglycemia is not directly connected to the importance of foot care. Decrease in circulation is related to vascular changes and is not associated with drugs administered for diabetes.

A patient is brought to the emergency department by the paramedics. The patient is a type 2 diabetic and is experiencing HHS. The nurse should identify what components of HHS? Select all that apply. A) Leukocytosis B) Glycosuria C) Dehydration D) Hypernatremia E) Hyperglycemia

Ans: B, C, D, E Feedback: In HHS, persistent hyperglycemia causes osmotic diuresis, which results in losses of water and electrolytes. To maintain osmotic equilibrium, water shifts from the intracellular fluid space to the extracellular fluid space. With glycosuria and dehydration, hypernatremia and increased osmolarity occur. Leukocytosis does not take place.

A patient newly diagnosed with type 2 diabetes is attending a nutrition class. What general guideline would be important to teach the patients at this class? A) Low fat generally indicates low sugar. B) Protein should constitute 30% to 40% of caloric intake. C) Most calories should be derived from carbohydrates. D) Animal fats should be eliminated from the diet.

Ans: C Feedback: Currently, the ADA and the Academy of Nutrition and Dietetics (formerly the American Dietetic Association) recommend that for all levels of caloric intake, 50% to 60% of calories should be derived from carbohydrates, 20% to 30% from fat, and the remaining 10% to 20% from protein.Low fat does not automatically mean low sugar. Dietary animal fat does not need to be eliminated from the diet.

A nurse is providing health education to an adolescent newly diagnosed with type 1 diabetes mellitus and her family. The nurse teaches the patient and family that which of the following nonpharmacologic measures will decrease the body's need for insulin? A) Adequate sleep B) Low stimulation C) Exercise D) Low-fat diet

Ans: C Feedback: Exercise lowers blood glucose, increases levels of HDLs, and decreases total cholesterol and triglyceride levels. Low fat intake and low levels of stimulation do not reduce a patient's need for insulin. Adequate sleep is beneficial in reducing stress, but does not have an effect that is pronounced as that of exercise.

A patient with type 1 diabetes mellitus is seeing the nurse to review foot care. What would be a priority instruction for the nurse to give the patient? A) Examine feet weekly for redness, blisters, and abrasions. B) Avoid the use of moisturizing lotions. C) Avoid hot-water bottles and heating pads. D) Dry feet vigorously after each bath.

Ans: C Feedback: High-risk behaviors, such as walking barefoot, using heating pads on the feet, wearing open-toed shoes, soaking the feet, and shaving calluses, should be avoided. Socks should be worn for warmth. Feet should be examined each day for cuts, blisters, swelling, redness, tenderness, and abrasions. Lotion should be applied to dry feet but never between the toes. After a bath, the patient should gently, not vigorously, pat feet dry to avoid injury.

A student with diabetes tells the school nurse that he is feeling nervous and hungry. The nurse assesses the child and finds he has tachycardia and is diaphoretic with a blood glucose level of 50 mg/dL (2.8 mmol/L). What should the school nurse administer? A) A combination of protein and carbohydrates, such as a small cup of yogurt B) Two teaspoons of sugar dissolved in a cup of apple juice C) Half of a cup of juice, followed by cheese and crackers D) Half a sandwich with a protein-based filling

Ans: C Feedback: Initial treatment for hypoglycemia is 15 g concentrated carbohydrate, such as two or three glucose tablets, 1 tube glucose gel, or 0.5 cup juice. After initial treatment, the nurse should follow with a snack including starch and protein, such as cheese and crackers, milk and crackers, or half of a sandwich. It is unnecessary to add sugar to juice, even it if is labeled as unsweetened juice, because the fruit sugar in juice contains enough simple carbohydrate to raise the blood glucose level and additional sugar may result in a sharp rise in blood sugar that will last for several hours.

A diabetes educator is teaching a patient about type 2 diabetes. The educator recognizes that the patient understands the primary treatment for type 2 diabetes when the patient states what? A) "I read that a pancreas transplant will provide a cure for my diabetes." B) "I will take my oral antidiabetic agents when my morning blood sugar is high." C) "I will make sure to follow the weight loss plan designed by the dietitian." D) "I will make sure I call the diabetes educator when I have questions about my insulin."

Ans: C Feedback: Insulin resistance is associated with obesity; thus the primary treatment of type 2 diabetes is weight loss. Oral antidiabetic agents may be added if diet and exercise are not successful in controlling blood glucose levels. If maximum doses of a single category of oral agents fail to reduce glucose levels to satisfactory levels, additional oral agents may be used. Some patients may require insulin on an ongoing basis or on a temporary basis during times of acute psychological stress, but it is not the central component of type 2 treatment. Pancreas transplantation is associated with type 1 diabetes.

A patient with type 1 diabetes has told the nurse that his most recent urine test for ketones was positive. What is the nurse's most plausible conclusion based on this assessment finding? A) The patient should withhold his next scheduled dose of insulin. B) The patient should promptly eat some protein and carbohydrates. C) The patient's insulin levels are inadequate. D) The patient would benefit from a dose of metformin (Glucophage).

Ans: C Feedback: Ketones in the urine signal that there is a deficiency of insulin and that control of type 1 diabetes is deteriorating. Withholding insulin or eating food would exacerbate the patient's ketonuria. Metformin will not cause short-term resolution of hyperglycemia.

A physician has explained to a patient that he has developed diabetic neuropathy in his right foot. Later that day, the patient asks the nurse what causes diabetic neuropathy. What would be the nurse's best response? A) "Research has shown that diabetic neuropathy is caused by fluctuations in blood sugar that have gone on for years." B) "The cause is not known for sure but it is thought to have something to do with ketoacidosis." C) "The cause is not known for sure but it is thought to involve elevated blood glucose levels over a period of years." D) "Research has shown that diabetic neuropathy is caused by a combination of elevated glucose levels and elevated ketone levels."

Ans: C Feedback: The etiology of neuropathy may involve elevated blood glucose levels over a period of years. High blood sugars (rather than fluctuations or variations in blood sugars) are thought to be responsible. Ketones and ketoacidosis are not direct causes of neuropathies.

Which of the following patients with type 1 diabetes is most likely to experience adequate glucose control? A) A patient who skips breakfast when his glucose reading is greater than 220 mg/dL B) A patient who never deviates from her prescribed dose of insulin C) A patient who adheres closely to a meal plan and meal schedule D) A patient who eliminates carbohydrates from his daily intake

Ans: C Feedback: The therapeutic goal for diabetes management is to achieve normal blood glucose levels without hypoglycemia. Therefore, diabetes management involves constant assessment and modification of the treatment plan by health professionals and daily adjustments in therapy (possibly including insulin) by patients. For patients who require insulin to help control blood glucose levels, maintaining consistency in the amount of calories and carbohydrates ingested at meals is essential. In addition, consistency in the approximate time intervals between meals, and the snacks, help maintain overall glucose control. Skipping meals is never advisable for person with type 1 diabetes.

A school nurse is teaching a group of high school students about risk factors for diabetes. Which of the following actions has the greatest potential to reduce an individual's risk for developing diabetes? A) Have blood glucose levels checked annually. B) Stop using tobacco in any form. C) Undergo eye examinations regularly. D) Lose weight, if obese.

Ans: D Feedback: Obesity is a major modifiable risk factor for diabetes. Smoking is not a direct risk factor for the disease. Eye examinations are necessary for persons who have been diagnosed with diabetes, but they do not screen for the disease or prevent it. Similarly, blood glucose checks do not prevent the diabetes.

A nurse is assessing a patient who has diabetes for the presence of peripheral neuropathy. The nurse should question the patient about what sign or symptom that would suggest the possible development of peripheral neuropathy? A) Persistently cold feet B) Pain that does not respond to analgesia C) Acute pain, unrelieved by rest D) The presence of a tingling sensation

Ans: D Feedback: Although approximately half of patients with diabetic neuropathy do not have symptoms, initial symptoms may include paresthesias (prickling, tingling, or heightened sensation) and burning sensations (especially at night). Cold and intense pain are atypical early signs of this complication.

A patient has received a diagnosis of type 2 diabetes. The diabetes nurse has made contact with the patient and will implement a program of health education. What is the nurse's priority action? A) Ensure that the patient understands the basic pathophysiology of diabetes. B) Identify the patient's body mass index. C) Teach the patient "survival skills" for diabetes. D) Assess the patient's readiness to learn.

Ans: D Feedback: Before initiating diabetes education, the nurse assesses the patient's (and family's) readiness to learn. This must precede other physiologic assessments (such as BMI) and providing health education.

A diabetes nurse educator is presenting the American Diabetes Association (ADA) recommendations for levels of caloric intake. What do the ADA's recommendations include? A) 10% of calories from carbohydrates, 50% from fat, and the remaining 40% from protein B) 10% to 20% of calories from carbohydrates, 20% to 30% from fat, and the remaining 50% to 60% from protein C) 20% to 30% of calories from carbohydrates, 50% to 60% from fat, and the remaining 10% to 20% from protein D) 50% to 60% of calories from carbohydrates, 20% to 30% from fat, and the remaining 10% to 20% from protein

Ans: D Feedback: Currently, the ADA and the Academy of Nutrition and Dietetics (formerly the American Dietetic Association) recommend that for all levels of caloric intake, 50% to 60% of calories come from carbohydrates, 20% to 30% from fat, and the remaining 10% to 20% from protein.

What is the most significant factor in the development of clinical symptoms associated with BPH? A. Size of the prostate B. Location of the enlargement C. Age of the patient D. Length of the urethra

Answer: B There is no direct relationship between the size of the prostate and degree of obstruction. The location of the enlargement significantly affects development of obstructive symptoms. For example, it is possible for mild hyperplasia to cause severe obstruction, and it is possible for extreme hyperplasia to cause few obstructive symptoms.

A medical nurse is aware of the need to screen specific patients for their risk of hyperglycemic hyperosmolar syndrome (HHS). In what patient population does hyperosmolar nonketotic syndrome most often occur? A) Patients who are obese and who have no known history of diabetes B) Patients with type 1 diabetes and poor dietary control C) Adolescents with type 2 diabetes and sporadic use of antihyperglycemics D) Middle-aged or older people with either type 2 diabetes or no known history of diabetes

Ans: D Feedback: HHS occurs most often in older people (50 to 70 years of age) who have no known history of diabetes or who have type 2 diabetes.

A nurse is teaching basic "survival skills" to a patient newly diagnosed with type 1 diabetes. What topic should the nurse address? A) Signs and symptoms of diabetic nephropathy B) Management of diabetic ketoacidosis C) Effects of surgery and pregnancy on blood sugar levels D) Recognition of hypoglycemia and hyperglycemia

Ans: D Feedback: It is imperative that newly diagnosed patients know the signs and symptoms and management of hypo- and hyperglycemia. The other listed topics are valid points for education, but are not components of the patient's immediate "survival skills" following a new diagnosis.

A patient has just been prescribed furosemide (Lasix). After reviewing the patient's medication history, what drug would cause the nurse concern when taken with furosemide (Lasix)? A) Acetaminophen B) Ferrous sulfate (Feosol) C) Naproxen sodium (Naprosyn) D) Ampicillin

Ans: D Feedback: Metformin has the potential to be nephrotoxic; consequently, the nurse should monitor the patient's renal function. This drug does not typically affect patients' neutrophils, liver function, or cognition.

An older adult patient with type 2 diabetes is brought to the emergency department by his daughter. The patient is found to have a blood glucose level of 623 mg/dL. The patient's daughter reports that the patient recently had a gastrointestinal virus and has been confused for the last 3 hours. The diagnosis of hyperglycemic hyperosmolar syndrome (HHS) is made. What nursing action would be a priority? A) Administration of antihypertensive medications B) Administering sodium bicarbonate intravenously C) Reversing acidosis by administering insulin D) Fluid and electrolyte replacement

Ans: D Feedback: The overall approach to HHS includes fluid replacement, correction of electrolyte imbalances, and insulin administration. Antihypertensive medications are not indicated, as hypotension generally accompanies HHS due to dehydration. Sodium bicarbonate is not administered to patients with HHS, as their plasma bicarbonate level is usually normal. Insulin administration plays a less important role in the treatment of HHS because it is not needed for reversal of acidosis, as in diabetic ketoacidosis (DKA).

A newly admitted patient with type 1 diabetes asks the nurse what caused her diabetes. When the nurse is explaining to the patient the etiology of type 1 diabetes, what process should the nurse describe? A) "The tissues in your body are resistant to the action of insulin, making the glucose levels in your blood increase." B) "Damage to your pancreas causes an increase in the amount of glucose that it releases, and there is not enough insulin to control it." C) "The amount of glucose that your body makes overwhelms your pancreas and decreases your production of insulin." D) "Destruction of special cells in the pancreas causes a decrease in insulin production. Glucose levels rise because insulin normally breaks it down."

Ans: D Feedback: Type 1 diabetes is characterized by the destruction of pancreatic beta cells, resulting in decreased insulin production, unchecked glucose production by the liver, and fasting hyperglycemia. Also, glucose derived from food cannot be stored in the liver and remains circulating in the blood, which leads to postprandial hyperglycemia. Type 2 diabetes involves insulin resistance and impaired insulin secretion. The body does not "make" glucose.

A patient with type 2 diabetes has been managing his blood glucose levels using diet and metformin (Glucophage). Following an ordered increase in the patient's daily dose of metformin, the nurse should prioritize which of the following assessments? A) Monitoring the patient's neutrophil levels B) Assessing the patient for signs of impaired liver function C) Monitoring the patient's level of consciousness and behavior D) Reviewing the patient's creatinine and BUN levels

Ans: D) To avoid rebound edema

A complication of prostatitis is A. prostatic cancer. B. benign prostatic hyperplasia. C. acute urinary retention. D. hydrocele.

Answer: C Acute urinary retention can develop in acute prostatitis, and it requires bladder drainage with suprapubic catheterization. Passage of a catheter through an inflamed urethra is contraindicated in acute prostatitis.

Six weeks after a vasectomy, the patient has a sperm-free semen analysis result. What should the couple conclude? A. The patient is sterile. B. One additional semen free test is required. C. Alternative contraception is still required. D. Intercourse should be avoided for 2 more weeks.

Answer: A After vasectomy, the patient should not notice any difference in the look or feel of the ejaculate because its major component is seminal and prostatic fluid. The patient should use an alternative form of contraception until semen examination reveals no sperm. This usually requires at least 10 ejaculations or 6 weeks to evacuate sperm distal to the surgical site.

Where is the urethra opening located in cases of hypospadias? A. On the ventral surface of the penis B. On the dorsal surface of the penis C. At the upper portion of the testicular vault D. Within a bladder extrophy

Answer: A Hypospadias is a urologic abnormality in which the urethral opening is located on the ventral surface of the penis anywhere from the corona to the perineum. Hormonal influences in utero, environmental factors, and genetic factors are possible causes.

Which instructions should you include in the discharge instructions for a patient with epididymitis? A. Refrain from sexual intercourse. B. Continue to participate in liberal exercise. C. Take frequent sitz baths. D. Eat a low-salt diet.

Answer: A Patients with epididymitis should be encouraged to refrain from sexual intercourse during the acute phase. Conservative treatment consists of bed rest with elevation of the scrotum, use of ice packs, and analgesics. Ambulation places the scrotum in a dependent position and increases pain. Salt does not affect epididymitis recovery.

During the bath of a male patient, the nursing assistive personnel reports to you that the patient's foreskin cannot be retracted over the head of the penis. You recognize this condition as A. phimosis. B. prostatitis. C. priapism. D. epispadias.

Answer: A Phimosis is tightness or constriction of the foreskin around the head of the penis, which makes retraction difficult. Paraphimosis is tightness of the foreskin resulting in the inability to pull it forward from a retracted position and preventing normal return over the glans. Prostatitis is inflammation of the prostate, and epispadias occurs when the urethral opening is on the dorsal surface of the penis.

What is an erection lasting longer than 6 hours called? A. Priapism B. Peyronie's disease C. Hydrocele D. Hypospadias

Answer: A Priapism is a painful erection lasting longer than 6 hours. It is caused by an obstruction of the venous outflow in the penis. The condition may constitute a medical emergency. Causes of priapism include thrombosis of the corpus cavernosal veins, leukemia, sickle cell anemia, diabetes mellitus, degenerative lesions of the spine, neoplasms of the brain or spinal cord, vasoactive medications injected into the corpus cavernosa, and medications (e.g., sildenafil, cocaine, trazodone).

What is the primary purpose of a three-way urinary catheter after a transurethral resection of the prostate (TURP)? A. Promote hemostasis and drainage of clots B. Relieve bladder spasms C. Reduce edema D. Increase bladder tone

Answer: A A large three-way indwelling catheter with a 30-mL balloon is inserted into the bladder after the procedure to provide hemostasis and to facilitate urinary drainage. The bladder is irrigated, either continuously or intermittently, usually for the first 24 hours to prevent obstruction from mucus and blood clots.

A patient scheduled for a prostatectomy for prostate cancer expresses the fear that he will have erectile dysfunction. In responding to the patient, you should keep in mind that A. erectile dysfunction can occur even with a nerve-sparing procedure. B. retrograde ejaculation affects sexual functioning more frequently than erectile dysfunction. C. the most common complication of this surgery is postoperative bowel incontinence. D. preoperative sexual functioning is the most important factor in determining postoperative erectile dysfunction.

Answer: A A major complication after a prostatectomy (even with nerve-sparing procedures) is erectile dysfunction

An elderly male patient is experiencing difficulty in initiating voiding and a feeling of incomplete bladder emptying. What causes these symptoms in benign prostatic hyperplasia (BPH)? A. Obstruction of the urethra B. Untreated chronic prostatitis C. Decreased bladder compliance D. Excessive secretion of testosterone

Answer: A BPH is a benign enlargement of the prostate gland. The enlargement of the prostate gradually compresses the urethra, eventually causing partial or complete obstruction. Compression of the urethra ultimately leads to the development of clinical symptoms.

After you perform teaching regarding medication therapy for erectile dysfunction (ED), you know the teaching was successful when the patient states, A. "I will take the drug once each day about an hour before sexual activity." B. "I will take the drug two times each day but not more than three." C. "I can take the drug in the morning, and I will be able to have an erection anytime throughout the day." D. "There are no precautions to take while I am on this medication."

Answer: A Medication is taken orally about 1 hour before sexual activity, but not more than once each day. These drugs have been found to be generally safe and effective for the treatment of most types of ED.

During examination of a 67-year-old man, the nurse notes bilateral enlargement of the breasts. The nurse's first action should be to a. palpate the breasts for the presence of any discrete lumps. b. explain that this is a temporary condition caused by hormonal changes. c. refer the patient for mammography and biopsy of the breast tissue. d. teach the patient about dietary changes to reduce the breast size.

Answer: A Rationale: If discrete, circumscribed lumps are present, the patient should be referred for further testing to determine whether breast cancer is present. Gynecomastia is usually a temporary change, but it can be caused by breast cancer. Mammography and biopsy will not be needed unless lumps are present in the breast tissue. Dietary changes will not affect the condition. Cognitive Level: Application Text Reference: p. 1348 Nursing Process: Implementation NCLEX: Physiological Integrity

During examination of a 67-year-old man, the nurse notes bilateral enlargement of the breasts. The nurse's first action should be to a. palpate the breasts for the presence of any discrete lumps. b. explain that this is a temporary condition caused by hormonal changes. c. refer the patient for mammography and biopsy of the breast tissue. d. teach the patient about dietary changes to reduce the breast size.

Answer: A Rationale: If discrete, circumscribed lumps are present, the patient should be referred for further testing to determine whether breast cancer is present. Gynecomastia is usually a temporary change, but it can be caused by breast cancer. Mammography and biopsy will not be needed unless lumps are present in the breast tissue. Dietary changes will not affect the condition. Pg. 1242

Which of these nursing interventions for the patient who has had right-sided breast-conservation surgery and an axillary lymph node dissection is appropriate to assign to an LPN/LVN? a. Administering an analgesic 30 minutes before the scheduled arm exercises b. Teaching the patient how to avoid injury to the right arm c. Assessment of the patient's range of motion for the right arm d. Evaluation of the patient's understanding of discharge instructions about drain care

Answer: A Rationale: LPN/LVN education and scope of practice include administration and evaluation of the effects of analgesics. Assessment, teaching, and evaluation of a patient's understanding of instructions are more complex tasks that are more appropriate to RN-level education and scope of practice. Cognitive Level: Application Text Reference: pp. 1356-1361 Nursing Process: Implementation NCLEX: Safe and Effective Care Environment

A 38-year-old woman is scheduled for a breast-conservation therapy with a lumpectomy. As the nurse prepares her for surgery, she begins to cry and says, "I just do not know how to handle all of this." An appropriate response to the patient by the nurse is, a. "Would you like to talk about how you are feeling right now?" b. "I can see you are really upset. Would you like to be alone for a while?" c. "The important thing is that the tumor was found and is going to be removed." d. "With this surgery you will have very little change in the appearance of your breast."

Answer: A Rationale: The nurse encourages the patient to express feelings about the diagnosis and surgery. The response beginning, "I can see you are really upset" may indicate that the nurse is uncomfortable being with the patient while she is upset. The response beginning, "The important thing is that the tumor was found" places the nurse's value system above the patient's current concerns. And the response, "With this surgery you will have very little change in the appearance of your breast" does not address all the patient's possible concerns and is not true. Cognitive Level: Application Text Reference: p. 1360 Nursing Process: Implementation NCLEX: Psychosocial Integrity

A 38-year-old woman is scheduled for a breast-conservation therapy with a lumpectomy. As the nurse prepares her for surgery, she begins to cry and says, "I just do not know how to handle all of this." An appropriate response to the patient by the nurse is, a. "Would you like to talk about how you are feeling right now?" b. "I can see you are really upset. Would you like to be alone for a while?" c. "The important thing is that the tumor was found and is going to be removed." d. "With this surgery you will have very little change in the appearance of your breast."

Answer: A Rationale: The nurse encourages the patient to express feelings about the diagnosis and surgery. The response beginning, "I can see you are really upset" may indicate that the nurse is uncomfortable being with the patient while she is upset. The response beginning, "The important thing is that the tumor was found" places the nurse's value system above the patient's current concerns. And the response, "With this surgery you will have very little change in the appearance of your breast" does not address all the patient's possible concerns and is not true. Pg. 1255

After the nurse completes discharge teaching for a patient who has had a left modified radical mastectomy and lymph node dissection, which statement by the patient indicates that no further teaching is needed? a. "I will avoid reaching over the stove with my left hand." b. "I will need to do breast self-examination on my right breast monthly." c. "I will keep my left arm elevated until I go to bed." d. "I will remember to use my right arm and to rest the left one."

Answer: A Rationale: The patient should avoid any activity that might injure the left arm, such as reaching over a burner. Breast self-examination should be done to the right breast and the left mastectomy site. The left arm should be elevated when the patient is lying down also. The left arm should be used to improve range of motion and function. Cognitive Level: Application Text Reference: pp. 1359-1360 Nursing Process: Evaluation NCLEX: Physiological Integrity

After the nurse completes discharge teaching for a patient who has had a left modified radical mastectomy and lymph node dissection, which statement by the patient indicates that no further teaching is needed? a. "I will avoid reaching over the stove with my left hand." b. "I will need to do breast self-examination on my right breast monthly." c. "I will keep my left arm elevated until I go to bed." d. "I will remember to use my right arm and to rest the left one."

Answer: A Rationale: The patient should avoid any activity that might injure the left arm, such as reaching over a burner. Breast self-examination should be done to the right breast and the left mastectomy site. The left arm should be elevated when the patient is lying down also. The left arm should be used to improve range of motion and function. Pg. 1254z

Which ethnic group has the highest incidence of prostate cancer? A. African Americans B. Asians C. Whites D. Hispanics

Answer: A The incidence of prostate cancer worldwide is higher among African Americans than in any other ethnic group. The reasons for the higher rate are unknown.

A patient with benign prostatic hyperplasia is scheduled for TURP. After you assess the patient's knowledge of the procedure and its effects on reproductive function, you determine a need for further teaching when the patient says, A. "It is possible that I'll be sterile after this procedure." B. "I understand that some retrograde ejaculation may occur." C. "I will have a catheter for several days to keep my urinary system open." D. "It is unlikely that I would become impotent from this procedure."

Answer: A The patient will not be sterile; he may experience retrograde ejaculation and some erectile dysfunction. It is unlikely he will become impotent. He will need a catheter.

You notice that the patient's urinary drainage 4 hours after TURP is redder than 1 hour ago. What is your priority intervention? A. To increase the rate of bladder irrigation B. To manually irrigate the urinary catheter C. To notify the physician D. To obtain vital signs

Answer: A With CBI, irrigating solution is continuously infused and drained from the bladder. The rate of infusion is based on the color of drainage. Ideally, urine drainage should be light pink without clots. Continuously monitor the inflow and outflow of the irrigant.

A patient asks you, "How can I decrease my risk of prostate cancer?" You teach the patient to avoid which foods (select all that apply)? A. Red meat B. High-fat dairy products C. Fruits D. Vegetables E. Chicken

Answer: A, B Dietary factors may be associated with prostate cancer. A diet high in red meat and high-fat dairy products, along with a low intake of vegetables and fruits, may increase the risk of prostate cancer.

Which factors place a patient at high risk for prostate cancer (select all that apply)? A. Age older than 65 years B. Asian or Native American ethnicity C. Personal history of BPH D. Brother diagnosed and treated for prostate cancer E. History of undescended testicle and testicular cancer

Answer: A,D Age, ethnicity, and family history are known risk factors for prostate cancer. The incidence of prostate cancer rises markedly after age 50, and more than 66% of men diagnosed are older than 65 years. The incidence of prostate cancer worldwide is higher among African Americans than in any other ethnic group. A family history of prostate cancer, especially first-degree relatives (fathers, brothers), is associated with an increased risk.

Which childhood disease is often associated with orchitis? A. Measles B. Mumps C. Strep throat D. Chickenpox

Answer: B Orchitis refers to an acute inflammation of the testis. In orchitis, the testis is painful, tender, and swollen. It usually occurs after an episode of bacterial or viral infection, such as mumps. The other disorders are not associated with mumps.

Which fact in the patient's history could be related to the presence of testicular cancer? A. Epispadias B. Cryptorchidism C. Hernia repair D. Uncircumcised penis

Answer: B The incidence of testicular cancer is four times higher among white men (especially those of Scandinavian descent) than African American males. It occurs more commonly in the right testicle than the left. Testicular tumors are also more common in men who have had undescended testes (cryptorchidism) or have a family history of testicular cancer or anomalies. Epispadias and lack of circumcision are not related to testicular cancer. Hernia repair is related to future hernias, but not testicular cancer.

You have provided teaching to the male patient regarding a vasectomy procedure. Which statement by the patient indicates the teaching has been successful? A. "I will need to not eat or drink anything for 24 hours before the procedure." B. "The physician will use local anesthetic, and I will be awake." C. "I will be able to have unprotected intercourse in 3 weeks." D. "I acknowledge there is a risk of impotence."

Answer: B The procedure requires only 15 to 30 minutes and is usually performed with the patient under local anesthesia on an outpatient basis. Vasectomy is considered a permanent form of sterilization but does not affect the ability to achieve an erection. After vasectomy, the patient should not notice any difference in the look or feel of the ejaculate because its major component is seminal and prostatic fluid. The patient should use an alternative form of contraception until semen examination reveals no sperm. This usually requires at least 10 ejaculations or 6 weeks to evacuate sperm distal to the surgical site.

A 73-year-old man admitted for total knee replacement states during the health history interview that he has no problems with urinary elimination except that the "stream is less than it used to be." You would give the patient anticipatory guidance that which condition is likely to be developing? A. A tumor of the prostate B. Benign prostatic hyperplasia C. Bladder atony because of age D. Age-related altered innervation of the bladder

Answer: B BPH is an enlarged prostate gland caused by an increased number of epithelial cells and stromal tissue. It occurs in about 50% of men older than 50 years and 80% of men older than 80 years.

You should encourage a patient with bacterial prostatitis to A. remain on bed rest until antibiotic therapy has been in place for 72 hours. B. significantly increase fluid intake of noncaffeinated beverages. C. empty the bladder every hour with a straight catheter. D. weigh daily before the first voiding of the morning.

Answer: B Because the prostate can serve as a source of bacteria, fluid intake should be kept at a high level for all patients experiencing prostatitis. Nursing interventions are aimed at encouraging the patient to drink plenty of fluids. This is especially important for those with acute bacterial prostatitis because of the increased fluid needs associated with fever and infection. Bed rest is not indicated, and activity should be encouraged. The bladder should be emptied, although not with a catheter, and it is not necessary to do so every hour. Weight gain should be reported, but the patient does not need to weigh before voiding.

You are teaching the patient with BPH about interventions that can assist in alleviating symptoms. Which behavior in the patient indicates successful teaching? A. The patient increases use of decongestants. B. The patient decreases intake of caffeinated beverages and artificial sweeteners. C. The patient increases activities such as walking. D. The patient voids every 30 minutes.

Answer: B In some patients who have symptoms that appear and then disappear, a conservative treatment approach has value. Dietary changes (decreasing intake of caffeine and artificial sweeteners, limiting spicy or acidic foods), avoiding medications such as decongestants and anticholinergics, and restricting evening fluid intake may result in improvement of symptoms. A timed voiding schedule may reduce or eliminate symptoms, negating the need for further intervention, but 30 minutes is too frequent.

When assessing a patient for breast cancer risk, the nurse considers that the patient has a significant family history of breast cancer if she has a a. cousin who was diagnosed with breast cancer at age 38. b. mother who was diagnosed with breast cancer at age 42. c. sister who died from ovarian cancer at age 56. d. grandmother who died from breast cancer at age 72.

Answer: B Rationale: A significant family history of breast cancer means that the patient has a first-degree relative who developed breast cancer, especially if the relative was premenopausal. Cognitive Level: Application Text Reference: p. 1348 Nursing Process: Assessment NCLEX: Health Promotion and Maintenance

When assessing a patient for breast cancer risk, the nurse considers that the patient has a significant family history of breast cancer if she has a a. cousin who was diagnosed with breast cancer at age 38. b. mother who was diagnosed with breast cancer at age 42. c. sister who died from ovarian cancer at age 56. d. grandmother who died from breast cancer at age 72.

Answer: B Rationale: A significant family history of breast cancer means that the patient has a first-degree relative who developed breast cancer, especially if the relative was premenopausal. Pg. 1243

A 20-year-old student comes to the student health center after discovering a small painless lump in her right breast. She is worried that she might have cancer because her mother had cervical cancer. The nurse's response to the patient is based on the knowledge that the most likely cause of the breast lump is a. fibrocystic complex. b. fibroadenoma. c. breast abscess. d. adenocarcinoma.

Answer: B Rationale: Fibroadenoma is the most frequent cause of breast lumps in women under 25 years of age. Fibrocystic changes occur most frequently in women ages 35 to 50. Breast abscess is associated with pain and other systemic symptoms. Breast cancer is uncommon in women younger than 25. Pg. 1242

A patient with a small breast lump is advised to have a fine needle aspiration (FNA) biopsy. The nurse explains that an advantage to this procedure is that a. only a small incision is necessary, resulting in minimal breast pain and scarring. b. if the specimen is positive for malignancy, the patient can be told at the visit. c. if the specimen is negative for malignancy, the patient's fears of cancer can be put to rest. d. FNA is guided by a mammogram, ensuring that cells are taken from the lesion.

Answer: B Rationale: An FNA should only be done when an experienced cytologist is available to read the specimen immediately. If the specimen is positive for malignancy, the patient can be given this information immediately. No incision is needed. If the specimen is negative for malignancy, the patient will require biopsy of the lump. FNA is not guided by mammography. Cognitive Level: Application Text Reference: p. 1345 Nursing Process: Planning NCLEX: Physiological Integrity

A patient with a small breast lump is advised to have a fine needle aspiration (FNA) biopsy. The nurse explains that an advantage to this procedure is that a. only a small incision is necessary, resulting in minimal breast pain and scarring. b. if the specimen is positive for malignancy, the patient can be told at the visit. c. if the specimen is negative for malignancy, the patient's fears of cancer can be put to rest. d. FNA is guided by a mammogram, ensuring that cells are taken from the lesion.

Answer: B Rationale: An FNA should only be done when an experienced cytologist is available to read the specimen immediately. If the specimen is positive for malignancy, the patient can be given this information immediately. No incision is needed. If the specimen is negative for malignancy, the patient will require biopsy of the lump. FNA is not guided by mammography. Pg. 1240

A 51-year-old woman at menopause is considering the use of hormone replacement therapy (HRT) but is concerned about the risk of breast cancer. When discussing this issue with the patient, the nurse explains that a. HRT does not appear to increase the risk for breast cancer unless there are other risk factors. b. she and her health care provider must weigh the benefits of HRT against the possible risks of breast cancer. c. HRT is a safe therapy for menopausal symptoms if there is no family history of BRCA genes. d. alternative therapies with herbs and natural drugs are as effective as estrogen in relieving the symptoms of menopause.

Answer: B Rationale: Because HRT has been linked to increased risk for breast cancer, the patient and provider must determine whether or not to use HRT. Breast cancer incidence is increased in women using HRT, independent of other risk factors. HRT increase the risk for non-BRCA-associated cancer as well as for BRCA-related cancers. Alternative therapies can be used but are not consistent in relieving menopausal symptoms. Cognitive Level: Application Text Reference: p. 1348 Nursing Process: Implementation NCLEX: Physiological Integrity

A 51-year-old woman at menopause is considering the use of hormone replacement therapy (HRT) but is concerned about the risk of breast cancer. When discussing this issue with the patient, the nurse explains that a. HRT does not appear to increase the risk for breast cancer unless there are other risk factors. b. she and her health care provider must weigh the benefits of HRT against the possible risks of breast cancer. c. HRT is a safe therapy for menopausal symptoms if there is no family history of BRCA genes. d. alternative therapies with herbs and natural drugs are as effective as estrogen in relieving the symptoms of menopause.

Answer: B Rationale: Because HRT has been linked to increased risk for breast cancer, the patient and provider must determine whether or not to use HRT. Breast cancer incidence is increased in women using HRT, independent of other risk factors. HRT increase the risk for non-BRCA-associated cancer as well as for BRCA-related cancers. Alternative therapies can be used but are not consistent in relieving menopausal symptoms. Pg. 1251

At a routine health examination, a woman whose mother had breast cancer asks the nurse about the genetic basis of breast cancer and the genes involved. The nurse explains that a. her risk of inheriting BRCA gene mutations is small unless her mother had both ovarian and breast cancer. b. changes in BRCA genes that normally suppress cancer growth can be passed to offspring, increasing the risk for breast cancer. c. because her mother had breast cancer, she has inherited a 50% to 85% chance of developing breast cancer from mutated genes. d. genetic mutations increase cancer risk only in combination with other risk factors such as obesity.

Answer: B Rationale: Family history is a risk factor for breast cancer, and the nurse should discuss testing for BRCA genes with the patient. Although the BRCA gene is associated with increased risk for breast and ovarian cancer, the patient may be at risk if her mother had either one. About 5% to 10% of patients with breast cancer may have a genetic abnormality that contributes to breast cancer development. Risk factors are cumulative, but a family history alone will increase breast cancer risk. Cognitive Level: Application Text Reference: p. 1349 Nursing Process: Implementation NCLEX: Physiological Integrity

At a routine health examination, a woman whose mother had breast cancer asks the nurse about the genetic basis of breast cancer and the genes involved. The nurse explains that a. her risk of inheriting BRCA gene mutations is small unless her mother had both ovarian and breast cancer. b. changes in BRCA genes that normally suppress cancer growth can be passed to offspring, increasing the risk for breast cancer. c. because her mother had breast cancer, she has inherited a 50% to 85% chance of developing breast cancer from mutated genes. d. genetic mutations increase cancer risk only in combination with other risk factors such as obesity.

Answer: B Rationale: Family history is a risk factor for breast cancer, and the nurse should discuss testing for BRCA genes with the patient. Although the BRCA gene is associated with increased risk for breast and ovarian cancer, the patient may be at risk if her mother had either one. About 5% to 10% of patients with breast cancer may have a genetic abnormality that contributes to breast cancer development. Risk factors are cumulative, but a family history alone will increase breast cancer risk. Pg. 1243-1244

A 20-year-old student comes to the student health center after discovering a small painless lump in her right breast. She is worried that she might have cancer because her mother had cervical cancer. The nurse's response to the patient is based on the knowledge that the most likely cause of the breast lump is a. fibrocystic complex. b. fibroadenoma. c. breast abscess. d. adenocarcinoma.

Answer: B Rationale: Fibroadenoma is the most frequent cause of breast lumps in women under 25 years of age. Fibrocystic changes occur most frequently in women ages 35 to 50. Breast abscess is associated with pain and other systemic symptoms. Breast cancer is uncommon in women younger than 25. Cognitive Level: Application Text Reference: p. 1347 Nursing Process: Implementation NCLEX: Physiological Integrity

When teaching a 22-year-old patient about breast self-examination (BSE), the nurse will instruct the patient that a. BSE will reduce the risk of dying from breast cancer. b. performing BSE right after the menstrual period will improve comfort. c. BSE should be done daily while taking a bath or shower. d. annual mammograms should be scheduled in addition to BSE.

Answer: B Rationale: Performing BSE at the end of the menstrual period will reduce the breast tenderness associated with the procedure. The evidence is not clear that BSE reduces breast cancer mortality. BSE should be done monthly. Annual mammograms are not routinely scheduled for women under age 40. Cognitive Level: Application Text Reference: p. 1344 Nursing Process: Implementation NCLEX: Health Promotion and Maintenance

When teaching a 22-year-old patient about breast self-examination (BSE), the nurse will instruct the patient that a. BSE will reduce the risk of dying from breast cancer. b. performing BSE right after the menstrual period will improve comfort. c. BSE should be done daily while taking a bath or shower. d. annual mammograms should be scheduled in addition to BSE.

Answer: B Rationale: Performing BSE at the end of the menstrual period will reduce the breast tenderness associated with the procedure. The evidence is not clear that BSE reduces breast cancer mortality. BSE should be done monthly. Annual mammograms are not routinely scheduled for women under age 40. p. 1239

A patient with a breast biopsy positive for cancer is to undergo lymphatic mapping and sentinel lymph node dissection (SLND). The nurse explains that this procedure a. can identify specific lymph nodes that have malignant cells, so only involved nodes need to be excised. b. reduces the need for extensive lymph node dissection for pathologic examination. c. eliminates the need for excision of more than one lymph node for staging of breast cancer. d. will confirm the absence of tumor spread if the sentinel lymph node is negative for malignant changes.

Answer: B Rationale: The SLND may eliminate further lymph node dissection if the initial nodes are negative for malignancy. The procedure identifies which lymph nodes drain first from the tumor site, but not which ones are malignant. Several lymph nodes may be dissected for pathologic examination. Tumor may have distant metastases even when no malignancies are found in the lymph nodes. Cognitive Level: Comprehension Text Reference: p. 1351 Nursing Process: Implementation NCLEX: Physiological Integrity

A patient with a breast biopsy positive for cancer is to undergo lymphatic mapping and sentinel lymph node dissection (SLND). The nurse explains that this procedure a. can identify specific lymph nodes that have malignant cells, so only involved nodes need to be excised. b. reduces the need for extensive lymph node dissection for pathologic examination. c. eliminates the need for excision of more than one lymph node for staging of breast cancer. d. will confirm the absence of tumor spread if the sentinel lymph node is negative for malignant changes.

Answer: B Rationale: The SLND may eliminate further lymph node dissection if the initial nodes are negative for malignancy. The procedure identifies which lymph nodes drain first from the tumor site, but not which ones are malignant. Several lymph nodes may be dissected for pathologic examination. Tumor may have distant metastases even when no malignancies are found in the lymph nodes. Pg. 1245-1246

A woman with a positive biopsy for breast cancer is considering whether to have a modified radical mastectomy or breast conservation surgery (lumpectomy) with radiation therapy. Which information should the nurse provide? a. The postoperative survival rate for each is about the same, but there is a decreased rate of cancer recurrence after mastectomy. b. The lumpectomy and radiation will preserve the breast, but this method can cause changes in breast sensitivity. c. The hair loss associated with post-lumpectomy chemotherapy is not acceptable to some patients. d. The treatment period for the mastectomy is shorter, and breast reconstruction can provide a normal-appearing breast.

Answer: B Rationale: The impact on breast function and appearance is less with lumpectomy and radiation, but there is some effect on breast sensitivity. The rate of cancer recurrence is the same for the two procedures. Chemotherapy may be used after either lumpectomy or mastectomy, but it is not always needed. The treatment period is shorter after mastectomy, but breast reconstruction does not provide a normal-appearing breast. Cognitive Level: Application Text Reference: pp. 1352-1353 Nursing Process: Implementation NCLEX: Physiological Integrity

A woman with a positive biopsy for breast cancer is considering whether to have a modified radical mastectomy or breast conservation surgery (lumpectomy) with radiation therapy. Which information should the nurse provide? a. The postoperative survival rate for each is about the same, but there is a decreased rate of cancer recurrence after mastectomy. b. The lumpectomy and radiation will preserve the breast, but this method can cause changes in breast sensitivity. c. The hair loss associated with post-lumpectomy chemotherapy is not acceptable to some patients. d. The treatment period for the mastectomy is shorter, and breast reconstruction can provide a normal-appearing breast.

Answer: B Rationale: The impact on breast function and appearance is less with lumpectomy and radiation, but there is some effect on breast sensitivity. The rate of cancer recurrence is the same for the two procedures. Chemotherapy may be used after either lumpectomy or mastectomy, but it is not always needed. The treatment period is shorter after mastectomy, but breast reconstruction does not provide a normal-appearing breast. Pg. 1248

A patient has a permanent breast implant inserted in the outpatient surgery area. Which instructions will the nurse include in the discharge teaching? a. Resume normal activities 2 to 3 days after the mammoplasty. b. Check wound drains for excessive blood or any foul odor. c. Wear a loose-fitting bra to decrease irritation of the sutures. d. Take aspirin every 4 hours to reduce inflammation.

Answer: B Rationale: The patient should be taught drain care because the drains will be in place for 2 or 3 days after surgery. Normal activities can be resumed after 2 to 3 weeks. A bra that provides good support is typically ordered. Aspirin will decrease coagulation and is typically not given after surgery. Cognitive Level: Comprehension Text Reference: p. 1362 Nursing Process: Implementation NCLEX: Physiological Integrity

Which best indicates that treatment for cancer of the prostate is effective? A. Increase in urinary stream B. Decrease of PSA to 2 ng/mL C. Decreased blood in the urine D. White blood cell (WBC) count of 10,000/μL

Answer: B The PSA value is used to detect prostate cancer and to monitor the success of treatment. When treatment has been successful in removing prostate cancer, PSA levels should decrease and reach normal levels (less than 4 ng/mL). The regular measurement of PSA levels after treatment is important to evaluate the effectiveness of treatment and possible recurrence of prostate cancer.

When providing patient teaching about medication therapy for BPH with 5α-reductase inhibitors such as finasteride (Proscar), what information should you include? A. Ninety percent of patients show improvement with the drug. B. The drug can be taken periodically as symptoms occur. C. Women who are pregnant should not handle the drug. D. Effects are seen in 1 week.

Answer: C Although more than 50% of men who are treated with the drug show symptom improvement, it takes about 6 months to be effective. The drug must be taken on a continuous basis to maintain therapeutic results. Women who may be or are pregnant should not handle tablets.

To decrease the patient's discomfort about care involving his reproductive organs, you should A. relate his sexual concerns to his sexual partner. B. arrange to have male nurses care for the patient. C. maintain a nonjudgmental attitude toward his sexual practices. D. use technical terminology when discussing reproductive function.

Answer: C Conducting routine health assessments on men places you in a unique position. It provides an opportunity to ask the patient questions pertaining to general health and to sexual health and function. Given the opportunity, men are less hesitant to answer these questions when they know that someone cares and can provide them with answers. You must remain nonjudgmental about sexual practices.

The patient with epididymitis asks you when he can resume his exercise routine. You should respond with which piece of information? A. When there is no evidence of urethral discharge B. After 1 week of antibiotic therapy C. When scrotal pain has subsided D. As soon as he feels able

Answer: C Most tenderness subsides within 1 week, although swelling may last for weeks or months. One week of antibiotic therapy may not relieve pain and swelling. "When the patient feels able" is a vague response, and the patient should remain on rest until scrotal swelling has subsided. There may be no urethral discharge in epididymitis.

What is the first step in the identification of the cause of male infertility? A. Estrogen levels B. Progesterone levels C. Semen analysis D. Time to ejaculation

Answer: C The first test in an infertility study is a semen analysis. The test determines the sperm concentration, motility, and morphology

What is the most common cause of erectile dysfunction? A. Aging B. Benign prostatic hyperplasia C. Vascular disease D. Psychological distress

Answer: C Common causes of ED include diabetes, vascular disease, side effects from medications, results of surgery (e.g., prostatectomy), trauma, chronic illness, decreased gonadal hormone secretion, stress, difficulty in a relationship, or depression. The most common cause of ED is vascular disease.

A 62-year-old man is seen at the health clinic because he is concerned about a gradual decrease in sexual performance. The nursing history does not identify any specific risk factors related to erectile dysfunction. What is your priority at this time? A. Consult with the health care provider about testing the patient for hypogonadism. B. Refer the patient to a qualified therapist to explore possible psychologic causes of decreased function. C. Explain normal age-related changes in sexual performance to the patient. D. Discuss the variety of aids and devices available to increase sexual performance.

Answer: C Normal physiologic age-related changes are associated with changes in erectile function and may be an underlying cause of ED for some men.

A 62-year-old patient complains to the nurse that mammograms are painful and a source of radiation exposure. She says she does breast self-examination (BSE) monthly and asks whether it is necessary to have an annual mammogram. The nurse's best response to the patient is, a. "If your mammogram was painful, it is especially important that you have it done annually." b. "An ultrasound examination of the breasts, which is not painful or a source of radiation, can be substituted for a mammogram." c. "Because of your age, it is even more important for you to have annual mammograms." d. "Unless you find a lump while examining your breasts, a mammogram every 2 years is recommended after age 60."

Answer: C Rationale: Annual mammograms are recommended for women over age 40 as long as they are in good health. The incidence of breast cancer increases in women over 60. Pain with a mammogram does not indicate any greater risk for breast cancer. Ultrasound may be used in some situations to differentiate cystic breast problems from cancer but is not a substitute for annual mammograms. Cognitive Level: Application Text Reference: p. 1344 Nursing Process: Implementation NCLEX: Health Promotion and Maintenance

A 62-year-old patient complains to the nurse that mammograms are painful and a source of radiation exposure. She says she does breast self-examination (BSE) monthly and asks whether it is necessary to have an annual mammogram. The nurse's best response to the patient is, a. "If your mammogram was painful, it is especially important that you have it done annually." b. "An ultrasound examination of the breasts, which is not painful or a source of radiation, can be substituted for a mammogram." c. "Because of your age, it is even more important for you to have annual mammograms." d. "Unless you find a lump while examining your breasts, a mammogram every 2 years is recommended after age 60."

Answer: C Rationale: Annual mammograms are recommended for women over age 40 as long as they are in good health. The incidence of breast cancer increases in women over 60. Pain with a mammogram does not indicate any greater risk for breast cancer. Ultrasound may be used in some situations to differentiate cystic breast problems from cancer but is not a substitute for annual mammograms. Pg. 1238

A 33-year-old patient tells the nurse that she has fibrocystic breasts but reducing her sodium and caffeine intake and other measures have not made a difference in the fibrocystic condition. An appropriate patient outcome for the patient is a. calls the health care provider if any lumps are painful or tender. b. states the reason for immediate biopsy of new lumps. c. monitors changes in size and tenderness of all lumps in relation to her menstrual cycle. d. has genetic testing for BRCA-1 and BRCA-2 to determine her risk for breast cancer.

Answer: C Rationale: Because fibrocystic breasts may increase in size and tenderness during the premenstrual phase, the patient is taught to monitor for this change and to call if the changes persist after menstruation. Pain and tenderness are typical of fibrocystic breasts, and the patient should not call for these symptoms. New lumps may be need biopsy if they persist after the menstrual period, but the biopsy is not done immediately. The existence of fibrocystic breasts is not associated with the BRCA genes. Cognitive Level: Application Text Reference: pp. 1346-1347 Nursing Process: Planning NCLEX: Health Promotion and Maintenance

A 33-year-old patient tells the nurse that she has fibrocystic breasts but reducing her sodium and caffeine intake and other measures have not made a difference in the fibrocystic condition. An appropriate patient outcome for the patient is a. calls the health care provider if any lumps are painful or tender. b. states the reason for immediate biopsy of new lumps. c. monitors changes in size and tenderness of all lumps in relation to her menstrual cycle. d. has genetic testing for BRCA-1 and BRCA-2 to determine her risk for breast cancer.

Answer: C Rationale: Because fibrocystic breasts may increase in size and tenderness during the premenstrual phase, the patient is taught to monitor for this change and to call if the changes persist after menstruation. Pain and tenderness are typical of fibrocystic breasts, and the patient should not call for these symptoms. New lumps may be need biopsy if they persist after the menstrual period, but the biopsy is not done immediately. The existence of fibrocystic breasts is not associated with the BRCA genes. Pg. 1241-1242

A patient at the clinic who has metastatic breast cancer has a new prescription for trastuzumab (Herceptin). The nurse will plan to a. teach the patient about the need to monitor serum electrolyte levels. b. ask the patient to call the health care provider before using any over-the-counter (OTC) pain relievers. c. instruct the patient to call if she notices ankle swelling. d. have the patient schedule frequent eye examinations.

Answer: C Rationale: Herceptin can lead to ventricular dysfunction, so the patient is taught to self-monitor for symptoms of heart failure. There is no need to monitor serum electrolyte levels. OTC pain relievers do not interact with Herceptin. Changes in visual acuity may occur with tamoxifen, but not with Herceptin. Cognitive Level: Analysis Text Reference: p. 1356 Nursing Process: Implementation NCLEX: Physiological Integrity

A patient at the clinic who has metastatic breast cancer has a new prescription for trastuzumab (Herceptin). The nurse will plan to a. teach the patient about the need to monitor serum electrolyte levels. b. ask the patient to call the health care provider before using any over-the-counter (OTC) pain relievers. c. instruct the patient to call if she notices ankle swelling. d. have the patient schedule frequent eye examinations.

Answer: C Rationale: Herceptin can lead to ventricular dysfunction, so the patient is taught to self-monitor for symptoms of heart failure. There is no need to monitor serum electrolyte levels. OTC pain relievers do not interact with Herceptin. Changes in visual acuity may occur with tamoxifen, but not with Herceptin. Pg. 1252

Following a modified radical mastectomy, a patient tells the nurse the health care provider has recommended a flap procedure for breast reconstruction but that she did not understand how this was done. The nurse explains that the most common procedure, a transverse rectus abdominis musculocutaneous (TRAM) flap, involves a. relocating muscle tissue from the back and using it to form a breast. b. removing a portion of an abdominal muscle to use as breast tissue. c. pulling part of the abdominal muscle up to the breast area through a tunnel in the chest. d. relocating the arteries from the abdominal muscle to improve circulation to the implant.

Answer: C Rationale: In the TRAM flap, part of the rectus abdominis muscle is tunneled to the breast area and molded to form a breast. In the latissimus dorsi musculocutaneous flap, muscle tissue from the back is used to replace breast tissue. The abdominal muscle is not detached but is still attached to the rectus muscle. The arteries are not relocated. Cognitive Level: Application Text Reference: pp. 1362-1363 Nursing Process: Implementation NCLEX: Physiological Integrity

Which statement by a 32-year-old patient newly diagnosed with stage I breast cancer indicates to the nurse that the goals of therapy are being met? a. "I am not sure how my husband will react when I tell him about this cancer." b. "I am ready to die if that is God's plan for me." c. "I need to know all the options before making a decision about treatment." d. "I will do whatever the doctor thinks is best."

Answer: C Rationale: One goal for the patient with breast cancer is active participation in the decision-making process. The response beginning, "I am not sure how my husband will react" indicates that the goal of satisfaction with the support provided by significant others is still unmet. The response, "I am ready to die if that is God's plan for me" suggests that the patient may not be willing to have treatment. The response, "I will do whatever the doctor thinks is best" indicates that the patient is not participating actively in treatment decisions. Cognitive Level: Application Text Reference: p. 1359 Nursing Process: Evaluation NCLEX: Psychosocial Integrity

A patient returns to the surgical unit following a right modified radical mastectomy with dissection of axillary lymph nodes. An appropriate intervention for the nurse to include in implementing postoperative care for the patient includes a. teaching the patient to use the ordered patient-controlled analgesia (PCA) every 10 minutes for the best pain relief. b. insisting that the patient examine the surgical incision when the dressings are removed. c. posting a sign at the bedside warning against blood pressures or venipunctures in the right arm. d. encouraging the patient to obtain a permanent breast prosthesis as soon as she is discharged from the hospital.

Answer: C Rationale: The patient is at risk for lymphedema and infection if blood pressures or venipuncture are done on the right arm. The patient is taught to use the PCA as needed for pain control rather than at a set time. The nurse allows the patient to examine the incision and participate in care when the patient feels ready. Permanent breast prostheses are usually obtained about 6 weeks after surgery. Cognitive Level: Application Text Reference: pp. 1358-1359 Nursing Process: Implementation NCLEX: Physiological Integrity

A patient returns to the surgical unit following a right modified radical mastectomy with dissection of axillary lymph nodes. An appropriate intervention for the nurse to include in implementing postoperative care for the patient includes a. teaching the patient to use the ordered patient-controlled analgesia (PCA) every 10 minutes for the best pain relief. b. insisting that the patient examine the surgical incision when the dressings are removed. c. posting a sign at the bedside warning against blood pressures or venipunctures in the right arm. d. encouraging the patient to obtain a permanent breast prosthesis as soon as she is discharged from the hospital.

Answer: C Rationale: The patient is at risk for lymphedema and infection if blood pressures or venipuncture are done on the right arm. The patient is taught to use the PCA as needed for pain control rather than at a set time. The nurse allows the patient to examine the incision and participate in care when the patient feels ready. Permanent breast prostheses are usually obtained about 6 weeks after surgery. Pg. 1254

Following a modified radical mastectomy, the health care provider recommends chemotherapy even though the lymph nodes were negative for cancer cells. The patient tells the nurse that she does not know what to do about chemotherapy because she has heard that she may not even need chemotherapy and that the side effects are uncomfortable. The nursing diagnosis that best reflects the patient's problem is a. anxiety related to prospect of additional cancer therapy. b. fear related to uncomfortable side effects of chemotherapy. c. decisional conflict related to lack of knowledge about prognosis and treatment options. d. risk for ineffective health maintenance related to reluctance to consider additional treatment.

Answer: C Rationale: The patient's statements indicate that she is having difficulty making a decision about treatment because of a lack of understanding about prognosis and treatment. Although she may have some anxiety and fear, these are not the priorities at this time. The patient expresses concerns about chemotherapy rather than reluctance to consider additional treatment. Cognitive Level: Application Text Reference: p. 1356 Nursing Process: Diagnosis NCLEX: Psychosocial Integrity

Following a modified radical mastectomy, the health care provider recommends chemotherapy even though the lymph nodes were negative for cancer cells. The patient tells the nurse that she does not know what to do about chemotherapy because she has heard that she may not even need chemotherapy and that the side effects are uncomfortable. The nursing diagnosis that best reflects the patient's problem is a. anxiety related to prospect of additional cancer therapy. b. fear related to uncomfortable side effects of chemotherapy. c. decisional conflict related to lack of knowledge about prognosis and treatment options. d. risk for ineffective health maintenance related to reluctance to consider additional treatment.

Answer: C Rationale: The patient's statements indicate that she is having difficulty making a decision about treatment because of a lack of understanding about prognosis and treatment. Although she may have some anxiety and fear, these are not the priorities at this time. The patient expresses concerns about chemotherapy rather than reluctance to consider additional treatment. Pg. 1250

During the first 4 hours after TURP, the patient receives 1200 mL of bladder irrigation solution, and his urine output is 1000 mL. What is your priority intervention? A. Slowing the rate of bladder irrigation B. Continuing to observe the patient C. Checking catheter patency D. Encouraging oral fluids

Answer: C You should continuously monitor the inflow and outflow of the irrigant. If outflow is less than inflow, assess the catheter for kinks or clots. If the outflow is blocked and patency cannot be reestablished by manual irrigation, stop the CBI and notify the physician.

When working in the emergency department you recognize the need for immediate diagnosis and treatment of the patient with A. orchitis. B. phimosis. C. epididymitis. D. testicular torsion.

Answer: D Torsion constitutes a surgical emergency. If the blood supply to the affected testicle is not restored within 4 to 6 hours, testicular ischemia will occur, leading to necrosis and possible removal.

What should you explain to the patient who has had a vasectomy? A. The procedure blocks the production of sperm B. Erectile dysfunction is temporary and will return with sexual activity. C. The ejaculate will be about half the volume it was before the procedure. D. An alternative form of contraception will be necessary for 6 to 8 weeks.

Answer: D After a vasectomy, the patient should use an alternative form of contraception until semen examination reveals no sperm. This usually requires at least 10 ejaculations or 6 weeks to evacuate sperm distal to the surgical site.

Postoperatively, a patient who has had a laser prostatectomy has continuous bladder irrigation with a three-way urinary catheter with a 30-mL balloon. When he complains of bladder spasms with the catheter in place, you should A. deflate the catheter balloon to 10 mL to decrease bulk in the bladder. B. deflate the catheter balloon and then reinflate it to ensure that it is patent. C. encourage the patient to try to have a bowel movement to relieve colon pressure. D. explain that this feeling is normal and that he should not try to urinate around the catheter.

Answer: D Bladder spasms occur as a result of irritation of the bladder mucosa from the insertion of the resectoscope, presence of a catheter, or clots leading to obstruction of the catheter. Instruct the patient not to urinate around the catheter because this increases the likelihood of spasm.

You recognize the need to begin testosterone therapy when testosterone levels drop below A. 750 ng/dL. B. 500 ng/dL. C. 400 ng/dL. D. 250 ng/dL.

Answer: D Normal testosterone levels can range from 280 to 1100 ng/dL. Replacement therapy may be considered when levels are below 250 ng/dL.

A patient with an abnormal mammogram is scheduled for stereotactic core biopsy. Which information will the nurse include when teaching the patient about the procedure? a. "You will need to avoid eating or drinking anything for 6 hours before the procedure." b. "Any discomfort after the biopsy may be treated with mild pain relievers such as aspirin." c. "The core biopsy is evaluated immediately and you will get the results before leaving." d. "Several samples of tissue in the abnormal area will be obtained during the procedure."

Answer: D Rationale: During stereotactic breast biopsy, a biopsy gun is used to remove several core samples in the area of abnormality. The procedure is done using a local anesthetic, so there is no need to be NPO before the procedure. Aspirin should not be used because it will increase bleeding at the site. The biopsy is sent to pathology, and results are not usually available immediately. Cognitive Level: Application Text Reference: p. 1345 Nursing Process: Implementation NCLEX: Health Promotion and Maintenance

While the nurse is obtaining a nursing history from a 52-year-old patient who has found a small lump in her breast, which question is most pertinent? a. "Do you currently smoke cigarettes?" b. "Have you ever had any breast injuries?" c. "Is there any family history of fibrocystic breast changes?" d. "At what age did you start having menstrual periods?"

Answer: D Rationale: Early menarche and late menopause are risk factors for breast cancer because of the prolonged exposure to estrogen that occurs. Cigarette smoking, breast trauma, and fibrocystic breast changes are not associated with increased breast cancer risk. Cognitive Level: Application Text Reference: p. 1349 Nursing Process: Assessment NCLEX: Physiological Integrity

While the nurse is obtaining a nursing history from a 52-year-old patient who has found a small lump in her breast, which question is most pertinent? a. "Do you currently smoke cigarettes?" b. "Have you ever had any breast injuries?" c. "Is there any family history of fibrocystic breast changes?" d. "At what age did you start having menstrual periods?"

Answer: D Rationale: Early menarche and late menopause are risk factors for breast cancer because of the prolonged exposure to estrogen that occurs. Cigarette smoking, breast trauma, and fibrocystic breast changes are not associated with increased breast cancer risk. Pg. 1243

A 34-year-old woman has undergone a modified radical mastectomy for a breast tumor. The pathology report identified the tumor as a stage I, estrogen-receptor-positive adenocarcinoma. The nurse will plan on teaching the patient about a. raloxifene (Evista). b. estradiol (Estrace). c. trastuzumab (Herceptin). d. tamoxifen (Nolvadex).

Answer: D Rationale: Tamoxifen is used for estrogen-dependent breast tumors in premenopausal women. Raloxifene is used to prevent breast cancer, but it is not used post-mastectomy to treat breast cancer. Estradiol will increase the growth of estrogen-dependent tumors. Trastuzumab is used to treat tumors that have the HER-2/neu antigen. Cognitive Level: Application Text Reference: p. 1355 Nursing Process: Planning NCLEX: Physiological Integrity

A 34-year-old woman has undergone a modified radical mastectomy for a breast tumor. The pathology report identified the tumor as a stage I, estrogen-receptor-positive adenocarcinoma. The nurse will plan on teaching the patient about a. raloxifene (Evista). b. estradiol (Estrace). c. trastuzumab (Herceptin). d. tamoxifen (Nolvadex).

Answer: D Rationale: Tamoxifen is used for estrogen-dependent breast tumors in premenopausal women. Raloxifene is used to prevent breast cancer, but it is not used post-mastectomy to treat breast cancer. Estradiol will increase the growth of estrogen-dependent tumors. Trastuzumab is used to treat tumors that have the HER-2/neu antigen. Pg. 1251

The nurse provides discharge teaching for a patient who has had a left modified radical mastectomy and axillary lymph node dissection. The nurse determines that teaching has been successful when the patient says, a. "I should keep my left arm supported in a sling when I am up until my incision is healed." b. "I may expose my left arm to the sun for several hours each day to increase circulation and promote healing." c. "I can do whatever exercises and activities I want as long as I do not elevate my left hand above my head." d. "I will continue to exercise my left arm with finger-walking up the wall or combing my hair."

Answer: D Rationale: The patient should continue with arm exercises to regain strength and range of motion. The left arm should be elevated to the level of the heart when the patient is up. Sun exposure is avoided because of the risk of sunburn. The left hand should be elevated at or above heart level to reduce swelling and lymphedema. Cognitive Level: Application Text Reference: pp. 1359-1360 Nursing Process: Evaluation NCLEX: Physiological Integrity

The nurse provides discharge teaching for a patient who has had a left modified radical mastectomy and axillary lymph node dissection. The nurse determines that teaching has been successful when the patient says, a. "I should keep my left arm supported in a sling when I am up until my incision is healed." b. "I may expose my left arm to the sun for several hours each day to increase circulation and promote healing." c. "I can do whatever exercises and activities I want as long as I do not elevate my left hand above my head." d. "I will continue to exercise my left arm with finger-walking up the wall or combing my hair."

Answer: D Rationale: The patient should continue with arm exercises to regain strength and range of motion. The left arm should be elevated to the level of the heart when the patient is up. Sun exposure is avoided because of the risk of sunburn. The left hand should be elevated at or above heart level to reduce swelling and lymphedema. Pg. 1254

A male patient complains of fever, dysuria, and cloudy urine. What additional information may indicate that these manifestations may be something other than a urinary tract infection (UTI)? A. E. coli bacteria in his urine B. A very tender prostate gland C. Complaints of chills and rectal pain D. Complaints of urgency and frequency

B. A very tender prostate gland A tender and swollen prostate is indicative of prostatis, which is a more serious male reproductive problem because an acute episode can result in chronic prostatis and lead to epididymitis or cystitis. E. coli in his urine, chills and rectal pain, and urgency and frequency are all present with a UTI and not specifically indicative of prostatis.

The nurse has been asked to participate in a healthy living workshop. While teaching about women's health, which of the following guidelines should the nurse provide to the audience? A. "Mammograms are necessary if you have a family history of breast cancer." B. "It's recommended that you get a mammogram each year after you turn 40." C. "If you are not able to perform breast self-examination (BSE), you should go for regular mammograms." D. "You should ensure that your primary care provider performs a breast exam each time you visit."

B. Annual mammograms are recommended after age 40. They are recommended for all women, not solely those with a family history of breast cancer. BSE is not a replacement for mammography and clinical breast examinations are not necessary at each office visit.

A 24-year-old patient who has undergone breast augmentation earlier in the day will be discharged home in the early evening. What instructions should the nurse provide in order to minimize the patient's risk of complications in the immediate recovery period? A. Avoid wearing a bra until postoperative day 3. B. Ask the patient to avoid strenuous exercise during her recovery period. C. Sleep in a semi-Fowler's position until her scheduled follow-up appointment. D. Enlist a friend or family member to perform passive range-of-motion exercises.

B. As with all types of breast surgery, strenuous exercise is contraindicated during the recovery period following breast augmentation. A bra should be worn to prevent dehiscence. Passive range-of-motion exercises should be avoided, and sleeping in a semi-Fowler's position is not necessary

A 73-year-old male patient admitted for total knee replacement states during the health history interview that he has no problems with urinary elimination except that the "stream is less than it used to be." The nurse should give the patient anticipatory guidance that what condition may be developing? A. A tumor of the prostate B. Benign prostatic hyperplasia C. Bladder atony because of age D. Age-related altered innervation of the bladder

B. Benign prostatic hyperplasia Benign prostatic hyperplasia is an enlarged prostate gland because of an increased number of epithelial cells and stromal tissue. It occurs in about 50% of men over age 50 and 80% of men over age 80. Only about 16% of men develop prostate cancer. Bladder atony and age-related altered innervations of the bladder do not lead to a weakened stream.

A 70-year-old male patient has sought care because of recent difficulties in establishing and maintaining a urine stream as well as pain that occasionally accompanies urination. How should the nurse document this abnormal assessment finding? A. Anuria B. Dysuria C. Oliguria D. Enuresis

B. Dysuria Painful and difficult urination is characterized as dysuria. Anuria is an absence of urine production, whereas oliguria is diminished urine production. Enuresis is involuntary nocturnal urination.

A 45-year-old man reports having recent problems attaining an erection. Which medication will the nurse further explore as the possible etiology of this patient's sexual dysfunction? A. Furosemide (Lasix) B. Fluoxetine (Prozac) C. Clopidogrel (Plavix) D. Nitroglycerin (Nitrostat)

B. Fluoxetine (Prozac) Fluoxetine is a selective serotonin reuptake inhibitor used in the treatment of depression. A common adverse effect of this medication is sexual problems (impotence, delayed or absent orgasm, delayed or absent ejaculation, decreased sexual interest) in nearly 70% of men and women.

A nurse is admitting a patient with the diagnosis of advanced renal carcinoma. Based upon this diagnosis, the nurse will expect to find what clinical manifestations as the "classic triad" occurring in patients with renal cancer? A. Fever, chills, flank pain B. Hematuria, flank pain, palpable mass C. Hematuria, proteinuria, palpable mass D. Flank pain, palpable abdominal mass, and proteinuria

B. Hematuria, flank pain, palpable mass There are no characteristic early symptoms of renal carcinoma. The classic manifestations of gross hematuria, flank pain, and a palpable mass are those of advanced disease.

Which of the following tasks should the registered nurse delegate to nursing assistive personnel (NAP) during the care of a patient who has had recent transverse rectus abdominis musculocutaneous (TRAM) flap surgery? A. Document the condition of the patient's incisions. B. Mobilize the patient in a slightly hunched position. C. Change the patient's abdominal and chest dressings. D. Change the parameters of the patient-controlled analgesic (PCA) pump.

B. Mobilization of a postsurgical patient may be delegated, and the patient who has had a TRAM flap should not stand or walk fully erect, in order to minimize strain on the incisions. Changing dressings, assessing wounds, and reprogramming a PCA pump are not tasks amenable to delegation.

A 22-year-old patient's blood pressure at her physical done for her new job was 110/68. At the health fair two months later, her blood pressure is 154/96. What renal problem should the nurse be aware of that could contribute to this abrupt rise in blood pressure? A. Renal trauma B. Renal artery stenosis C. Renal vein thrombosis D. Benign nephrosclerosis

B. Renal artery stenosis Renal artery stenosis contributes to an abrupt rise in blood pressure, especially in people under 30 or over 50 years of age. Renal trauma usually has hematuria. Renal vein thrombosis causes flank pain, hematuria, fever, or nephrotic syndrome. Benign nephrosclerosis usually occurs in adults 30 to 50 years of age and is a result of vascular changes resulting from hypertension.

Which nursing intervention is most appropriate in providing care for an adult patient with newly diagnosed adult onset polycystic kidney disease (PKD)? A. Help the patient cope with the rapid progression of the disease. B. Suggest genetic counseling resources for the children of the patient. C. Expect the patient to have polyuria and poor concentration ability of the kidneys. D. Implement appropriate measures for the patient's deafness and blindness in addition to the renal problems.

B. Suggest genetic counseling resources for the children of the patient. PKD is one of the most common genetic diseases. The adult form of PKD may range from a relatively mild disease to one that progresses to chronic kidney disease. Polyuria, deafness, and blindness are not associated with PKD.

As a component of the head-to-toe assessment of a patient who has been recently transferred to the clinical unit, the nurse is preparing to palpate the patient's kidneys. How should the nurse position the patient for this assessment? A. Prone B. Supine C. Seated at the edge of the bed D. Standing, facing away from the nurse

B. Supine To palpate the right kidney, the patient is positioned supine, and the nurse's left hand is placed behind and supports the patient's right side between the rib cage and the iliac crest. The right flank is elevated with the left hand, and the right hand is used to palpate deeply for the right kidney. The normal-sized left kidney is rarely palpable because the spleen lies directly on top of it.

A 50-year-old patient is preparing to begin breast cancer treatment with tamoxifen (Nolvadex). Which of the following points should the nurse emphasize when teaching the patient about her new drug regimen? A. "You may find that your medication causes some breast sensitivity." B. "It's important that you let your care provider know about any changes in vision." C. "You'll find that this drug often alleviates some of the symptoms that accompany menopause." D. "It's imperative that you abstain from drinking alcohol after you begin taking tamoxifen."

B. Tamoxifen has the potential to cause cataracts and retinopathy. The drug is likely to exacerbate rather than alleviate perimenopausal symptoms. Breast tenderness is not associated with tamoxifen, and it is not necessary for the patient to abstain from alcohol.

The nurse is caring for a patient with a nephrostomy tube. The tube has stopped draining. After receiving orders, what should the nurse do? A. Keep the patient on bed rest. B. Use 5 mL of sterile saline to irrigate. C. Use 30 mL of water to gently irrigate. D. Have the patient turn from side to side.

B. Use 5 mL of sterile saline to irrigate. With a nephrostomy tube, if the tube is occluded and irrigation is ordered, the nurse should use 5 mL or less of sterile saline to gently irrigate it. The patient with a ureteral catheter may be kept on bed rest after insertion, but this is unrelated to obstruction. Only sterile solutions are used to irrigate any type of urinary catheter. With a suprapubic catheter, the patient should be instructed to turn from side to side to ensure patency.

A 32-year-old patient has oral contraceptives prescribed for endometriosis. The nurse will teach the patient to a. expect to experience side effects such as facial hair. b. take the medication every day for the next 9 months. c. take calcium supplements to prevent developing osteoporosis during therapy. d. use a second method of contraception to ensure that she will not become pregnant.

B. When oral contraceptives are prescribed to treat endometriosis, the patient should take the medications continuously for 9 months. Facial hair is a side effect of synthetic androgens. The patient does not need to use additional contraceptive methods. The hormones in oral contraceptives will protect against osteoporosis

A 32 year old pt has stage IIB breast cancer that is estrogen receptor positive. What medication should the nurse avoid administering to the patient? A. diethylstilbestrol (DES) B. exemestane (Aromasin) C. trastuzumab (Herceptin) D. doxorubicin (Adriamycin)

B. exemestane (Aromasin) Is an aromatase inhibitor is used in the treatment of postmenopausal women. This drug interferes with the enzyme that helps synthesize endogenous estrogen but does not block the production of ovarian estrogen. Aromatase inhibitors are of little benefir in premenopausal women with breast cancer.

A nurse is teaching a health promotion workshop to a group of women in their 40s and 50s. Which of the following points about nipple discharge should the nurse teach to participants? A. Inappropriate lactation necessitates breast biopsy. B. Nipple discharge of any type is considered a precursor to cancer. C. Unexpected nipple discharge of any type warrants medical follow-up. D. Galactorrhea is a normal age-related change and a frequent perimenopausal symptom.

C. Although most cases of nipple discharge are not related to malignancy, further assessment is indicated. Galactorrhea is not considered a normal age-related change, nor is it a common perimenopausal symptom.

The nurse teaches a 30-year-old man with a family history of prostate cancer about dietary factors associated with prostate cancer. The nurse determines that teaching is successful if the patient selects which menu? A. Grilled steak, French fries, and vanilla shake B. Hamburger with cheese, pudding, and coffee C. Baked chicken, peas, apple slices, and skim milk D. Grilled cheese sandwich, onion rings, and hot tea

C. Baked chicken, peas, apple slices, and skim milk A diet high in red meat and high-fat dairy products along with a low intake of vegetables and fruits may increase the risk of prostate cancer.

An older male patient visits his primary care provider because of burning on urination and production of urine that he describes as "foul smelling." The health care provider should assess the patient for what factor that may put him at risk for a urinary tract infection (UTI)? A. High-purine diet B. Sedentary lifestyle C. Benign prostatic hyperplasia (BPH) D. Recent use of broad-spectrum antibiotics

C. Benign prostatic hyperplasia (BPH) BPH causes urinary stasis, which is a predisposing factor for UTIs. A sedentary lifestyle and recent antibiotic use are unlikely to contribute to UTIs, whereas a diet high in purines is associated with renal calculi.

The nurse is caring for a patient diagnosed with breast cancer who just underwent an axillary lymph node dissection. Which of the following interventions would the nurse use to decrease the lymphedema? A. Keep affected arm flat at the patient's side. B. Apply an elastic bandage on the affected arm. C. Assess blood pressure on unaffected arm only. D. Restrict exercise of the affected arm for 1 week.

C. Blood pressure readings, venipunctures, and injections should not be done on the affected arm. Elastic bandages should not be used in the early postoperative period because they inhibit collateral lymph drainage. The affected arm should be elevated above the heart, and isometric exercises are recommended to reduce fluid volume in the arm.

To accurately monitor progression of a symptom of decreased urinary stream, the nurse should encourage the patient to have which primary screening measure done on a regular basis? A. Uroflowmetry B. Transrectal ultrasound C. Digital rectal examination (DRE) D. Prostate-specific antigen (PSA) monitoring

C. Digital rectal examination (DRE) Digital rectal examination is part of a regular physical examination and is a primary means of assessing symptoms of decreased urinary stream, which is often caused by benign prostatic hyperplasia in men over 50 years of age. The uroflowmetry helps determine the extent of urethral blockage and the type of treatment needed but is not done on a regular basis. Transrectal ultrasound is indicated with an abnormal DRE and elevated PSA to differentiate between BPH and prostate cancer. The PSA monitoring is done to rule out prostate cancer, although levels may be slightly elevated in patients with BPH.

A 29-year-old primiparous patient has a 3-week-old infant whom she is breastfeeding. The woman has sought care because of recent breast tenderness, redness, and fever. Which of the following teaching points should the nurse prioritize when following up her care? A. Encourage patient to continue breastfeeding her infant. B. Refer patient for a mammogram as quickly as possible. C. Ensure patient adheres to her prescribed antibiotic regimen. D. Teach patient to use warm compresses and educate her about self-limiting nature of illness.

C. Mastitis normally requires antibiotic therapy, the success of which is often dependent on close adherence to the prescribed regimen. Breastfeeding should indeed be continued if possible, but effective treatment of her infection would be the immediate priority. Mastitis is not necessarily self limiting and mammography is not normally indicated.

The patient has had cardiovascular disease for some time and has now developed erectile dysfunction. He is frustrated because he cannot take erectogenic medications because he takes nitrates for his cardiac disease. What should the nurse do first to help this patient? A. Give the patient choices for penile implant surgery. B. Recommend counseling for the patient and his partner. C. Obtain a thorough sexual, health, and psychosocial history. D. Assess levels of testosterone, prolactin, LH, and thyroid hormones.

C. Obtain a thorough sexual, health, and psychosocial history. The nurse's first action to help this patient is to obtain a thorough sexual, health, and psychosocial history. Alternative treatments for the cardiac disease would then be explored if that had not already been done. Further examination or diagnostic testing would be based on the history and physical assessment, including hormone levels, counseling, or penile implant options.

When caring for a patient with nephrotic syndrome, the nurse should know the patient understands dietary teaching when the patient selects which food item? A. Peanut butter and crackers B. One small grilled pork chop C. Salad made of fresh vegetables D. Spaghetti with canned spaghetti sauce

C. Salad made of fresh vegetables Of the options listed, only salad made with fresh vegetables would be acceptable for the diet that limits sodium and protein as well as saturated fat if hyperlipidemia is present. Peanut butter and crackers are processed so they contain significant sodium, and peanut butter contains some protein. A pork chop is a high-protein food with saturated fat. Canned spaghetti sauce is also high in sodium.

The patient has a low-grade carcinoma on the left lateral aspect of the prostate gland and has been on "watchful waiting" status for 5 years. Six months ago his last prostate-specific antigen (PSA) level was 5 ng/mL. Which manifestations now indicate that the prostate cancer may be growing and he needs a change in his care (select all that apply)? A. Casts in his urine B. Presence of α-fetoprotein C. Serum PSA level 10 ng/mL D. Onset of erectile dysfunction E. Nodularity of the prostate gland

C. Serum PSA level 10 ng/mL E. Nodularity of the prostate gland The manifestations of increased PSA level along with the new nodularity of the prostate gland potentially indicate that the tumor may be growing. Casts in the urine, presence of α-fetoprotein, and new onset of erectile dysfunction do not indicate prostate cancer growth.

A patient is one day postoperative following a transurethral resection of the prostate (TURP). Which event is not an expected normal finding in the care of this patient? A. The patient requires two tablets of Tylenol #3 during the night. B. The patient complains of fatigue and claims to have minimal appetite. C. The patient has continuous bladder irrigation (CBI) infusing, but output has decreased. D. The patient has expressed anxiety about his planned discharge home the following day.

C. The patient has continuous bladder irrigation (CBI) infusing, but output has decreased. A decrease or cessation of output in a patient with CBI requires immediate intervention. The nurse should temporarily stop the CBI and attempt to resume output by repositioning the patient or irrigating the catheter. Complaints of pain, fatigue, and low appetite at this early postoperative stage are not unexpected. Discharge planning should be addressed, but this should not precede management of the patient's CBI.

Which effect of aging on the urinary system is most likely to affect the action of bumetanide (Bumex)? A. Benign enlargement of prostatic tissues B. Decreased sensation of bladder capacity C. Decreased function of the loop of Henle D. Less absorption in the Bowman's capsule

C.Decreased function of the loop of Henle Bumetanide (Bumex) is a loop diuretic that acts in the loop of Henle to decrease reabsorption of sodium and chloride. Because the loop of Henle loses function with aging, the excretion of drugs becomes less and less efficient. Thus the circulating levels of drugs are increased and their effects prolonged. The benign enlargement of prostatic tissue, decreased sensation of bladder capacity, and loss of concentrating ability do not directly affect the action of loop diuretics.

A 51-year-old woman has recently had a unilateral, right total mastectomy and axillary node dissection for the treatment of breast cancer. Which of the following interventions should the nurse include in the patient's care? A. Immobilize the patient's right arm until postoperative day 3. B. Maintain the patient's right arm in a dependent position when at rest. C. Administer diuretics prophylactically for the prevention of lymphedema. D. Promote gradually increasing mobility as soon as possible following surgery.

D. Mobility should be encouraged beginning in postanesthetic recovery and increased gradually throughout the patient's recovery. Immobilization is counterproductive to recovery and the limb should never be in a dependent position. Diuretics are not used to prevent lymphedema but may be used in active treatment of the problem.

Which task can the nurse delegate to an unlicensed assistive personnel (UAP) in the care of a patient who has recently undergone prostatectomy? A. Assessing the patient's incision B. Irrigating the patient's Foley catheter C. Assessing the patient's pain and selecting analgesia D. Performing cleansing of the meatus and perineal region

D. Performing cleansing of the meatus and perineal region Performing perineal care is an appropriate task for delegation. Selecting analgesia, irrigating the patient's catheter, and assessing his incision are not appropriate skills or tasks for unlicensed personnel.

When the patient reports acute, severe, renal colic pain in the lower abdomen, the nurse knows that the patient is most likely to have an obstruction at which area? A.Kidney B. Urethra C. Bladder D. Ureterovesical junction

D. Ureterovesical junction The ureterovesical junction (UVJ) is the narrowest part of the urethra and easily obstructed by urinary calculi. With a stone in the kidney or at the ureteropelvic junction (UPJ), the pain may be dull costovertebral flank pain. Stones in the bladder do not cause obstruction or symptoms unless they are staghorn stones. The urethra seldom has obstruction related to stones.

Which urinalysis result should the nurse recognize as an abnormal finding? A. pH 6.0 B. Amber yellow color C. Specific gravity 1.025 D. White blood cells (WBCs) 9/hpf

D. White blood cells (WBCs) 9/hpf Correct Normal WBC levels in urine are below 5/hpf, with levels exceeding this indicative of inflammation or urinary tract infection. A urine pH of 6.0 is average; amber yellow is normal coloration, and the reference ranges for specific gravity are 1.003 to 1.030.

The patient is wondering why anesthesia is needed when the lithotripsy being done is noninvasive. The nurse explains that the anesthesia is required to ensure the patient's position is maintained during the procedure. The nurse knows that this type of lithotripsy is called A. laser lithotripsy. B. electrohydraulic lithotripsy. C. percutaneous ultrasonic lithotripsy. D. extracorporeal shock-wave lithotripsy (ESWL).

D. extracorporeal shock-wave lithotripsy (ESWL). ESWL is noninvasive, but anesthesia is used to maintain the patient's position. The other types of lithotripsy are invasive. Laser lithotripsy uses an ureteroscope and small fiber to reach the stone. Electrohydraulic lithotripsy positions a probe directly on the stone; then continuous saline irrigation flushes are used to rinse the stone out. Percutaneous ultrasonic lithotripsy places an ultrasonic probe in the renal pelvis via a percutaneous nephroscope inserted through an incision in the flank.

ANS: B, D, E Patients are taught that shedding of the virus and infection of sexual partners can occur even in asymptomatic periods, that recurrent episodes resolve more quickly, and that sitz baths can be used to relieve pain caused by the lesions. Antiviral medications decrease the number of outbreaks, but do not cure herpes simplex infections. Infected areas may be kept dry if this decreases pain and itching.

Which topics will the nurse include when preparing to teach a patient with recurrent genital herpes simplex (select all that apply)? a. Infected areas should be kept moist to speed healing. b. Sitz baths may be used to relieve discomfort caused by the lesions. c. Genital herpes can be cured by consistent use of antiviral medications. d. Recurrent genital herpes episodes usually are shorter than the first episode. e. The virus can infect sexual partners even when you do not have symptoms of infection.

What do you tell the patient with chronic bacterial prostatitis who is undergoing antibiotic therapy (select all that apply)? A. Hospitalization is required of all patients. B. Pain will lessen after treatment has ended. C. The course of treatment usually is 2 to 4 weeks. D. Long-term therapy may be indicated for immunocompromised patients. E. If the condition is unresolved and untreated, the patient is at risk for prostate cancer.

answer: B, D Patients with chronic bacterial prostatitis are usually given antibiotics for 4 to 12 weeks. However, antibiotics may be given for a lifetime if the patient is immunocompromised. Although patients with chronic bacterial prostatitis tend to experience a great amount of discomfort, the pain resolves as the infection is treated. If the patient with acute bacterial prostatitis has high fever or other signs of impending sepsis, hospitalization and intravenous antibiotics are prescribed.

*3. The immunologic mechanisms involved in acute poststreptococal glomerulonephritis include:* a. tubular blocking by precipitates of bacteria and antibody reactions b. deposition of immune complexes and complement along the GBM c. thickening of the GBM from autoimmune microangiopathic changes d. destruction of glomeruli by proteolytic enzymes contained in the GBM

b. deposition of immune complexes and complement along the GBM


Kaugnay na mga set ng pag-aaral

Chapter 16- Innate Immunity: Nonspecific Defenses of the Host

View Set

Accounting chapter 17 special journals: purchases an cash payments

View Set

Trigonometric Graphs (Precalculus)

View Set

Completa Oracion con el Verbo/ El imperativo Informal

View Set

Male Repro Pot (Pharm on last pages)

View Set